NTA CUET (UG) 10 Mock Test Sample Question Papers Biology (2024) 9789357288095, 9357288090

Description of the product: • 100% Exam Ready With 2023 CUET(UG) Exam Papers (2 Slots) – Fully Solved with Explanations

125 98 23MB

English Pages 139 [138] Year 2023

Report DMCA / Copyright

DOWNLOAD PDF FILE

Table of contents :
Cover page
Copyright
Contents
Oswaal Books Expert Tips to Crack CUET (UG) in the first Attempt
Latest CUET Syllabus
CUET Solved Paper 2023 (25th May 2023)
CUET Solved Paper, 2022 (18th August Slot-1, 20th August Slot-1 &
30th August Slot-2)
CUET Solved Paper 2021 (23rd Sept. 2021 Shift-2 UIQP02)
Sample Question Papers
Sample Question Paper - 1
Sample Question Paper - 2
Sample Question Paper - 3
Sample Question Paper - 4
Sample Question Paper - 5
Sample Question Paper - 6
Sample Question Paper - 7
Sample Question Paper - 8
Sample Question Paper - 9
Sample Question Paper - 10
Solutions
Sample Question Paper - 1
Sample Question Paper - 2
Sample Question Paper - 3
Sample Question Paper - 4
Sample Question Paper - 5
Sample Question Paper - 6
Sample Question Paper - 7
Sample Question Paper - 8
Sample Question Paper - 9
Sample Question Paper - 10
Recommend Papers

NTA CUET (UG) 10 Mock Test Sample Question Papers Biology (2024)
 9789357288095, 9357288090

  • 0 0 0
  • Like this paper and download? You can publish your own PDF file online for free in a few minutes! Sign Up
File loading please wait...
Citation preview

For 2024 Exam

BEST SELLER

BIOLOGY Section II (Domain Specific Subject) Strictly ar per the Latest Examination Pattern issued by NTA

The ONLY book you need to Ace CUET (UG)

1

2

3

4

5

Crisp Revision

Valuable Exam Insights

100% Exam Readiness

Extensive Practice

Concept Clarity

With On-Tips Notes

With Latest Solved

With 10 Solved

With 650+ NCERT

With 450+

& Updated Mind Maps

Paper 2023

Sample Question Papers

- based MCQs

Explanations & Smart Answer Keys

(i)

3rd EDITION

ISBN SYLLABUS COVERED

YEAR 2023-24 “9789357288095”

CUET (UG) CERTIFICATE OF COMMON UNIVERSITY ENTRANCE TEST

PUBLISHED BY

C OPYRIG HT

RESERVED BY THE PUBLISHERS

All rights reserved. No part of this book may be reproduced, stored in a retrieval system, or transmitted, in any form or by any means, without written permission from the publishers. The author and publisher will gladly receive information enabling them to rectify any error or omission in subsequent editions.

OSWAAL BOOKS & LEARNING PVT. LTD. 1/11, Sahitya Kunj, M.G. Road, Agra - 282002, (UP) India

1010, Cambourne Business Centre Cambridge, Cambridgeshire CB 236DP, United kingdom

0562-2857671

[email protected]

www.OswaalBooks.com

DI SC L A IMER

This book is published by Oswaal Books and Learning Pvt Ltd (“Publisher”) and is intended solely for educational use, to enable students to practice for examinations/tests and reference. The contents of this book primarily comprise a collection of questions that have been sourced from previous examination papers. Any practice questions and/or notes included by the Publisher are formulated by placing reliance on previous question papers and are in keeping with the format/pattern/ guidelines applicable to such papers. The Publisher expressly disclaims any liability for the use of, or references to, any terms or terminology in the book, which may not be considered appropriate or may be considered offensive, in light of societal changes. Further, the contents of this book, including references to any persons, corporations, brands, political parties, incidents, historical events and/or terminology within the book, if any, are not intended to be offensive, and/or to hurt, insult or defame any person (whether living or dead), entity, gender, caste, religion, race, etc. and any interpretation to this effect is unintended and purely incidental. While we try to keep our publications as updated and accurate as possible, human error may creep in. We expressly disclaim liability for errors and/or omissions in the content, if any, and further disclaim any liability for any loss or damages in connection with the use of the book and reference to its contents”.

Kindle( Edition ii )

Preface National Testing Agency (NTA) has been established in November 2017 under the Societies Registration Act (1860) by the Ministry of Education as a premier, specialist, autonomous, and self-sustained testing organization to conduct entrance examinations for admission/fellowship in higher educational institutions. The Common University Entrance Test (CUET (UG) - 2022) is being introduced for admission into all UG Programmes in all Central Universities for the academic session 2023 under the Ministry of Education, (MoE). The Common University Entrance Test (CUET) will provide a common platform and equal opportunities to candidates across the country, especially those from rural and other remote areas, and help establish a better connection with the Universities. A single examination will enable the Candidates to cover a wide outreach and be part of the admissions process to various Central Universities. CUET – UG Computer Based Test (CBT) for the Central Universities is to be conducted by the National Testing Agency (NTA). The curriculum for CUET is based on the National Council of Educational Research and Training (NCERT) syllabus for class 12 only. CUET scores are mandatorily required while admitting students to undergraduate courses in 44 central universities. A merit list will be prepared by participating Universities/organizations. Universities may conduct their individual counselling on the basis of the scorecard of CUET (UG)-2023 provided by NTA.

A few benefits of studying from Oswaal Sample Question Papers • • • • •

Crisp Revision With On-Tips Notes & updated Mind Maps Valuable Exam Insights With Latest Solved Papers 2023 100% Exam Readiness With 10 Solved Sample Question Papers Extensive Practice With 550+ NCERT - based MCQs Concept Clarity With 350+ Explanations & Smart Answer Keys

Our Heartfelt Gratitude! Finally, we would like to thank our authors, editors, and reviewers. Special thanks to our students who send us suggestions and constantly help improve our books. We promise to always strive towards ‘Making Learning Simple’ for all of you. Wish you all Happy Learning!

( iii )

-Team Oswaal Books

Oswaal Books Expert Tips to Crack Oswaal BooksCUET Expert (UG) Tips toinCrack in the First A empt the CUET First (UG) Attempt Excited about your UG but unsure if you will get admission to your preferred university? In a major announcement by the chairman of the University Grants Commission, the Naonal Tesng Agency will be conducng the Common Universies Entrance Test (CUET (UG) 2022) for undergraduate programs in Central Universies for the upcoming academic session. However, the UGC Chairperson also stated that CUET (UG) will not just be limited to admissions to Central Universies. Many prominent private universies have indicated that they would also like to adopt a common entrance exam for undergraduate admissions and take admissions on the basis of CUET (UG) scores. This makes CUET (UG) a very important examinaon in itself and hence it becomes mandatory to be aware of the ps & tricks that could help you ace the exam on the first a† empt.

The first step is to understand The pa† ern of the examinaon. CUET includes three secons, secon 1 includes queson based on languages, secon 2 includes 27 domain-based subjects and secon 3 includes General Test. The syllabus of the upcoming Common University Entrance Test, CUET 2022, will be completely based on the syllabus of class 12 th . No queson will be asked from class 11th syllabus.

While preparing for the exam, it is i m p o r t a n t t o i d e n f y t h e important topics and pracce important quesons from those t o p i c s . P r a c c e i m p o r t a n t q u e s o n s t h r o u g h O s w a a l Q u e so n B a n k a n d S a m p l e Q u e s o n P a p e r s , L i s n g topics also helps in idenfying the weak areas that need special effort and me. The aspirants can start preparing to focus on the areas that they consider to be tough, followed by the ones that are their strengths.

Devote a sufficient amount of me to all the secons of the examinaons. This requires a wellmade plan and an honest adherence to the said plan. Priorize the most important topics or the topics that the aspirants are not familiar with to be able to master them in me.

Make a habit of preparing notes f ro m t h e b e g i n n i n g o f t h e preparaon. It will not only help in making the study systemac but also make the revision of the syllabus easy even when you might have limited me to revise.

Collecng and preparing from the appropriate study material cannot be ignored as irrelevant. The books chosen by the aspirants to study from should be on the lines of the current syllabus and the ones that could help you with swi• revision before the examinaon.

Make sure to revise as much as possible. The revision will help the aspirants in keeping the concepts fresh in their minds unl the day of the final examinaons. They may refer to a few good pracce quesons and concise revision notes to achieve their desired results.

With this said, an important queson that is gaining ground amongst students who will be appearing for this exam is if they should take coaching to get themselves ready for the exams. The answer is a simple no, the exam will simply not require any coaching as it is completely based on the Class 12th syllabus which will be quite fresh in students' minds as they will be just out of school. All they need is a good revision and pracce of quesons from Oswaal Queson Bank and Sample Queson Papers for CUET (UG) preparaons.

Contents l Oswaal Books Expert Tips to Crack CUET (UG) in the first Attempt l Latest CUET Syllabus

l CUET Solved Paper 2023 (25th May 2023)

l CUET Solved Paper, 2022 (18th August Slot-1, 20th August Slot-1 &



30th August Slot-2)

l CUET Solved Paper 2021 (23rd Sept. 2021 Shift-2 UIQP02)

iv - iv

vii - vii

2 - 10

11 - 36 37 - 40

Sample Question Papers

1 - 4 5 - 7 8 - 10 11 - 14 15 - 17 18 - 21 22 - 25 26 - 29 30 - 33 34 - 37

l Sample Question Paper - 1 l Sample Question Paper - 2 l Sample Question Paper - 3 l Sample Question Paper - 4 l Sample Question Paper - 5 l Sample Question Paper - 6 l Sample Question Paper - 7 l Sample Question Paper - 8 l Sample Question Paper - 9 l Sample Question Paper - 10

Solutions 38 - 42 43 - 47 48 - 52 53 - 58 59 - 63 64 - 68 69 - 73 74 - 78 79 - 83 84 - 88

l Sample Question Paper - 1 l Sample Question Paper - 2 l Sample Question Paper - 3 l Sample Question Paper - 4 l Sample Question Paper - 5 l Sample Question Paper - 6 l Sample Question Paper - 7 l Sample Question Paper - 8 l Sample Question Paper - 9 l Sample Question Paper - 10 



(v)

Positive Affirmations

I courageuosly move in the direction of my dreams at I e wh ol z i l a ntr I re ot Co d cann the goo t e l d w an s flo thing

I am Sup porte fully by th d e universe

I am wra pp loving en ed in the ergy of universe the I allow m y desires to flow to me now

Affirmations for the new

"YOU"

sly I effortles y m attract s desire

Accept yourself, love yourself, and keep moving forward. If you want to fly, you have to give up what weighs you down.

All of my thoughts are aligned with m y desires

en to I am op riences e p new ex lcome and we ce into n abunda my life

I have the p ower to shift my m indset and see the goo d in everything .

o, I create When I let g mething space for so e bett r.

( vi )

Latest Syllabus BIOLOGY - 304 Note:

There will be one Question Paper which will have 50 questions out of which 40 questions need to be attempted. BIOLOGY/BIOLOGICAL STUDIES/BIOTECNOLOGY/BIOCHEMISTRY

contribution, Modern Synthetic theory of Evolution; Mechanism of evolution– Variation (Mutation and Recombination) and Natural Selection with examples, types of natural selection; Gene flow and genetic dirft; HardyWeinberg’s principle;Adaptive Radiation; Human evolution.

Unit I: Reproduction

Reproduction in organisms: Reproduction, a characteristic feature of all organisms for continuation of species; Modes of reproduction – Asexual and sexual; Asexual reproduction; Modes- Binary fission, sporulation, budding, gemmule, fragmentation; vegetative propagation in plants. Sexual reproduction in flowering plants: Flower structure; Development of male and female gametophytes; Pollination– types, agencies and examples; Outbreedings devices; PollenPistil interaction; Double fertilization; Post fertilization events– Development of endosperm and embryo, Development of seed and formation of fruit; Special modes– apomixis, parthenocarpy, polyembryony; Significance of seed and fruit formation. Human Reproduction: Male and female reproductive systems; Microscopic anatomy of testis and ovary; Gametogenesis- spermatogenesis & oogenesis; Menstrual cycle; Fertilisation, embryo development upto blastocyst formation, implantation; Pregnancy and placenta formation (Elementary idea); Parturition (Elementary idea); Lactation (Elementary idea). Reproductive health: Need for reproductive health and prevention of sexually transmitted diseases (STD); Birth control- Need and Methods, Contraception and Medical Termination of Pregnancy (MTP); Amniocentesis; Infertility and assisted reproductive technologies – IVF, ZIFT, GIFT (Elementary idea for general awareness).

Unit II: Genetics and Evolution





Heredity and variation: Mendelian Inheritance; Deviations from Mendelism– Incomplete dominance, Co-dominance, Multiple alleles and Inheritance of blood groups, Pleiotropy; Elementary idea of polygenic inheritance; Chromosome theory of inheritance; Chromosomes and genes; Sex determination– In humans, birds, honey bee; Linkage and crossing over; Sex linked inheritance- Haemophilia, Colour blindness; Mendelian disorders in humans– Thalassemia; Chromosomal disorders in humans; Down’s syndrome, Turner’s and Klinefelter’s syndromes. Molecular Basis of Inheritance: Search for genetic material and DNA as genetic material; Structure of DNA and RNA; DNA packaging; DNA replication; Central dogma; Transcription, genetic code, translation; Gene expression and regulation– Lac Operon; Genome and human genome project; DNA finger printing. Evolution: Origin of life; Biological evolution and evidences for biological evolution (Paleontological, comparative anatomy, embryology and molecular evidence); Darwin’s

Unit III: Biology and Human Welfare



Health and Disease: Pathogens; parasites causing human diseases (Malaria, Filariasis, Ascariasis, Typhoid, Pneumonia, common cold, amoebiasis, ring worm); Basic concepts of immunology–vaccines; Cancer, HIV and AIDs; Adolescence, drug and alcohol abuse. Improvement in food production: Plant breeding, tissue culture, single cell protein, Biofortification; Apiculture and Animal husbandry. Microbes in human welfare: In household food processing, industrial production, sewage treatment, energy generation and as biocontrol agents and biofertilizers.

Unit IV: Biotechnology and Its Applications

Principles and process of Biotechnology: Genetic engineering (Recombinant DNA technology). Application of Biotechnology in health and agriculture: Human insulin and vaccine production, gene therapy; Genetically modified organisms- Bt crops; Transgenic Animals; Biosafety issues– Biopiracy and patents.

Unit V: Ecology and environment

Organisms and environment: Habitat and niche; Population and ecological adaptations; Population interactions– mutualism, competition, predation, parasitism; Population attributes–growth, birth rate and death rate, age distribution. Ecosystems: Patterns, components; productivity and decomposition; Energy flow; Pyramids of number, biomass, energy; Nutrient cycling (carbon and phosphorous); Ecological succession; Ecological Services– Carbon fixation, pollination, oxygen release. Biodiversity and its conservation: Concept of Biodiversity; Patterns of Biodiversity; Importance of Biodiversity; Loss of Biodiversity; Biodiversity conservation; Hotspots, endangered organisms, extinction, Red Data Book, biosphere reserves, National parks and sanctuaries. Environmental issues: Air pollution and its control; Water pollution and its control; Agrochemicals and their effects; Solid waste management; Radioactive waste management; Greenhouse effect and global warming; Ozone depletion; Deforestation; Any three case studies as success stories addressing environmental issues.



Don't Stop Reading !

You never know what might be asked in the exam. To download Chapter-wise Mind Maps scan the code below

To download On Tips Notes scan the code below

SCAN

( vii )

Exclusive School Books Suppliers VIJAYAWADA WEST KAMENG BANGLORE RAJKOT

MAHARASHTRA

ANDHRA PRADESH

Sri Vikas Book Centre, 9848571114, 9440715700,

PUNE

ASSAM

JALNA

Dutta Book Stall, 8729948473

KARNATAKA

CHENNAI

Satish Agencies, 8861630123

GUJRAT

HYDERABAD

Royal Stationers, 9824207514

KOLKATA

ANDHRA PRADESH

GUWAHATI

PATNA

INDORE

Akshaya Books Corner, 9666155555

Bookmark-IT, 7305151653

TELANGANA

Sri Balaji Book Depot , 9676996199, (040) 27613300

WEST BENGAL

United Book House, 9831344622

Bhaiya Industries, 9893326853, Sushil Prakashan,(0731) 2503333, 2535892, 9425322330, Bhaiya Store, 9425318103, Arun Prakashan, 9424890785, Bhaiya Book Centre, 9424081874, Seva Suppliers, 9826451052

ANDAMAN & NICOBAR ISLAND PORTBLAIR

Anil Paper Mart, 9422722522, (02482) 230733

TAMIL NADU

Our Distributors

VISAKHAPATHAM JBD Educational, 9246632691, 9246633693, Sri Rajeshwari Book Link, 9848036014 VIJAYAWADA

Madhusheela Books & Stationery, 7875899892

Krishna Book Centre, 9474205570, Kumar Book Depot, 9932082455, Kumar Book Depot, 9932082455, Sree aditya Book Centre, 8332972720, 7013300914

ASSAM

Book Emporium, 9675972993, 6000763186, Ashok Publication, 7896141127, Kayaan Enterprises, (0361) 2630443, Orchid Book house, 9864624209, Newco, 9864178188

JABALPUR

Vinay Pustak Sadan, 8962362667, Anand Books and Stationers, 9425323508

SAGAR

Princi Book Depot, Sagar, 9977277011

KATNI

Shri Mahavir Agency, 9425363412

UJJAIN BHOPAL

BIHAR

Nova Publisher & Distributors, (0612) 2666404, Shri Durga Pustak Mandir, 9334477386, Sharda Pustak Bhandar, 9334259293, Vikas Book Depot, 9504780402, Alka Book Agency, 9835655005, Metro Book(E&C), Ishu Pustak Bhandar, 8294576789, Gyan Ganga Limited, 6203900312, Ishu Pustak Bhandar, ( E & C ), 9334186300/8294576789

PUNE

Shreenath Book Depot, 9827544045 Gupta Brother, 9644482444

MAHARASHTRA

Natraj Book Depot, (020) 24485054, 9890054092, Vikas Book House, 9921331187, Pravin Sales, 9890683475, New Saraswati Granth Bhandar, 9422323859, Akshar Books & Stationary, 7385089789, Vardhaman Educational, 9860574354, Yash Book Centre, 9890156763, Pragati Book Centre, (ISC), 9850039311, Praveen Sales, Pragati Book Centre, Pune ( E & C ), 9850039311 Shree Sainath Agencies, 7350294089, Maya Book Centre, (ISC), 9372360150 Vidyarthi Sales Agencies, 9819776110, New Student Agencies, 7045065799, Shivam Books & Stationery, 8619805332

Pustak Bhandar, 7870834225

CHATTISGARH

AURANGABAD MUMBAI

AMBIKAPUR

Saini Brothers, 9425582561, M.P Department Stores, 9425254264

JALGAON

BOKARO BHILAI

Bokaro Student Friends Pvt. Ltd, Bokaro, 7277931285 Anil Book Depot, 9425234260

LATUR KOLHAPUR

Yash Book House, 9637936999, Shri Ganesh Pustakalay, 9730172188 Granth the Book World, 9922295522

KORBA

Kitab Ghar, Korba ( E & C ), 9425226528

NANDED

MUZAFFARPUR

DURG

RAIPUR RAIGARH DELHI

Bhagwati Bhawani Book Depot, 0788-2327620, 9827473100

NAGPUR

Sharma Book Depot & Stat. (ISC), 9421393040

Laxmi Pustakalay and Stationers, (0712) 2727354, Vijay Book Depot, 9860122094

Shri Ramdev Traders, 9981761797, Gupta Pustak Mandir, 7974220323, Anil Publication, 9691618258/7999078802

NASHIK

Renuka Book distributor, 9765406133, Novelty Book Depot, 9657690220, Karamveer Book Depot, 9923966466, Arun Book & Stationers, 9423110953 Abhang Pustakalaya, 9823470756/9175940756 Rahul Book Centre, 9970849681, New India Book House, 9623123458

DELHI

YAVATMAL

Shri Ganesh Pustkalaya, 9423131275

Sindhu Book Deopt, 9981935763

DHULE

Mittal Books, (011) 23288887, 9899037390, Singhania Book & Stationer, 9212028238, AoneBooks, New Delhi, 8800497047, Radhey Book Depot, 9818314141, Batheja Super Store, 9871833924, Lov Dev & Sons, Delhi ( E & C ), 9811182352, Zombozone, 9871274082, LDS Marketing, 9811182352/9999353491

VASAI

Navjeevan Book Stall, 7020525561

Prime Book Centre, Vasai, 9890293662

ODISHA A. K. Mishra Agencies, 9437025991, 9437081319

GUJARAT

CUTTACK

BHAVNAGAR DAHOD VAPI

Patel Book, 9898184248, 9824386112, 9825900335, Zaveri Agency, 9979897312, 9979890330, Hardik Book Agency, (ISC) 079-24110043, 9904659821 Samir Book Stall, Bhavnagar (ISC) 9586305305 Collegian Book Corner, 9925501981 Goutam Book Sellers, 9081790813

BHUBANESHWAR M/s Pragnya, 8847888616, 9437943777, Padmalaya, 9437026922, Bidyashree, 9937017070, Books Godown, 7894281110 BARIPADA Trimurti Book World, 9437034735 KEONJHAR Students corner, 7008435418

VALSAD NAVSARI

Mahavir Stationers, 9429474177 College Store, (ISC) NO CALL 02637-258642, 9825099121

AMBALA PATIALA

VADODARA

Umakant Book Sellers & Stationer, 9624920709

HARYANA

FEROZPUR LUDHIANA

ROHTAK

Manish Traders, 9812556687, Swami Kitab Ghar, 9355611088,

CHANDIGARH

REWARI

Sanjay book depot, 9255447231

Kashi Ram Kishan lal, 9289504004, 8920567245 Natraj Book Distributors, 7988917452

AJMER KOTA

BHUNA

Khurana Book Store, 9896572520

BHILWARA

JAMMU

JAIPUR

Sahitya Sangam, 9419190177

UDAIPUR

Nakoda Book Depot, (01482) 243653, 9214983594, Alankar Book Depot, 9414707462 Ravi Enterprises, 9829060694, Saraswati Book House, (0141) 2610823, 9829811155, Goyal Book Distt., 9460983939, 9414782130 Sunil Book Store, 9828682260

Crown Book Distributor & Publishers, (0651) 2213735, 9431173904, Pustak Mandir, 9431115138, Vidyarthi Pustak Bhandar, 9431310228

AGARTALA

Book Corner, 8794894165, 8984657146, Book Emporium, 9089230412

KARNATAKA

COIMBATORE

SURAT

BALLABGARH HISAR

BOKARO RANCHI DUMKA

PUNJAB

Shopping Point, 9824108663

JALANDHAR

Babu Ram Pradeep Kumar, 9813214692

JHARKHAND

Bokaro Student Friends, (0654) 2233094, 7360021503, Bharati Bhawan Agencies, 9431740797

JODHPUR

Renuka Book Distributor, (0836) 2244124, Vidyamandir Book Distributors, 9980773976 CHENNAI

BANGLORE

Krishna book house, 9739847334, Hema Book Stores, 9986767000, Sapna Book House Pvt. Ltd., 9980513242, Hema Book World, (Chamrajpet) (ISC) 080-40905110, 9945731121

BELLERI

Chatinya book centre, 9886064731

PUDUCHERRY

ERNAKULAM

Academic Book House, (0484) 2376613, H & C Store, 9864196344, Surya Book House, 9847124217, 9847238314 Book Centre, (0481) 2566992 Academic Book House, (0471) 2333349, 9447063349, Ponni Book Stall, 9037591721

TRICHY

KOTTAYAM TRIVANDRUM CALICUT

Aman Book Stall, (0495) 2721282,

MADHYA PRADESH

CHHINDWARA

Pustak Bhawan, ( E & C ), 8982150100

GWALIOR

Agarwal Book Depot, 9425116210

Cheap Book Store, 9872223458, 9878258592, City Book Shop, 9417440753, Subhash Book Depot, 9876453625, Paramvir Enterprises, 9878626248 Sita Ram book Depot, 9463039199, 7696141911 Amit Book, 9815807871, Gupta Brothers, 9888200206, Bhatia Book Centre, 9815277131 Mohindra Book Depot, 9814920226

RAJASTHAN

Laxmi General Store, Ajmer, 0145- 2428942 9460652197 Vardhman Book Depot, 9571365020, 8003221190 Raj Traders, 9309232829

Second Hand Book Stall, 9460004745

TRIPURA

TAMIL NADU

HUBLI

KERALA

Bharat Book Depot, 7988455354 Goel Sons, 9463619978, Adarsh Enterprises, 9814347613

SALEM

THENI MADURAI VELLORE

HYDERABAD

Majestic Book House, (0422) 2384333, CBSC Book Shop, 9585979752

Arraba Book Traders, (044) 25387868, 9841459105, M.R. Book Store (044) 25364596, Kalaimagal Store, (044) 5544072, 9940619404, Vijaya Stores, 9381037417, Bookmark It-Books & Stat. Store, 7305151653, M.K. Store, 9840030099, Tiger Books Pvt. Ltd., 9710447000, New Mylai Stationers, 9841313062, Prince Book House, Chennai, 0444-2053926, 9952068491, S K Publishers & Distributors, 9789865544, Dharma Book Shop, 8667227171 Sri Lakshmi Book Seller, 7871555145 Pattu book centre, 9894816280

P.R.Sons Book Seller, 9443370597, Rasi Publication, 9894816280 Maya Book Centre, 9443929274 Selvi Book Shoppe, 9843057435, Jayam Book Centre, 9894658036 G.K book centre and collections, 9894517994

TELANGANA

Sri Balaji Book Depot, (040) 27613300, 9866355473, Shah Book House, 9849564564 Vishal Book Distributors, 9246333166, Himalaya Book World, 7032578527

( viii )

0808

AHMEDABAD

Contd... UTTARAKHAND

GORAKHPUR

Central Book House, 9935454590, Friends & Co., 9450277154, Dinesh book depot, 9125818274, Friends & Co., 9450277154

DEHRADUN

Inder Book Agencies, 9634045280, Amar Book Depot , 8130491477, Goyal Book Store, 9897318047, New National Book House, 9897830283/9720590054

JHANSI

Bhanu Book Depot, 9415031340

MUSSORIE

Ram Saran Dass Chanda kiran, 0135-2632785, 9761344588

KANPUR

Radha News Agency, 8957247427, Raj Book Dist., 9235616506, H K Book Distributors, 9935146730, H K Book Distributors, 9506033137/9935146730

UTTAR PRADESH

LUCKNOW

Sparsh Book Agency, 9412257817, Om Pustak Mandir, (0562) 2464014, 9319117771,

MEERUT

AGRA

Sanjay Publication, 8126699922 Arti book centre, 8630128856, Panchsheel Books, 9412257962, Bhagwati Book Store, (E & C), 9149081912

Vyapar Sadan, 7607102462, Om Book Depot, 7705871398, Azad Book Depot Pvt. Ltd.,

7317000250, Book Sadan, 9839487327, Rama Book Depot(Retail), 7355078254, Ashirwad Book Depot, 9235501197, Book.com, 7458922755, Universal Books,

9450302161, Sheetla Book Agency, 9235832418, Vidyarthi Kendra Publisher & Distributor Pvt Ltd, (Gold), 9554967415, Tripathi Book House, 9415425943

Ideal Book Depot, (0121) 4059252, 9837066307

ALLAHABAD

Mehrotra Book Agency, (0532) 2266865, 9415636890

NOIDA

Prozo (Global Edu4 Share Pvt. Ltd), 9318395520, Goyal Books Overseas Pvt.Ltd., 1204655555 9873387003

AZAMGARH

Sasta Sahitya Bhandar, 9450029674

PRAYAGRAJ

Kanhaiya Pustak Bhawan, 9415317109

ALIGARH

K.B.C.L. Agarwal, 9897124960, Shaligram Agencies, 9412317800, New Vimal Books, 9997398868, T.I.C Book centre, 9808039570

MAWANA

Subhash Book Depot, 9760262264

BULANDSHAHAR

Rastogi Book Depot, 9837053462/9368978202

BALRAMPUR

Universal Book Center, 8933826726

KOLKATA

BAREILLY

Siksha Prakashan, 9837829284

RENUKOOT

HARDOI

Mittal Pustak Kendra, 9838201466

WEST BENGAL Oriental Publishers & Distributor (033) 40628367, Katha 'O' Kahini, (033) 22196313, 22419071, Saha Book House, (033), 22193671, 9333416484, United Book House, 9831344622, Bijay Pustak Bhandar, 8961260603, Shawan Books Distributors, 8336820363, Krishna Book House, 9123083874

Om Stationers, 7007326732

DEORIA

Kanodia Book Depot, 9415277835

COOCH BEHAR

S.B. Book Distributor, Cooch behar, 9002670771

VARANASI

Gupta Books, 8707225564, Bookman & Company, 9935194495/7668899901

KHARAGPUR

Subhani Book Store, 9046891334

MATHURA

Sapra Traders, 9410076716, Vijay Book House , 9897254292

SILIGURI

Agarwal Book House, 9832038727, Modern Book Agency, 8145578772

FARRUKHABAD

Anurag Book Agencies, 8844007575

DINAJPUR

Krishna Book House, 7031748945

NAJIBABAD

Gupta News Agency, 8868932500, Gupta News Agency, ( E & C ), 8868932500

MURSHIDABAD

New Book House, 8944876176

DHAMPUR

Ramkumar Mahaveer Prasad, 9411942550

Entrance & Competition Distributors PATNA

BIHAR

CUTTAK

A.K.Mishra Agencies, 9437025991

Metro Books Corner, 9431647013, Alka Book Agency, 9835655005, Vikas Book Depot, 9504780402

BHUBANESHWAR

M/s Pragnya, 9437943777

CHATTISGARH KORBA

Kitab Ghar, 9425226528, Shri Ramdev Traders, 9981761797

PUNJAB JALANDHAR

DELHI DELHI

Singhania Book & Stationer, 9212028238, Radhey Book depot, 9818314141, The KOTA Book Shop, 9310262701, Mittal Books, 9899037390, Lov Dev & Sons, 9999353491

NEW DELHI

Anupam Sales, 9560504617, A ONE BOOKS, 8800497047

JAIPUR

HARYANA AMBALA

BOKARO

Cheap Book Store, 9872223458, 9878258592

RAJASTHAN Vardhman Book Depot, 9571365020, Raj Traders, 9309232829 Goyal Book Distributors, 9414782130

UTTAR PRADESH

Bharat Book Depot, 7988455354

AGRA

BHAGWATI BOOK STORE, 9149081912, Sparsh Book Agency, 9412257817, Sanjay Publication, 8126699922

JHARKHAND

ALIGARH

New Vimal Books, 9997398868

Bokaro Student Friends Pvt. Ltd, 7360021503

ALLAHABAD

Mehrotra Book Agency, (532) 2266865, 9415636890

MADHYA PRADESH

GORAKHPUR

Central Book House, 9935454590

INDORE

Bhaiya Industries, 9109120101

KANPUR

Raj Book Dist, 9235616506

CHHINDWARA

Pustak Bhawan, 9827255997

LUCKNOW

Azad Book Depot PVT LTD, 7317000250, Rama Book Depot(Retail), 7355078254 Ashirwad Book Depot , 9235501197, Book Sadan, 8318643277, Book.com , 7458922755, Sheetla Book Agency, 9235832418

MAHARASHTRA

PRAYAGRAJ

Format Center, 9335115561, Garg Brothers Trading & Services Pvt. Ltd., 7388100499

Laxmi Pustakalay and Stationers, (0712) 2727354

PUNE

Pragati Book Centre, 9850039311

MUMBAI

New Student Agencies LLP, 7045065799

ODISHA BARIPADA

UTTAR PRADESH DEHRADUN

Inder Book Agancies, 9634045280

WEST BENGAL KOLKATA

Trimurti Book World, 9437034735

(1)

Bijay Pustak Bhandar Pvt. Ltd., 8961260603, Saha Book House, 9674827254 United Book House, 9831344622, Techno World, 9830168159

0808

NAGPUR

CUET (UG) Exam Paper 2023 National Testing Agency Held on 25th May 2023

BIOLOGY Solved

(This includes Questions pertaining to Domain Specific Subject only) Max. Marks: 200

Time allowed: 45 Minutes

General Instructions: 1. The test is of 45 Minutes duration. 2. The test contains 50 questions out of which 40 questions needs to be attempted. 3. Marking Scheme of the test: a. Correct answer or the most appropriate answer: Five marks (+5). b. Any incorrect option marked will be given minus one mark (-1). c. Unanswered/Marked for Review will be given 100 mark (0).

1. Select

the organism which does not undergoes parthenogenesis. (1) Honeybee (2) Rotifer (3) Turkey (4) Fruit fly Ans. Option (4) is correct. Explanation: Parthenogenesis is development of an egg into a complete individual without fertilisation. It occurs commonly among lower plants and invertebrate animals (particularly rotifers, aphids, ants, wasps, and bees) and rarely among higher higher vertebrates.

2. The

method of producing thousands of plants through tissue culture is called as: (1) Somaclones (2) Micro-propagation (3) Somatic hybridisation (4) Vegetative propagation Ans. Option (2) is correct Explanation : Micro propagation, also known as plant tissue culture, is the technique used for rapid vegetative multiplication of plants by using explants (living tissue or cells of a plant). Through this process, a large number of plants can be produced in a short span of time.

3. Which

of the following statements are correct regarding decomposition? A. Decomposition is largely oxygen-requiring process. B. The rate of decomposition is controlled by chemical composition of detritus and climatic factors.

C. Dry and cold environment favour the decomposition. D. If detritus is rich in lignin and chitin, the decomposition rate is faster.

Choose the correct answer from the options given below:

(1) A and B only (2) C and D only (3) B and C only (4) A and D only Ans. Option (1) is correct Explanation : Decomposition is the process in which complex organic matter breaks down into inorganic substances like carbon dioxide, water and nutrients. It is largely an oxygen requiring process. The rate of decomposition is controlled by the chemical composition of detritus and climatic factors. It is slower if detritus is rich in lignin and chitin and quicker if detritus is rich in nitrogen and water soluble substances like sugars.

4. Identify the organism which is a source of single cell protein and is grown on commercial scale. (1) Azotobacter (2) Rhizobium (3) Azospirillum (4) Spirulina Ans. Option (4) is correct

Explanation : Microbes like Spirulina can be grown easily on material like waste material. They are being grown on an industrial scale as a source of good protein. It is used as a single celled protein.

Solved Paper - 2023 5. Match List I with List II

8. Identify the equation representing logistic growth

LIST I A

Wheat

of a population.

LIST II I

Perisperm

B

Black pepper

II

Polyembryony

C

Cashew

II.

Albuminous seed

D

Citrus

IV False fruits

Choose the correct answer from the options given below: (1) A-III, B-II, C-I, D-IV (2) A-II, B-I, C-III, D-IV (3) A-III, B-I, C-IV, D-II (4) A-I, B-II, C-IV, D-III

Explanation : Wheat is an albuminous seed, which have endosperm with persistent stores of nutrients. Perispermic seeds are seen in black pepper and beetroot. False fruits are seen in apple, strawberry and cashew nut whereas polyembryony is seen in Citrus and Mango.

Choose the correct answer from the options given below: (1) A, D, B, E (2) A, B, D, C (3) A, B, C, D (4) C, D, B, A

Ans. Option (1) is correct Explanation : During fertilisation, the sperm and egg unite in one of the fallopian tubes to form a zygote. Then the zygote travels down the fallopian tube, where it becomes a morula. Once it reaches the uterus, the morula becomes a blastocyst. The blastocyst then burrows into the uterine lining by a process called implantation. Hence, the correct sequence of events is: Formation of zygote → Formation of Morula → Formation of blastocyst → Implantation.

dN N −K = N  dt  K 

(4)

dt = rN dN

Greek thinkers thought that units of life called spores were transferred to different planets including earth. Identify the term associated with the above. (1) Abiogenesis (2) Panspermia (3) Spontaneous generation (4) Biogenesis

Ans. Option (2) is correct Explanation : According to early Greek thinkers, units of life were called panspermia. They assumed that these panspermias were transferred to different planets including earth, which resulted in life.

10. In

gel electrophoresis the separated DNA fragments can be visualised : (1) In visible light (2) In visible light with staining (3) In UV radiation without staining (4) In UV radiation after staining with ethidium bromide

Ans. Option (4) is correct Explanation : Agarose gel electrophoresis is used to segregate DNA fragments according to the mass and size. The separated DNA fragments can be visualised only after staining the DNA with a compound known as ethidium bromide followed by exposure to UV radiation. We cannot see pure DNA fragments in the visible light and without staining.

cell as 'Competent cell’ to take up DNA? (1) Carbonate ion (2) Calcium ion (3) Sodium ion (4) Sulphate ion

Explanation : In order to make the bacterial cells competent, they are first treated with a specific concentration of a divalent cation, such as calcium. It increases the efficiency with which DNA enters the bacterium through pores in its cell wall.

(3)

9. Early

7. Which of the following is used to make the bacterial

Ans. Option (2) is correct

dN dN K−N = rN (2) = rN   dt dt  K 

Explanation : The logistic growth curve, which is J-shaped, shows population growth when resources are limited. Carrying capacity is the maximum size of the population of a species that a certain environment can support for an extended period of time. The logistic growth equation can be given as dN/dt= rN (K-N/K). Here, r = the intrinsic rate of growth, N = the number of organisms in a population, and K = the carrying capacity.

6. Arrange the following events in correct sequence. Formation of zygote Formation of blastocyst Implantation Formation of morula

(1)

Ans. Option (1) is correct

Ans. Option (3) is correct

A. B. C. D.

3

11. Match List I with List II LIST 1

LIST II

A.

Tubectomy

I.

Barrier method

B.

Copper ions

II.

Surgical method of sterilisation in human male.

4

OSWAAL CUET (UG) Sample Question Papers, BIOLOGY

C.

Cervical cap

III.

Surgical method of sterilisation in human female.

D.

Vasectomy

IV.

Suppress motility of sperms

Choose the correct answer from the options given below: (1) A-III, B-IV, C-I, D-II (2) A-III, B-I, C-IV, D-II (3) A-II, B-I, C-IV, D-III (4) A-IV, B-II, C-III, D-I

recessive disease resulting from mutations in the α- and β-globin gene clusters on chromosome 16 and chromosome 11, respectively. Klinefelter’s syndrome (sometimes called XXY) is where boys and men are born with an extra X chromosome. Turner’s syndrome, a condition that affects only females, results when one of the X chromosomes (sex chromosomes) is missing or partially missing.

14. Match List I with List II LIST I (Drugs)

Ans. Option (1) is correct Explanation : Tubectomy is the sterilisation procedure in females. In this, a small part of the fallopian tube is removed or tied up through a small incision in the abdomen or through vagina. Copper releasing IUDs, namely Cu-T, Cu-7 release copper that suppress the sperm motility which in turn reduces its fertilising capacity. Cervical cap is a barrier method of contraception which prevents conception by blocking the entry of sperms through the cervix. Vasectomy is the sterilisation procedure in males. In this, a small part of the vas deferens is removed or tied up through a small incision on the scrotum.

12. Mammals are able to survive in Antarctica or in the Sahara desert as they are (1) Conformers (2) Partial regulators (3) Regulators (4) Migrants

Ans. Option (3) is correct Explanation : Mammals are able to survive in Antarctica or in Sahara desert as they are regulators. Evolutionary biologists believe that the success of mammals is largely due to their ability to maintain a constant body temperature and thrive whether they live in Antarctica or in the Sahara desert.

13. Match List I with List II LIST I

LIST II

A.

Down's syndrome

I.

44 + XXY

B.

Thalassemia

II.

Autosomal recessive trait

C.

Klinefelter's syndrome

III. 44 + XO

D.

Turner's Syndrome

IV.

45 +XY/ XX

Choose the correct answer from the options given below: (1) A-IV, B-II, C-III, D-I (2) A-IV, B-II, C-I, D-III (3) A-I, B-III, C-IV, D-II (4) A-II, B-IV, C-III, D-I

Ans. Option (2) is correct Explanation : Down’s syndrome is a genetic disorder which arises due to the presence of an extra chromosome number 21 in every cell of the body. So, 47 chromosomes are present instead of the normal number 46. Thalassemia is an inherited autosomal

LIST II (Obtained from)

A. Heroin

I.

Tobacco plant

B.

II.

Erythroxvium coca

Cannabinoids

C. Cocaine

III. Cannabis sativa

D. Nicotine

IV.

Papaver somniferum

Choose the correct answer from the options given below: (1) A-I, B-II, C-III, D-IV (2) A-II, B-I, C-III, D-IV (3) A-I, B-II, C-IV, D-III (4) A-IV, B-III, C-II, D-I

Ans. Option (4) is correct Explanation : Heroin is an opium derivative. Opium is obtained from dried latex of unripe capsules of Poppy plant (Papavar somniferum). Natural cannabinoids are obtained from Cannabis sativa (Hemp plant).Cocaine is obtained from coca plant, Erythroxylum coca.

15. Methanogens do not produce : (1) (2) (3) (4)

Carbon dioxide and methane Methane and Hydrogen Hydrogen and Carbon dioxide Nitrogen and Oxygen

Ans. Option (4) is correct Explanation : Methanogens are the microorganisms that produce methane, hydrogen sulphide and carbon dioxide in their metabolic reactions.

16. The

correct sequence of steps involved in polymerase chain reaction (PCR) are: A. DNA polymerase is used to extends the primers using oligonucleotides. B. Desired DNA fragments are denatured. C. Amplified fragments are ligated with the vector for cloning. D. Oligonucleotides are added

Choose the correct answer from the options given below: (1) C, A, B, D (3) A, D, B, C

Ans. Option (4) is correct

(2) A, C, B, D (4) B, D, A, C

Solved Paper - 2023 Explanation : Each cycle has three steps: Denaturation, primer annealing and extension of primers. Denaturation: The double stranded DNA molecules are heated to a high temperature (94˚C) so that the two strands separate into a single stranded DNA molecule. This process is called denaturation. Each strand acts as template for DNA synthesis. Annealing: In this step, the two oligonucleotide primers anneal (hybridize) to each of the single stranded DNA template, since, the sequence of the primers is complementary to the 3’ends of the template DNA. This step is carried out at a lower temperature depending on the length and sequence of the primers. This results in the duplication of the original DNA molecule. Extension of primers: DNA polymerase (Taq polymerase) extends the primers using the nucleotides provided in the reaction. The optimum temperature for this polymerization step is 72˚C. This process is repeated over several cycles to obtain multiple copies of rDNA fragment.

17. Select the correct statement/s from the following: A. Spermatogonia always undergo meiotic cell division. B. Primary spermatocytes divide by mitotic cell division. C. Secondary spermatocytes have 23 chromosomes and undergo second meiotic division. D. Spermatozoa are transformed into spermatids.

Choose the correct answer from the options given below: (1) A and C only (3) D only

(2) C only (4) B and C only

Ans. Option (2) is correct Explanation : Statement C is correct. Spermatogonia are diploid cells on the inside wall of seminiferous tubules that multiply by mitotic divisions. Some of the spermatogonia called primary spermatocyte undergo meiosis-I to give rise to secondary spermatocytes, which are haploid. Each secondary spermatocyte undergoes meiosis-II to give rise to two haploid spermatids which are transferred to spermatozoa by spermiogenesi.

18. Which of the following metal can not be recovered from manual recycling process of e-wastes ? (1) Gold (2) Mercury (3) Copper (4) Nickel

Ans. Option (2) is correct Explanation : E wastes are electronic wastes that generally include electronic goods such as computers etc. Such wastes are rich in metals such copper, iron, silicon, gold etc. These metals are highly toxic and pose serious health hazards. Recycling is the only solution for the treatment of e-waste, provided it is carried out in an environment friendly manner.

5

However toxicant such as asbestos, lead, mercury etc. cannot be recovered from manual recycling process of e-wastes.

19. Bt

toxin produced by Bacillus thuringiensis is

resistant to : (1) Drought (3) Nematodes

(2) Insects (4) Viruses

Ans. Option (2) is correct Explanation : Some strains of Bacillus thuringiensis have proteins that kill insects like coleopterans (beetles), lepidopterans (tobacco budworm, armyworm) and dipterans (flies, mosquitoes etc). Bacillus thuringiensis forms protein crystals during a particular phase of their growth. These crystals contain a toxic insecticidal protein. These proteins are present in inactive protoxin form, but become active toxin in the alkaline pH of insect gut. The activated toxin binds to the surface of midgut epithelial cells and creates pores that cause cell swelling and lysis and eventually cause death of the insect.

20. In some organisms, male has ZZ chromosomes and

female has ZW chromosomes. This type of sexdetermination is found in : (1) Drosophila (2) Hen (3) Cockroach (4) Snail

Ans. Option (2) is correct Explanation : Sex determination in birds and some reptiles is of ZW-ZZ types in which females are heteromorphic (ZW) and males have homomorphic sex chromosomes (ZZ).

21. The organisation sct up by the Indian Government to take decisions regarding the validity of GM research and the safety of introducing” GMorganism is: (1) Genetic Engineering Approval Committee (GEAC) (2) Genetic Modification Approval Committee (GMAC) (3) Indian Council of Agricultural Research (ICAR) (4) All India Institute of Medical Sciences (AIIMS) Ans. Option (1) is correct Explanation : Indian Government has set up organisations like GEAC (Genetic Engineering Approval Committee), which make decisions about the validity of GM research and the safety of GMorganisms for public services.

22. Which

of the following is a hormone releasing IUD? (1) Multiload 375 (2) LNG-20 (3) Cervical Cap (4) Vault

Ans. Option (2) is correct

6

OSWAAL CUET (UG) Sample Question Papers, BIOLOGY Explanation : LNG 20 is a hormone releasing IUDs, which makes the uterus unsuitable for implantation and the cervix hostile to the sperms.

23. The puffed up appearance of dough is due to the production of: (1) Oxygen gas (3) Ethyl alcohol

(2) CO2 gas (4) Pyruvic acid

Ans. Option (2) is correct Explanation : The dough, which is used for making bread, is fermented by using baker’s yeast (Saccharomyces cerevisiae). The puffed up appearance of dough is due to the production of CO2 gas.

24. Select the statements which do not hold true for

Cancer : A. Cancerous cells show a property of contact inhibition. B. Metastasis is the most feared property of malignant tumors. C. Malignant tumor cells invade and damage the surrounding tissue. D. Malignant tumor cells grows slowly.

Choose the correct answer from the options given below: (1) A and B only (3) A and D only

(2) B and C only (4) B and D only

Ans. Option (3) is correct Explanation : Cancer is an abnormal and uncontrolled multiplication of cells resulting in the formation of tumor. Cancerous cells appear to have lost the property of contact inhibition. They just continue to divide giving rise to masses of cell called tumors. Tumors are of two types: Benign tumors: They remain confined to the place of its origin. Hence, they cause little damage while malignant tumors are the mass of proliferating cells called neoplastic or tumor cells. These cells grow very rapidly and invade and damage surrounding tissues.

25. (p+q)2

= p2 + 2pq+q2 =1 represents Hardy Weinberg equation. It is used in: (1) Population genetics (2) Mendelian genetics (3) Biometrics (4) Molecular genetics

Ans. Option (1) is correct Explanation : Hardy-Weinberg law is an algebraic equation that describes the genetic equilibrium within a population. It states that in a large, random-mating population, the gene pool (total genes and their alleles in a population) tends to remain constant from generation to generation unless outside forces act to change it. In such a way even the rarest forms of genes, which one would assume would disappear, are preserved.

26. Mature mRNA is a fully processed (1) hnRNA (3) 28S RNA

(2) snRNA (4) 5srRNA

Ans. Option (1) is correct Explanation : Mature mRNA is the completely processed mRNA molecule in the cell of eukaryotes. The mRNA is a type of RNA involved in protein synthesis. It is synthesized based on a DNA template on the nucleus.

27. Which forest also known as the 'lungs of the planet earth’ ? (1) Amazon rain forest (2) Rain forest of north-east India (3) Tiaga forest (4) Tundra forest

Ans. Option (1) is correct Explanation : The Tropical rainforest of Amazon is known as the lungs of the planet earth' because they are responsible for the production of about 20% of the earth's oxygen. They remove a large amount of carbon dioxide from the air and give out oxygen.

28. Inbreeding

is carried out in animal husbandry because it: (1) Increase hybrid vigour (2) Improves the breed (3) Increase heterozygosity (4) Increase homozygosity

Ans. Option (4) is correct Explanation : Inbreeding is the mating of more closely related individuals within the same breed for 4-6 generations. It is carried out in animal husbandry because it increases homozygosity to evolve a pure line animal.

29. Which is known as 'Terror of Bengal'.? (1) Water Lilly (3) Water hyacinth

(2) Hydrilla (4) Lantana

Ans. Option (3) is correct Explanation : Water hyacinth is known as the terror of Bengal. It is an exotic shrub which is growing at an alarming rate on the surface of water body. It inhibits the growth of fish and other aquatic organisms due to cut down of light and lack of oxygen. It is also an example of loss of biodiversity due to alien species invasion. It is also competing with the other native species and has also eliminated many.

30. Threatened animals and plants are taken out from their natural habitat and placed in special settings, protected and given special care is. (1) In situ conservation (2) Ex situ conservation (3) Conservation in national park (4) Conservation in biospheres

Solved Paper - 2023 Ans. Option (2) is correct

Ans. Option (3) is correct

Explanation : Ex situ conservation (off site) is the method of conservation in which threatened species of plants and animals are taken out of their habitats and are kept in special settings such as genetic resource centres, zoological parks, botanical gardens, gene banks.

31. Match List I with List II

Explanation : The megaspore mother cell (MMC) undergoes meiotic division to produce four megaspores. In a majority of flowering plants, only one megaspore is functional while the other three degenerate. The single functional megaspore develops into the female gametophyte.

33. In

LIST I

LIST II

A.

Dominant trait of pod colour in garden pea.

I.

Polygenic traits

B.

The physical association of two genes on a chromosome.

II.

Pleiotropy

C

The traits generally III. Yellow controlled by three or more genes.

D

When a single gene exhibit multiple phenotypic expression.

IV.

Linkage

V.

Green

Choose the correct answer from the options given below (1) A-III, B-II, C-I, D-V (2) A-II, B-III, C-V, D-I (3) A-III, B-II, C-I, D-IV (4) A-V, B-IV, C-I, D-II

which process unusual nucleotide (methyl guanosine triphosphate) is added to the 5' end of hnRNA. (1) Splicing (2) Capping (3) Tailing (4) Transcription factor

Ans. Option (2) is correct Explanation : Capping is the process in which an unusual nucleotide called methyl guanosinetriphosphate (cap) is added to the 5’ end of hnRNA. Tailing (Polyadenyaltion) is the process in which adenylate residues (200-300) are added at 3’ end of hnRNA in a template independent manner. Splicing is a biological process where a newly synthesized pre-mRNA is transformed into a mature mRNA. It involves the removal of noncoding sequences known as introns and then, joining the coding regions known as exons. Transcription factors are proteins involved in the process of converting, or transcribing, DNA into RNA.

34. Select the incorrect statements given below.

Ans. Option (4) is correct Explanation : Green pod colour was dominant to yellow in garden pea. Linkage is defined as the co-existence of two or more genes in the same chromosome. If the genes are situated on the same chromosome and lie close to each other, then they are inherited together and are said to be linked genes. Pleiotropy is the ability of a gene to have multiple phenotypic effects as it influences a number of characters simultaneously. Polygenic inheritance occurs when one characteristic is controlled by two or more genes. Inheritance of skin pigmentation is an example of polygenic inheritance.

32. Sequence

the following steps in formation of female gametophyte of flowering plants A. Cell walls are laid down. B. Formation of seven cells with eight nuclei. C. Meiosis in megaspore mother cell and formation of megaspore tetrad. D. Functional megaspore undergoes three mitotic divisions, results in formation of eight nuclei.

Choose the correct answer from the options given below: (1) A, D, C, B (3) C, D, A. B

7

(2) A, C, B, D (4) C, A, B, D

A. Methane and Carbon dioxide are green house gases. B. The Montreal Protocol is associated with the control of emission of ozone depleting substances. C. Use of incinerators is not crucial to dispose off hospital wastes. D. Dobson units is used to measure water quality.

Choose the correct answer from the options given below: (1) A and B (3) A and C

(2) C and D (4) B and D

Ans. Option (2) is correct Explanation : The thickness of the ozone (O3) in a column of air from the ground to the top of the atmosphere is measured in terms of Dobson units (DU).Hospital wastes contain disinfectants and other harmful chemicals, and also pathogenic micro-organisms. The incinerators are used to dispose hospital wastes.

35. Biofertilisers

are the organisms that enrich the nutrient quality of the soil. Which of the following is not a source of biofertiliser? (1) Bacteria (2) Fungi (3) Baculoviruses (4) Cyanobacteria

Ans. Option (3) is correct

8

OSWAAL CUET (UG) Sample Question Papers, BIOLOGY Explanation : Biofertilisers are living organisms, which help increase the fertility of soil. It involves the selection of beneficial microorganisms like Bacteria, fungi, cyanobacteria etc. Baculoviruses are suitable for species-specific, narrow spectrum insecticidal applications. This is desirable in IPM (Integrated pest management) program to conserve beneficial insects.

(2) Primary succession (3) Secondary succession (4) Mesarch succession Ans. Option (3) is correct Explanation : Succession occurring after flood is called secondary succession. Successions where natural communities develop in an area that was earlier occupied by living organisms that was somehow lost are called secondary succession.

36. Match List I with List II LIST I

LIST II

A. The primates with brain capacity of around 900 cc.

I

Neanderthal man

B. The primates with brain capacity of 1400 cc and lived in east and central Asia

II

Homo erectus

C. The primates which arose in Africa and moved across continents and developed into distinct races

III Homo sapiens

D. The primates with the brain capacities between 650-800 cc and probably did not eat meat.

IV Australopithecines

39. Identify the statement which do not hold true for Deoxyribose Nucleic Acid. A. Only purine bases are present in DNA. B. Deoxyribose sugar is present C. A nitrogenous base is linked to the 1'C pentose sugar through N-glycosidic linkage. D. Phosphate group is linked to OH of 4'C of a nucleotide.

Choose the correct answer from the options given

V

below: (1) A and D only (3) C and D only

Ans. Option (1) is correct Explanation : DNA has two purine bases: Adenine and guanine, which form hydrogen bonds with their complementary pyrimidines derivatives i.e. thymine, and cytosine, respectively. The phosphate is linked by a phosphodiester linkage to the 5’ carbon of a nucleotide.

Homo habilis

Choose the correct answer from the options given below: (1) A-III, B-I, C-V, D-II (2) A-II, B-I, C-III, D-V (3) A-I, B-IV, C-III, D-V (4) A-I, B-III, C-IV, D-II

Ans. Option (2) is correct Explanation : Homo erectus is the primates with brain capacity of around 900 cc. Neaderthal man is the primates with brain capacity of 400 cc, which lived in east and central Asia. Homo sapiens are the primates which arose in Africa and moved across continents and developed into distinct races.Homo habilis are the primates with the brain capacities between 650- 800 cc, which probably did not eat meat.

37. Identify

the gene which is effective against corn bore (1) cryLAc (2) crylAb (3) cryILAb (4) z gene

Ans. Option (2) is correct Explanation : The protein coded by gene cryIAb controls corn borer whereas the protein coded by genes cryIAc and cryIIAb control the cotton bollworms.

38. Succession occurring after flood is: (1) Hydrarch succession

(2) A and B only (4) B and C only

40. Match List I with List II LIST I

LIST II

A.

Salmonella phi

I.

Common Cold

B.

Rhmo viruse

II

Typhoid

C.

Sreptecoccus preumoniae

III Malaria

D.

Plasmodium vivax

IV Pneumonia

Choose the correct answer from the options given below: (1) A-II, B-IV, C-I, D-III (2) A-II, B-IV, C-III, D-I (3) A-II, B-I, C-IV, D-III (4) A-I, B-II, C-III, D-IV

Ans. Option (3) is correct Explanation : Typhoid fever is a life-threatening infection caused by the bacterium Salmonella typhi. Rhinoviruses are the most common cause of the common cold.The most common bacteria causing pneumonia is Streptococcus pneumoniae.

Read the passage and answer the question given below.

Malaria is caused by the Plasmodium parasite. The parasite is spread to humans through the bites of infected mosquitoes. When a host is exposed to

Solved Paper - 2023 antigens, which may be in the form of living or dead microbes or other proteins,antibodies are produced in the host body. This type of immunity is called active immunity. Active immunity is slow and takes time to give its full effective response. Injecting the microbes deliberately during immunisation or infectious organisms gaining access into body during natural infection induce active immunity, When antibodies are directly given to protect the body against foreign agents, it is called passive immunity.

41. Colostrum secreted by mother during initial stage

of lactation provides passive immunity as it is rich in. (1) Placental lactogen (2) Prolactin (3) Antigen (4) IgA antibodies

Ans. Option (4) is correct Explanation : The yellowish milk produced by the new mother during the initial few days of lactation is called colostrum. It which contains immunoglobin A which is essential to the new born baby to develop resistance to various antigens.

42. Which of the following is passive immunisation? (1) (2) (3) (4)

injecting inactivated pathogens injecting antigenic preparation injecting preformed antibodies injected weakened pathogens

9

45. Match List I with List II LIST I

LIST II

A.

Physical Barrier

I.

Saliva

B.

Cellular Barrier

II.

Interferons

C.

Physiological Barner III. Skin

D.

Cytokine Barrier

IV.

Polymorpho-nuclear leukocytes

Choose the correct answer from the options given below: (1) A-III, B-I, C-II, D-IV (2) A-III, B-IV, C- l, D-II (3) A-I, B-IV, C-III, D-II (4) A-IV, B-I, C-III, D-II

Ans. Option (2) is correct Explanation : Skin is an example of physical barrier. Polymorpho-nuclear leukocytes (PMNLneutrophils) kill pathogen by phagocytosis. They act as cellular barrier. Saliva in mouth is an example of physiological barrier while virus infected cells secrete proteins called interferon which protect non-infected cells from spread of viral infection is an example of cytokinin barrier.

Observe

the diagram and answer the question given below.

Ans. Option (3) is correct Explanation : Passive immunity is a type of acquired immunity in which ready made antibodies are transferred from one individual to another. It is fast and provides immediate relief.

43.

Immunity provided to the foetus from the mother through placenta during pregnancy is : (1) Active immunity (2) Passive immunity (3) Non-specific immunity (4) Innate immunity

Ans. Option (2) is correct Explanation : Immunity provided to the foetus from the mother through placenta during pregnancy is called passive immunity.

44. Production

of antibodies against antigens in the

body is : (1) Innate immunity (2) Passive immunity (3) Active immunity (4) Non-Specific immunity

Ans. Option (3) is correct Explanation : Production of antibodies against antigens in the body is called active immunity. It is a slow process but has long lasting effect.

46. Which of the following constitutes the first trophic level? (1) Plants (3) Carnivores

(2) Herbivores (4) Omnivores

Ans. Option (1) is correct Explanation : Producers always occupy the first trophic level of every food chain because only they have the ability to trap solar energy with the help of chlorophyll and synthesize their own food.

47. Sparrow

comes under which of the following categories : (1) Producers (2) Primary consumers (3) Secondary consumers (4) Tertiary consumers

Ans. Option (3) is correct

10

OSWAAL CUET (UG) Sample Question Papers, BIOLOGY Explanation : Sparrow is a primary consumer when it eats seeds, fruits, peas and a secondary consumer when it eats insects and worms.

48. Each

trophic level has a certain mass of living

material at a particular time called as : (1) Standing crop (2) Living crop (3) Standing life (4) Time crop Ans. Option (1) is correct Explanation : Each trophic level has a certain mass of living material at a particular time called as the standing crop. The standing crop is measured as the mass of living organism (biomass) or the number in a unit area. The biomass of a species is expressed in terms of fresh or dry weight.

49. Which

among the following is not a primary consumer? (1) Snake (2) Grasshopper Cl (3) Zooplankton (4) Caterpillar Ans. Option (1) is correct Explanation : Snakes are not primary consumers as they are not herbivores.

50. Which of the following is a part of detritus food chain? (1) Earthworm (3) Man Ans. Option (1) is correct

(2) Birds (4) Snake

Explanation : The detritus food chain is a type of food chain that starts with dead organic matter. The earthworm acts as the primary consumer in a detritus food chain.



CUET Question Paper 2022 NATIONAL TESTING AGENCY 18th August 2022—Slot-1

Biology [This includes Questions pertaining to Domain Specific Subject only]

SOLVED . Time Allowed: 3 hrs.

Maximum Marks: 200

General Instructions: 1.

The test is of 45 Minutes duration.

2.

The test contains is 50 Questions out of which 40 questions needs to be attempted.

3.

Marking Scheme of the test: a. b. c.

Correct answer or the most appropriate answer: Five marks (+5). Any incorrect option marked will be given minus one mark (–1). Unanswered/marked for review will be given no mark (0).

1. The protein formed by the encoded gene expression in a heterologous host is called: (A) Structural protein (B) Recombinant protein (C) Transposons (D) Prohormone Ans. Option (B) is correct Explanation: Structural proteins are the most abundant class of proteins in nature and form structural elements. Transposons are repetitive DNA sequences that have the capacity to move from one location to another in genome and prohormones are the precursors of hormones. Therefore, Option (B) is the correct answer as any protein encoding gene is expressed in a heterologous host is called recombinant protein. 2. A kind of population interaction in which one species benefits and the other is neither harmed nor benefited? (A) Commensalism (B) Ammensalism (C) Mutualism (D) Parasitism Ans. Option (A) is correct Explanation: A kind of population interaction in which one species benefits and the other is neither harmed nor benefited is commensalism. It can also be denoted by (+, 0). 3. Fruit which develop only from the ovary are called— (A) False fruits (B) Parthenocarpic fruits (C) True fruits (D) Apomictic fruits Ans. Option (C) is correct Explanation: Fruit which develops only from the ovary are called true fruits, while in false fruits, parts other than ovary also contributes in its formation. 4. Algal blooms do not cause— (A) Imbalance in ecosystem dynamics.

(B) Deterioration of the water quality and fish mortality. (C) Reduction in BOD. (D) Increase in organic matters in water body. Ans. Option (C) is correct Explanation: Algal blooms are formed as a result of overproduction of algae in a water body that is nutrient-rich or eutrophicated water-bodies. They can cause harmful effects on aquatic species by increasing organic matter and thus BOD of water body increases too.

5.

Observe the given figure and name the step used in Recombinant DNA Technology. (A) Selecting (B) Scrolling (C) Spiraling (D) Spooling Ans. Option (D) is correct Explanation: Spooling which is a method of extraction of substance like DNA in the form of a spool over a glass rod. 6. Perisperm differs from endosperm is that it is— (A) Haploid having reserve food. (B) Polyploid having reserve food. (C) Triploid having no reserve food. (D) Diploid having no reserve food. Ans. Option (D) is correct Explanation: Perisperm is functionally similar to the endosperm. But perisperm is diploid as it is

12

7.

OSWAAL CUET (UG) Sample Question Papers, Biology developed from nucellus by mitosis, while endosperm is triploid. Identify and name the two parts in a flower which are most important units of sexual reproduction?

(A) 1 - Style, 3 - stamen (B) 4 - filament, 6 - thalamus (C) 3 - Anther, 7 - ovary (D) 2 – Stigma, 5 – sepals Ans. Option (C) is correct Explanation: The important units of sexual reproduction are anther and ovary. So correct option is (C). 8. Density of population tells us about— (A) total number of individuals of a species (B) total area occupied by a species (C) number of individuals present per unit space in a given time (D) population growth in a particular time span Ans. Option (C) is correct. Explanation: Population density is a measurement of the number of people in an area. It is an average number. It is generally measured as number of individuals of species present in unit space in a given time. 9. Select the hormones produced in women only during pregnancy . A. Estrogen B. Human chronic gonadotroph C. Progesterone D. Human placental lactogen E. Relaxin Choose the correct answer from the options given below: (A) B and D only (B) B and E only (C) A, B and C only (D) B, D and E only Ans. Option (D) is correct. Explanation: hCG (human chorionic gonadotropin), hpL (human placental lactogen) and relaxin are produced in humans only during pregnancy while Estrogen and progesterone are secreted in a non-pregnant woman also. 10.Which of the following is not an example of terrestrial ecosystem? (A) Wetland (B) Grassland (C) Forest (D) Desert Ans. Option (A) is correct. Explanation: Wetland is not a terrestrial ecosystem, it is an aquatic ecosystem. Rest all are examples of terrestrial ecosystems.

11. Transfer of an ovum collected from a donor into fallopian tube is called _____ method. (A) ZIFT (B) ICST (C) GIFT (D) IVF Ans. Option (C) is correct. Explanation: GIFT is the method in which transfer of ovum collected from a donor into the fallopian tube of another female who cannot produce one, but can provide suitable environment for fertilisation takes place. Whereas, ZIFT stands for Zygote intra fallopian transfer which involves transfer of zygote (upto 8 blastomeres) into the fallopian tube. ICSI stands for Intracytoplasmic sperm injection and is the method of injecting the sperms directly into the ovum in-vitro. IVF stands for in- vitro fertilisation. 12. Given below are two statements: one is labelled as Assertion A and the other is labelled as Reason R. Assertion (A): Leydig cells synthesise and secrete male testicular hormones called androgens. Reason (R): Androgens, stimulate the process of spermatogenesis. In the light of the above statements, choose the most appropriate answer from the options given below: (A) Both A and R is correct and R is the correct explanation of A. (B) Both A and R are correct are R is NOT the correct explanation of A. (C) A is correct but R is not correct. (D) A is not correct but R is correct. Ans. Option (A) is correct. Explanation: LH acts on the Leydig cells and stimulates synthesis and secretion of androgens. This happens at the age of puberty due to significant increase in the secretion of gonadotropin releasing hormone there is an increase in the secretion of gonadotropins i.e., LH and FSH. 13. The inactive protoxin gets converted into an active form due to the ................ . (A) Alkaline pH of insect's gut (B) Temperature and acidic pH of gut (C) Exposure to light (D) Exposure to light and acidic pH of gut Ans. Option (A) is correct. Explanation: The Bt toxin is produced by the bacterium Bacillus thuringiensis through the process of sporulation. The Bt toxin protein exist as inactive protoxins but once an insect ingests the inactive toxin, it is converted into an active form of toxin due to alkaline pH of the gut which solubilize the crystals. 14. Which of the following gets embedded in the endometrium during implantation? (A) Zygote (B) Morula (C) Blastocyst (D) Embryo

CUET Solved Paper, 2022 Ans. Option (C) is correct Explanation: The blastocyst gets embedded in the endometrium of the uterus. After attachment, the uterine cells divide rapidly and causes the blastocyst to become embedded in the endometrium of the uterus. This leads to pregnancy. 15. Introduction of which one of the following organism’s species did cause decline or extinction of indigenous species? (A) Eicchornia Crassipes (B) Nile Perch (C) Clarias gariepinus (D) Steller's Sea cow Ans. Option (B) is correct Explanation: The Nile perch introduction into Lake Victoria led eventually to the extinction of ecologically unique assemblage of more than 200 species of cichlid fish in the lake. 16. Match List I with List II List-I List-II A. Progestasert I. once a week pill B. Saheli II. hormone releasing IUD C. Lippe's Loop III. Non-medicated IUD D. Periodic IV. Natural method of abstinence birth control Choose the correct answer form the options given below: (A) A - II, B - III, C - I, D - IV (B) A - I, B - II, C - IV, D - III (C) A - II, B - I, C - III, D - IV (D) A - II, B - IV, C - I, D - III Ans. Option (C) is correct Explanation: Progestasert is a hormonereleasing IUD. Saheli is a 'once a week' pill. Lippe's loop is a non-medicated IUD. Periodic abstinence is a natural method of contraception in which couples abstain sexual intercourse from day 10 to 17 of a 28 days menstrual cycle. 17. The vital link that ensures continuity of species between organisms of one generation and the next generation is : (A) Male gamete (B) Female gamete (C) Zygote (D) Syngamy Ans. Option (C) is correct Explanation: Gametes are reproductive cells of a sexually reproducing organisms. The vital link that ensures continuity of species between organisms of one generation and the next generation is the zygote. Whereas, the process of fusion of gamete is called syngamy and it results in formation of diploid zygote.

13

18. In case of COVID positive patients, presence of corona virus is suspected only when the pathogen has produced a disease symptom. But when the symptoms of the disease has not appeared, the corona virus in the body can be detected by– (A) Enzyme linked immuno-sorbent Assay (ELISA) only. (B) Recombinant DNA technology only. (C) Reverse transcriptase Polymerase Chain Reaction (RT-PCR). (D) Widal Test Ans. Option (C) is correct Explanation: As in RT-PCR, RNA of retrovirus is reverse transcripted with the help of enzyme reverse transcriptase. DNA formed is amplified with the help of PCR. So, a minute quantity of corona virus can also be detected with the help of RT-PCR. 19. Bacillus thuringiensis is a biocontrol agent against ________ . (A) Nematode (B) Fungal pathogen (C) Insect pests (D) Bacterial pathogen Ans. Option (C) is correct Explanation: Bacillus thuringiensis is a bio control agent against insect pests. 20. Arrange the following events in the female reproductive cycle in their natural sequence. A. Ovulation B. Growth of corpus luteum C. Sudden increase in level of LH D. Secretion of FSH E. Growth of ovarian follicle and oogenesis Choose the correct answer from the options given below: (A) E - D - A - B – C (B) D - E - C - A - B (C) E - C - A - D – B (D) D - A - C - E – B Ans. Option (B) is correct Explanation: The correct sequence of the events in the female reproductive cycle is: D - Secretion of FSH E - FSH leads to the growth of ovarian follicles into a fully mature Graafian follicle. C - During the mid of menstrual cycle, there is sudden increase in the level of LH (LH surge). A - LH surge leads to ovulation. B - It is followed by luteal phase in which the remaining parts of the Graafian follicle transform as the corpus luteum. 21. Which of the following methods are commonly used in DNA fingerprinting? (A) Genetic transformation (B) PCR and RFLP (C) Bioprospecting (D) Molecular diagnosis Ans. Option (B) is correct

14

OSWAAL CUET (UG) Sample Question Papers, Biology

Explanation: PCR and RFLP are commonly used in DNA fingerprinting to increase the sensitivity of the technique. 22. Which layer of microsporangium is nutritive in function? (A) Epidermis (B) Endothecium (C) Middle Layers (D) Tapetum Ans. Option (D) is correct Explanation: Tapetum provides nourishment to developing pollen grains and it is the innermost wall layer of microsporangium. While layers like epidermis, endothecium and middle layers perform the function of protection and help in dehiscence of anther to release pollen. 23. Match List I with List II List-I List-II A. Catalytic I. Particulate matter converter B. Incinerators II. Organic waste C. Electrostatic III. Hospital Waste precipitator D. Sewage treatment IV. Carbon monoxide plant and nitrogen oxides Choose the correct answer form the options given below: (A) A - III, B - II, C - IV, D - I (B) A - IV, B - III, C - I, D - II (C) A - II, B - III, C - I, D - IV (D) A - I, B - II, C - IV, D - III Ans. Option (B) is correct Explanation: When exhaust passes through catalytic converter then carbon monoxide and nitrogen oxide are converted to carbon dioxide and nitrogen gas. The use of incinerator is crucial for disposal of hospital waste. Incineration is the high temperature burning of a waste. Electrostatic precipitator can remove over 99% of particulate matter. They worked on the principle of the attraction of a charged particle for an oppositely charged collector. In sewage treatment plant biodegradation of organic matter occurs by microorganisms. 24. During gene therapy, which vector is used to introduce functional ADA cDNA into lymphocyte (A) Plasmid (B) Bacteriophage (C) pBR322 (D) Retrovirus Ans. Option (D) is correct Explanation: The retrovirus is used as a vector to introduce functional ADA cDNA into the lymphocytes, which are subsequently returned to the patient. 25. The animals that feed on herbivores like insects, birds and mammals in terrestrial ecosystem, are called _____ .

(A) Saprotrophs (B) Primary consumers (C) Secondary carnivores (D) Primary carnivores Ans. Option (D) is correct Explanation: The animals that feed on herbivores like insects, birds and mammals in terrestrial ecosystem, are called primary carnivores. These are also called secondary consumers. Herbivores are primary consumers. 26. Recombinant Proteins are expressed in: (A) Cloning vector (B) Heterology Host (C) Homologous chromosomes (D) Promotor Ans. Option (B) is correct Explanation: Recombinant proteins are expressed in heterologous host. The hosts which have foreign genes are called heterologous host. 27.The chemical carcinogens present in tobacco smoke is the major cause of : (A) AIDS (B) Lung Cancer (C) Allergy (D) Pneumonia Ans. Option (B) is correct Explanation: The chemical carcinogens present in tobacco smoke have been identified as a major cause of lung cancer. 28.Match the features that are required to facilitate cloning of alien DNA into a vector List-I List-II (Features to facilitate) (Cloning Vector) A. Origin of I. Agrobacterium replication (ori) tumefaciens B. Selectable Marker II. Recognition sites commonly used for restriction enzymes C. Cloning sites III. Helps in identifying and eliminating nontransformants D. Vectors for IV. Sequence from cloning genes in where replication plants starts Choose the correct answer form the options given below: Ans. Option (A) is correct Explanation: Origin of the replication (ori) is the sequence of DNA at which replication starts on a chromosome, plasmid or virus. Selectable markers in vector help in identifying and eliminating non-transformants and selectively permitting the growth of the transformants. Cloning sites are the recognition sites commonly used for restriction enzymes.

CUET Solved Paper, 2022 Agrobacterium tumefaciens is a gram negative bacterium, it is used as a cloning vector for plants. It is called as natural genetic engineer of plants. 29. Which of the following is effect of steroid in males? (A) Premature baldness (B) Deepening of voice (C) Excessive hair growth on face and body (D) Enlargement of clitoris Ans. Option (A) is correct Explanation: The side effects of the use of anabolic steroids in males include premature baldness, increased aggressiveness, breast enlargement, etc. 30.Which of the following cells produce antibodies? (A) Monocytes (B) PMNL neutrophils (C) T-lymphocytes (D) B- lymphocytes Ans. Option (D) is correct Explanation: B-lymphocytes produce proteins that act as antibody in response to pathogens into our blood to fight with them. These proteins are called antibodies. The T-cells themselves do not secrete antibodies but help B cells to produce them. Neutrophils and monocytes are phagocytic cells. 31. Nutrient cycles are of two types— (A) Gaseous and solid (B) Liquid and sedimentary (C) Gaseous and sedimentary (D) Aquatic and Gaseous Ans. Option (C) is correct Explanation: Nutrient cycles are of two types: Gaseous cycle - Exchange of nutrients occur in gaseous or vapour form. Reservoir pool is atmosphere or hydrosphere. e.g., Nitrogen; Carbon, Oxygen and Hydrogen cycle. Sedimentary cycle - Sedimentary cycles are a type of biogeochemical cycle, in which the reservoir is Earth's crust. e.g., Sulphur and Phosphorus cycle. 32. Technology of biogas production in India was developed due to the efforts of— A. GEAC B. ICAR C. IARI D. IRRI E. KVIC Choose the correct answer from the options given below: (A) A and B only (B) C and D only (C) B and D only (D) C and E only Ans. Option (D) is correct Explanation: The technology of biogas production was developed in India due to the efforts of Indian Agricultural Research Institute (IARI) and Khadi and Village Industries Commission (KVIC).

15

33. The bacterium responsible for breakdown of cellulose in a biogas plant is: (A) Acetobacter aceti (B) Lactobacillus (C) Clostridium (D) Methanobacterium Ans. Option (D) is correct Explanation: The bacteria that are collectively called methanogens and one such bacterium responsible for breakdown cellulose in a biogas plant is Methanobacterium. These bacteria grow anaerobically on cellulosic material produce large amount of methane along with Co2 and H2 after breakdown. 34. With reference to processing of hn RNA, which of the following statements is/are INCORRECT? A. Introns are removed and exons are joined directly splicing. B. Capping and Tailing occurs at 5’ end and 3’ end respectively. C. Addition of 200-300 adenylated residues means capping. D. Addition of guanosine triphosphate takes place at 5’ end. E. Processing take place in the nucleus and converts hn RNA into functional RNA. Choose the correct answer from the options given below: (A) B only (B) C only (C) B and C only (D) B and D only Ans. Option (B) is correct Explanation: Splicing involves cutting off introns and linking exons in a specific order takes place during the processing of human RNA. At the 5’ end of hn RNA, methyl guanosine triphosphate is added. It is known as capping. Tailing occurs by the addition of adenylate residues of about 200-300 at 3’ end of hn RNA. The mRNA, or completely processed in RNA, is transferred outside of the nucleus for translation. 35. Match List I with List II List-I List-II A. Alec Jeffreys I. Lac Operon B. Francois II. Deciphering of Jacob and genetic code Jacque Monod C. Marshall III. Semiconservative Nirenberg replication of DNA D. Meselson IV. DNA Stahl Fingerprinting Choose the correct answer form the options given below: (A) A-IV, B-I, C-II, D-III (B) A-I, B-III, C-IV, D-II (C) A-II, B-III, C-I, D-IV (D) A-IV, B-II, C-III, D-I Ans. Option (A) is correct Explanation: Alec Jeffreys pioneered the DNA fingerprinting process.

16

OSWAAL CUET (UG) Sample Question Papers, Biology

The Operon model of gene control in bacteria was proposed by Jacob and Monod. By converting a synthetic poly U RNA into polyphenylalanine, Nirenberg and Matthaei were able to interpret the genetic code. Meselson and Stahl demonstrated that E. coli DNA replication follows a semi-conservative paradigm. 36. The pyramid of biomass in sea is generally inverted because: (A) Sunlight is filtered through sea water leading to less photosynthesis. (B) Of the high salt content of sea water. (C) The biomass of phytoplankton far exceeds that of fishes. (D) The biomass of fishes far exceeds that of phytoplankton. Ans. Option (D) is correct Explanation: In sea ecosystem, since fishes have much larger bodies and longer lifespans than producers (phytoplankton) and primary consumers (zooplanktons), the biomass pyramid in the marine ecosystem is typically inverted. 37. One of the following is NOT a characteristic/criteria of genetic materials, identify it: (A) Genetic material should be able to generate its replica. (B) Genetic material should be stable chemically and structurally. (C) It should not provide the scope for mutations. (D) It should be able to express itself in the form of Mendelian characters. Ans. Option (C) is correct Explanation: The following requirements must be met by a molecule before it may serve as genetic material; It needs to be stable both chemically and structurally. It should be able to create a copy of itself. It ought to allow for the kind of slow mutation that evolution requires. It ought to be able to manifest itself as Mendelian characteristics. 38. Adenine pairs with Thymine through how many hydrogens bonds? (A) 2 (B) 3 (C) 4 (D) 5 Ans. Option (A) is correct Explanation: Thymine base pair is held together by 2 hydrogen bonds while the GuanineCytosine base pair is held together by 3 hydrogen bonds. 39. A pure breeding garden pea plant was crossed with a pure dwarf plant. The plant produced 400 seeds. The seeds were sown to produce plants. The phenotype of the plants in next generation will be (A) All tall

(B) All dwarf (C) 300 tall and 100 dwarf plants (D) All plants of intermediate height Ans. Option (A) is correct Explanation: Tallness and dwarfness in pea plants are dominant and recessive characters respectively. The cross can be represented as follows:

40. If in a pond there were 150 carps found last year and through reproduction 450 new carps are added in the pond, what will be the birth rate here? (A) 4 offsprings per carp per year (B) 3 offsprings per carp per year (C) 2 offsprings per carp per year (D) 1 offspring per carp per year Ans. Option (B) is correct Explanation: Number of individuals last year (N) = 150 Number of individuals added in one year (N) = 450 Time period (T) = 1 Year 450 Birth rate = 150 = 3 offsprings per carp per year.

41. When does the life appear on the earth after its formation? (A) After 200 million years (B) After 300 million years (C) Atter 500 million years (D) After 250 million years Ans. Option (C) is correct Explanation: Life appeared 500 million years after the formation of earth, i.e., almost four billion years ago. 42. According to the early Greek thinkers, the unit of life which were transferred to different planets were— (A) Spores (B) Water (C) Oxygen (D) Methane Ans. Option (A) is correct Explanation: According to the early Greek thinkers, the unit of life called spores were transformed to different planets including earth. It is still a favorite idea of some astronomers. 43. Louis Pasteur by careful experimentation demonstrated that life comes from— (A) Killed yeast (B) Rotting matter (C) Other plants (D) Pre-existing life Ans. Option (D) is correct Explanation: Louis Pasteur through careful experimentation proved that life originates from pre-existing life. According to the theory of

CUET Solved Paper, 2022 spontaneous generation, living things develop from rotting and decomposing waste. Louis Pasteur used experiments to refute this. 44. Oparin and Haldane proposed the theory that the first form of life could have come from— (A) Non-living organic molecules (B) Inorganic molecules (C) UV rays (D) CO2 and water Ans. Option (A) is correct Explanation: Oparin and Haldane hypothesized that the emergence of life was preceded by chemical evolution and that the earliest forms of life may have evolved from pre-existing nonliving molecules (such as RNA, protein, etc.). 45. The theory of Chemical evolution was studied and tested in laboratory by— (A) Charles Darwin (B) S.L. Miller (C) Louis Pasteur (D) Haldane Ans. Option (B) is correct Explanation: S.L. Miller provided experimental evidence of the chemical evolution in 1953 in a laboratory set-up based on the hypothesis proposed by Oparin and Haldane., 46. The explant used in tissue culture must show _____. (A) Encystation (B) Sporulation (C) Dioecy (D) Totipotency Ans. Option (D) is correct Explanation: The capacity to generate a whole plant from any cell/ex plant is called totipotency. Therefore, the explant used in tissue culture must show totipotency. 47. Virus free plants of banana are developed by using _______ . (A) Protoplasts (B) Meristem (C) Cotyledon (D) Leaf

17

Ans. Option (B) is correct Explanation: Through meristem culture, it is possible to obtain plant clones that are free of viruses, since meristems have a high concentration of auxins and they divide quickly. 48. Pomato is developed by ______ technique. (A) Micropropagation (B) Mutation breeding (C) Biofortification (D) Somatic hybridization Ans. Option (D) is correct Explanation: Pomato was developed by somatic hybridization (protoplast fusion) of two different genera tomato and potato. It is intergeneric somatic hybrid. 49. In tissue culture, the nutrient medium usually contains ________ as a carbon source. (A) Sucrose (B) Maltose (C) Carbon dioxide (D) Calcium carbonate Ans. Option (A) is correct Explanation: In tissue culture, the nutrient medium-usually contains sucrose as carbon source, Sol. In tissue culture, sucrose is typically used as a carbon source in the nutritional media. 50. Protoplasts are obtained by digestion of ________ of the cells. (A) Nuclei (B) Plasma membranes (C) Cell walls (D) Proteins Ans. Option (C) is correct. Explanation: The two cells of the target plants are first treated with the pectinase and cellulase enzymes in preparation for protoplast fusion. These enzymes break down the cell wall, resulting in the creation of bare protoplasts.

❑❑

CUET Question Paper 2022 NATIONAL TESTING AGENCY 20th August 2022—Slot-1

Biology [This includes Questions pertaining to Domain Specific Subject Only]

SOLVED . Time Allowed: 3 hrs.

Maximum Marks: 200

General Instructions: 1.

The test is of 45 Minutes duration.

2.

The test contains is 50 Questions out of which 40 questions needs to be attempted.

3.

Marking Scheme of the test: a.

Correct answer or the most appropriate answer: Five marks (+5).

b.

Any incorrect option marked will be given minus one mark (–1).

c.

Unanswered/marked for review will be given no mark (0).

1. Arrange the following options in a sequential

B.

Fimbriae

II.

manner to demonstrate degradation caused by improper resource utilisation and maintenance.

C.

Clitoris

III. Site of fertilisation

D. Cervical Canal

Along with vagina is birth canal

IV. Helps in collection of ovum after ovulation

A.

Barren patches of land that become large and lead to desertification.

B.

Fertile top soil can be removed easily by over cultivation.

Choose the correct answer from the options given

Deposition of thin crust of salt on land surface.

(A) A-IV, B-I, C-II, D-III

C.

D. Unrestricted grazing of animals. E.

Irrigation without proper drainage of water leading to water logging in the soil.

Choose the correct answer from the options given below: (A) A, B, C, D, E

(B) C, D, A, B, E

(C) B, D, E, C, A

(D) E, D, B, C, A

Ans. Option (C) is correct Explanation: The loss of top soil due to wind, rain and other forces is a natural process but some other activities like over-cultivation, unrestricted grazing of animals, poor irrigation practices result in barren patches of land. Barren lands’ refers to area of land where plant growth is sparse, stunted and/or contain limited biodiversity. When large barren patches extend over time, desert is created.

2. Match List-I with List-II List-I A.

Ampulla

List-II I.

Present at the upper part of labia minora

below: (B) A-III, B-IV, C-I, D-II (C) A-I, B-II, C-IV, D-III (D) A-II, B-III, C-I, D-IV Ans. Option (B) is correct Explanation: The fertilisation of the ovum with sperm takes place in the ampulla region of the fallopian tube. As the egg is released (a process called ovulation) it is captured by finger-like projections of the end of the fallopian tubes (fimbriae). The fimbriae sweep the egg into the tube. Clitoris, is a small, sensitive protrusion, where two labia minora meet. The cervical canal passes through the cervix [the lower end of the womb (uterus)]. It helps the baby to pass from the womb into the vagina. Thus, along with vagina it is a birth canal.

3. Mucosa associated lymphoid tissue constitutes .................... % of lymphoid tissue in human body. (A) 10%

(B) 20%

(C) 40%

(D) 50%

Ans. Option (D) is correct Explanation: MALT (Mucosa Associated Lymphoid Tissue), is a diffuse system of small concentrations of lymphoid tissue present in

CUET Solved Paper, 2022 various submucosal membrane sites of the body, such as breast, lung, eyes, etc. They constitute about 50% of the lymphoid tissue in human body.

4. Which of the following is INCORRECT statement? A.

In IUD’s, released Cu ions increases sperm motility and fertilising capacity of sperms.

B.

Multiload 375 is medicated IUD.

C.

Lippes loop is non-medicated IUD.

D. LNG-20 is hormones releasing IUD. E.

Vault is an IUD.

Choose the correct answer from the options given below: (A) A & E only

(B) A & B only

(C) B & D only

(D) B & C only

Ans. Option (A) is correct Explanation:. There are three different IUD types: (i)

Non-medicated IUDs—These devices, such as Lippes loops, promote the phagocytosis of sperm within the uterus.

(ii) Copper-releasing IUDs—In addition to phagocytosis of the sperms, the released copper ions also inhibit sperm motility and sperm fertility. For example: Multiload-375, Cu-T, and Cu-7. (iii) Hormone-releasing IUDs, such as progestasert and LNG-20, render the uterus unsuitable for implantation and the cervix hostile to sperm. Vaults are a type of barrier method which blocks the entry of sperm and prevents conception.

5. Which of the following is not a vegetation propaganda? (A) Runner of grass (B) Offset of water hyacinth (C) Rhizome of Ginger (D) Bud of Hydra Ans. Option (D) is correct Explanation: Considering the misprinted words vegetation propaganda as vegetative propagules. Vegetative propagation is a form of asexual reproduction. It is a process in which plants reproduce from stem, roots and leaves. In plants, the units of vegetative propagation are runner, rhizome, sucker, tuber, offset, bulb etc. as they are capable of giving rise to new offspring. Buds in Hydra are asexual reproductive structures.

6. Select the correct sequence of development of following stages in a primary succession.

A.

Scrub Stage

B.

Marsh-meadow stage

C.

Submerged free floating plant stage

19

D. Reed-swamp stage E.

Submerged plant stage

Choose the correct answer from the options given below (A) C, E, B, D, A

(B) E, C, D, B, A

(C) D, E, B, C, A

(D) E, C, D, A, B

Ans. Option (B) is correct Explanation: Ecological succession is the gradual change in the species composition of a given area. In primary succession in water, the pioneers are replaced with time by rooted submerged plants (submerged plant stage)— Submerged free floating plant stage—Reed swamps stage—Marsh meadow stage—scrub stage—Forest.

7. Which of the following process involves the theory of antigen-antibody interaction? (A) PCR

(B) ELISA

(C) Gene Therapy

(D) Gel electrophoresis

Ans. Option (B) is correct Explanation: ELISA (Enzyme Linked Immunosorbent Array), is a test that detects and measures antibodies in the blood. Antibodies are proteins, that body produces in response to harmful substances called antigens. It helps to examine the presence of antibodies in the body, in case of certain infectious diseases. PCR (Polymerase Chain Reaction) is a test used to detect genetic material from a specific organism, such as a virus. Gene therapy treats or prevents disease by correcting the underlying genetic problem. Gel electrophoresis is used to separate mixtures of DNA, RNA or proteins according to their molecular size.

8. Following are the animals that recently became extinct, except .................... . (A) Dodo

(B) Quagga

(C) Thylacine

(D) Clarias garipinus

Ans. Option (D) is correct Explanation: Clarias garipinus is indigenous to the inland waters of much of Africa and they are also endemic in Asia Minor in countries such as Israel. It is commonly known as African Catfish. It is a hardy fish that can be densely stocked in low oxygen waters making it ideal for culture in areas with a limited water supply. It's high fecundity, fast growth rate resistance to disease, etc., lead to

20

OSWAAL CUET (UG) Sample Question Papers, Biology its illegal introduction for agriculture purposes is posing a threat to it.

9. Given below are two statements Statement 1: Baculoviruses belonging to genus Nucleopolyhedrovirus are used as biological control agent. Statement 2: These viruses are used as they are species-specific and have narrow spectrum insecticidal applications, so can be a good insecticide. In the light of above statements, choose the most appropriate answer from the options given below (A) Both statement 1 and statement 2 are correct. (B) Both statement 1 and statement 2 are incorrect. (C) Statement 1 is correct and statement 2 is incorrect. (D) Statement 1 is incorrect and statement 2 is correct. Ans. Option (A) is correct Explanation: Baculoviruses are biological insecticides used to control pests insects in agriculture and horticulture. The majority of baculoviruses which are used as biological control agents belong to the genus Nucleopolyhedrovirus. They are species-specific, narrow spectrum insecticidal applications. They do not show negative effects on mammals, birds, plants, fish or even non-target insects.

10. Complete the analogy Migration : Siberian Cranes : : .................... : Bear (A) Regulation

(B) Hibernation

(C) Aestivation

(D) Diapause

Ans. Option (B) is correct Explanation: Siberian crane is the world's third rarest crane. It is listed in IUCN red data list as critically endangered species. It migrates from Siberia to Keoladeo Ghana National Park in Rajasthan, India to avoid harsh weather conditions. Hibernation also known as winter sleep, is a state of minimal activity. It is a seasonal heterothermy characterized by low body-temperature. Hibernation is the best way to survive the flood scarcity each winter. Bear escapes in time during winter by hibernation process.

11.Development of analogous structure or organs are a result of .................... . (A) Divergent evolution (B) Convergent evolution (C) Adaptive radiation (D) Saltation Ans. Option (B) is correct Explanation: Analogous organs, are those organs which have similar function but are different in

anatomical structures, e.g., wings of an insect and bird. Convergent evolution is the independent evolution of similar features in species of different periods. Thus, convergent evolution creates analogous structures.

12.The treatment of the bacterial cell with divalent cations leads to: (A) Increase in hydrophilic nature of DNA molecule. (B) Decreases efficiency with which DNA enters the bacterium. (C) Change in permeability of cell wall. (D) Increased efficiency with which DNA enters the bacterium. Ans. Option (D) is correct Explanation: The cell membrane is composed of a bilayer of phospholipids. The hydrophilic heads of the phospholipids are arranged on either side of the membrane. The hydrophobic tails are towards the inner side, to protect them from the surrounding environment. The DNA being a hydrophilic molecule unable to pass through the hydrophobic lipid bilayer of the cell membrane. Addition of divalent cations like calcium or magnesium, causes the condensation or shrinkage of DNA molecules and the positive charge of these divalent molecules facilities the crossing of the negatively charged cell membrane.

13. A person complaining of stomach ache, weakness and sustained high fever was recommended Widal test by the doctor. Identify the pathogen responsible for the disease the person is suffering from: (A) Streptococcus pneumonia (B) Rhino Virus (C) Salmonella typhi (D) Plasmodium vivax Ans. Option (C) is correct Explanation: The Widal test, developed in 1896 was named after its inventor, Georges-Fernand Widal. It is an agglutination test which detects the presence of serum agglutinins (H and O) in patients serum with typhoid and paratyphoid fever. It is characterized by sustained high fever (39° to 40°C), weakness, stomach pain, constipation, headache and loss of appetite. Salmonella typhi is a pathogenic bacterium which causes typhoid fever.

14. If 100 deaths take place in a population of 10,00,000 per year, then the death rate will be: (A) 0.001 per person per year (B) 0.0001 per person per year (C) 0.1 per person per year (D) 1 per person per year

CUET Solved Paper, 2022 Ans. Option (B) is correct

where

N = Population density at time t

Explanation: The death rate

Number of deaths per year = Total population

21

r = Intrinsic rate of natural increase K = Carrying capacity

18. In the life cycle of Plasmodium, the gametocytes

100 = 10, 00, 000

develop in

= 0.0001 person per year.

(B) RBC of human host

(A) WBC of human host

15. The genetic variation shown by medicinal plant Rauwolfia vomitoria growing in Himalayan ranges, is an example of ..................... (A) Ecosystem Diversity (B) Ecological Diversity (C) Species Diversity (D) Genetic Diversity Ans. Option (D) is correct Explanation: Genetic variation refers to differences in the genetic makeup of individuals in a population. It is necessary in natural selection. Genetic diversity is the diversity or variability within species community or assemblage. The diversity shown by the medicinal plant Rauwolfia vomitoria growing in different Himalayan ranges in an example of genetic diversity.

16. Prime contaminant of lake for eutrophication is :

(C) Gut of mosquito (D) Saliva of anopheles mosquito Ans. Option (B) is correct Explanation: Male and female gametocytes are the components of the malaria parasite life cycle which are taken up from an infected host bloodstream i.e., RBC of human host by mosquitoes and thus mediate disease transmission.

19. Match List I with List II List-I A.

Chilli

I.

Himgiri

B.

Cauliflower

II.

Pusa sadabahar

C.

Brassica

III. Pusa snowball K-1

D. Wheat

(A) A-I, B-III, C-IV, D-II

(B) Algal bloom

(B) A-II, B-VI, C-I, D-III

(C) Nitrate and phosphate

(C) A-II, B-III, C-I, D-II

Ans. Option (C) is correct Explanation: Eutrophication is a natural process in which an entire body of water, or its port get enriched with minerals and nutrients. The prime contaminants that caused such eutrophication are nitrates and phosphates. It leads to an increased algal growth, as the level of nutrients increases.

17. In Verhulst Pearl logistic growth curve equation, dN K−N = rN   , ‘r’ refers to: dt  K  (A) Population density at time t (B) Population density at time Zero

IV. Pusa Swarnim

Choose the correct answer form the options given below

(A) Dissolved oxygen

(D) Fungi and Bacteria

List-II

(D) A-IV, B-III, C-I, D-II Ans. Option (C) is correct Explanation: None of the options matches the crops with their varieties given above. As per NTA option C is correct, as first two crops are matched correctly. Crops

Varieties

1.

Chilli

Pusa Sadabahar

2.

Cauliflower

Pusa Snowball K-1

3.

Brassica

Pusa Swarnim

4.

Wheat

Himgiri

20. Identify the step involved in polymerase chain reaction from the given figure below

(C) Intrinsic rate of natural increase (D) Carrying capacity Ans. Option (C) is correct Explanation: Verhulst-Pearl Logistic Growth curve equation is,

dN K−N = rN   dt  K 

(A) Primer extension

(B) Denaturation

(C) Extension

(D) Annealing

Ans. Option (D) is correct Explanation: In the figure shown above, complementary base pairing between the primer

22

OSWAAL CUET (UG) Sample Question Papers, Biology and complementary part of the single strands of the DNA template is taking place. Thus, this process is known as annealing.

21. ‘Rivet popper’ hypothesis was proposed by

based on the fact that the process of ovulation does not occur during the period of intense lactation.

25. The Graafian follicle ruptures to release the ovum

(A) Alexander Von Humboldt

from the ovary by the process called

(B) Paul Ehrlich

(A) Ovulation

(B) Menstruation

(C) Edward Wilson

(C) Implantation

(D) Copulation

Ans. Option (A) is correct

(D) Robert May Ans. Option (B) is correct Explanation: ‘Rivet popper hypothesis’ was proposed by Paul Ehrlich, which is an analogy to understand the importance of each and every species in an ecosystem.

Explanation: Ovulation is defined as the process in which a mature Graafian follicle ruptures and releases an ovum (oocyte).

26. Match List I with List II List-I

22. Which of the following genes control corn borer disease? (A) ampR

(B) cry I Ac

(C) cry I Ab

(D) cry II Ab

Ans. Option (C) is correct Explanation: Cry gene codes for the Bt toxin. The proteins encoded by the genes cry I Ab control corn borer.

23. The rate of decomposition in a particular climatic condition is quicker, if detritus is rich in .................... & .................... (A) Sugars, Phosphorus (B) Lignin, Phosphorous (C) Sugars, Nitrogen (D) Lignin, Chitin Ans. Option (C) is correct Explanation: Decomposition is an oxygenrequiring process. It is the process by which bacteria and fungi break dead organisms into their simple compounds. There are number of factors on which rate of depends like litter quality and environmental conditions, etc. Presence of water-soluble sugars and rich nitrogen content in the soil increases the rate of decomposition.

24. Which of the following is a natural method of birth control?

List-II (Australian Marsupials)

(Placental mammals) A.

Anteater

I.

Tasmanian cat

B.

Lemur

II.

Tasmanian Wolf

C.

Bobcat

III. Spotted cuscus

D. Wolf

tiger

IV. Numbat

Choose the correct answer from the options given below: (A) A-III, B-I, C-II, D-IV (B) A-IV, B-III, C-I, D-II (C) A-IV, B-I, C-III, D-II (D) A-IV, B-II, C-III, D-I Ans. Option (B) is correct Explanation: Numbat (Myremecobius fasciotus) also known as banded anteater, is one of the few diurnal Australian marsupials. Anteater shows convergent evolution with numbat. The common spotted cuscus (Spilocuscus maculatus) also known as white cuscus. Lemurs are a unique group of primates. They show convergent evolution with spotted cuscus. The bobcat (Lynax rufus) also known as red lynax is a medium-sized cat. It showed convergent evolution with Tasmanian tiger cat. Wolf shows convergent Tasmanian wolf.

evolution

with

27. Select the statement that are correct for GM plants with genetic modifications.

(A) Condoms

A.

Made crops tolerant to abiotic stresses.

(B) Lactational amenorrhea

B.

Increased the reliance on chemical pesticides.

(C) IUD

C.

Helped to reduce post-harvest losses.

(D) Diaphragms

D. Decrease efficiency of mineral usage by plants.

Ans. Option (B) is correct Explanation: Lactational amenorrhea is also known as postpartum infertility Lactation means your body is making breastmilk and amenorrhea refers to absence of monthly period in females. It is a method of natural contraception, which is

E.

Enhanced nutritional value of food.

Choose the correct answer from the options given below: (A) C and E only

(B) B and C only

(C) A, C and E only

(D) C, D and E only

CUET Solved Paper, 2022 Ans. Option (C) is correct Explanation: GM is a technology that involves inserting DNA into the genome of an organism. To produce a GM plant new DNA is transferred into plant cells. The aim is to introduce a new trait to the plant which does not occur naturally in the species. E.g., in food crops, it include resistance to certain pests, made crops tolerant to abiotic stresses, reduction of spoilage or improving the nutrients content of the crop.

28. Father’s blood group is A and mother’s blood group is B. One of their offspring is with group AB. What is the percentage of probability of AB blood group offspring? (A) NIL

(B) 25%

(C) 50%

(D) 75%

Ans. Option (B, C) are correct Explanation: If father's blood group is A and mother's blood group is B. There would be 4 combinations of genotype Case 1: Both father and mother are heterozygous Parents—

IAI0 × IBI0

F1 generation—

IAIB IAI0 IBI0 I0I0

(25% offspring of blood group AB 25% offspring of blood group A 25% offspring of blood group B) Case 2: Father homozygous (IAIA) and mother is heterozygous (IBI0) Parents—

IAIA × IBI0

F1 generation— IAIB IAI0 IAIB IAI0 (50% offspring of AB blood group and 50% offspring are of A blood group) Case 3: Father heterozygous (IAI0) and mother is homozygous (IBIB) Parents—

IAI0 × IBIB

F1 generation— IAIB IAIB IBI0 IBI0 (50% offspring of AB blood group and 50% of B blood group). Case 4: Both father and mother are homozygous, i.e., IAIA, IBIB Parents—

IAIA × IBIB

F1 generation— IAIB IAIB IAIB IAIB (All offspring will be AB blood group)

29. The embryo with 8 to 16 blastomeres is called .................... . (A) Morula

(B) Blastula

(C) Zygote

(D) Foetus

Ans. Option (A) is correct Explanation: Morula is a solid mass of blastomeres, produced by a series of cleavage

23

divisions of the early embryo. The embryo with 8 to 16 blastomeres is called morula.

30. The species that invades a bare area is called .................... . (A) Seral stage

(B) Alien species

(C) Endemic species

(D) Pioneer species

Ans. Option (D) is correct Explanation: The species that invade a bare area is called pioneer species.

31. Mr. X wants to start Bee-keeping. Important points for successful bee-keeping are .................... . A.

Selection of suitable location for keeping beehives.

B.

Beehives can be kept in fruit orchards.

C.

Catching and hiving of swarms.

D. Management of beehives during rainy season. E.

Regular visit by veterinary doctor is mandatory.

Choose the correct answer from the options given below: (A) A, B, C and E

(B) A, B and C

(C) A, B, D and E

(D) A, C, D and E

Ans. Option (B) is correct Explanation: The following points are important for successful bee-keeping: (i)

Nature and habits of bees should be known.

(ii) Suitable location for keeping the beehives. (iii) Management of beehives during different seasons. (iv) Catching and living of swarms. (v) Keeping beehives in fruit orchards increases pollination efficiency and improves the yield.

32. Given below are two statements; one is labelled as assertion A and other is labelled as Reason R Assertion A: In gel-electrophoresis, DNA fragments are separated. Reason R: DNA is negatively charged, so it moves towards the cathode under the influence of electric field. In the light of the above sentences, choose the correct answer from the options given below (A) Both A and R are true and R is the correct explanation of A (B) Both A and R are true and R is not the correct explanation of A (C) A is true but R is false (D) A is false but R is true

OSWAAL CUET (UG) Sample Question Papers, Biology

24

Ans. Option (C) is correct

Ans. Option (B) is correct

Explanation: Fragments of DNA can be separated by a technique known as get electrophoresis. As DNA is a negatively charged molecule, hence it moves towards the positive electrode (anode) not towards the negative electrode (cathode) under an electric field through a medium matrix.

33. Identify the drug shown below in the diagram

Explanation: Phenylketonuria is an autosomal recessive disorder. The individuals suffering from this disorder lack an enzyme that converts the amino acid phenylamine into tyrosine. Down's syndrome is caused due to an additional copy of the chromosome number 21 (trisomy of 21) Turner's syndrome is caused due to the absence of one of the X-chromosomes, i.e., 45 with X0 (monosomy of X-chromosome) Hemophilia is sex-linked recessive disease.

36. The first restriction endonuclease isolated was (A) Eco RI

(B) pBR 322

(C) Hind II

(D) Bam HI

Ans. Option (C) is correct

(A) Cocaine

(B) Heroin

(C) Cannabinoid

(D) Morphine

Explanation: The first restriction endonuclease isolated was Hind II. It always cut DNA molecules at a particular point by recognizing a specific sequence of six base pairs.

37. Which of the following are true, caused due to Inbreeding depression?

Ans. Option (D) is correct Explanation: The given structure is a morphine.

34. Which of the following does not show ecological diversity?

A.

Reduces fertility and productivity.

B.

Helps in accumulation of inferior genes.

C.

Helps to evolve a hybrid in any animal.

D. Increases productivity inbreed population.

(A) Coral reef

(B) Alpine meadows

E.

(C) Wetlands

(D) Rauwolfia vomitoria

Choose the correct answer from the options given below:

Ans. Option (D) is correct Explanation: The medicinal plant Rauwolfia vomitoria show genetic diversity having different concentration and potency of active chemical reserpine.

35. Match List I with List II List-I

List-II

A.

Phenylketonuria

I.

Sex linked recessive disease

B.

Down’s syndrome

II.

Autosomal Recessive disease

C.

Turner’s syndrome

III. Trisomy chromosome number 21

D. Hemophilia

of

IV. Monosomy of Xchromosome

Choose the correct answer from the options given below: (A) A-I, B-III, C-IV, D-II (B) A-II, B-III, C-IV, D-I (C) A-II, B-IV, C-III, D-I (D) A-III, B-II, C-IV, D-I

Retention of less desirable genes.

(A) B and C only

(B) A and D only

(C) C and E only

(D) B and E only

Ans. Option (B) is correct Explanation: Inbreeding depression in the reduced biological fitness of an individual or in other words inbreeding depression is the reduced survival and fertility of offspring of related individuals. It helps in elimination of less desired genes, thus, where there is selection at each step, it increases the productivity of inbreed population.

38. Which of the following microbe is used in production of antibiotics? A.

Penicillium notatum

B.

Streptomyces griseus

C.

Aspergillus niger

D. Clostridium butylicium E.

Saccharomyces cerevisiae

Choose the correct answer from the options given below: (A) A and B only (B) C and D only (C) B and E only

(D) C and E only

CUET Solved Paper, 2022 Ans. Option (A) is correct Explanation: Alexander Fleming discovered penicillin, the first natural antibiotic in 1928. Penicillium notatum (fungus) produces penicillin, the first antibiotic. Streptomyces griseus is a species of bacteria, they are well known producers of antibiotic, streptomycin. It is the first antibiotic ever reported from a bacterium. 39. Select the correct statements from the following A.

The presence of Thymine at the place of Uracil gives more stability to DNA.

B. C.

Both nucleic acids, i.e., DNA and RNA mutate. DNA is dependent on RNA for synthesis of proteins. D. RNA mutate at slower rate than DNA. E. DNA is better genetic material. Choose the correct answer from the options given below: (A) A and D only (B) A and B only (C) A, B, C and D only (D) A, B, C and E only Ans. Option (D) is correct Explanation: DNA is better genetic material. Structurally and chemically DNA is more stable than RNA. This additional stability to DNA is due to the presence of thymine at the place of uracil. DNA mutate slowly with respect to RNA. 2-OH groups in RNA makes it easily degradable. DNA, is dependent on RNA for synthesis of proteins. 40. Mr. X had purchased a fruit juice bottle from the market and is puzzled to see that is completely clear. It is due to (A) Lipase and Pectinase (B) Amylase and Polymerase (C) Pectinase and Protease (D) Protease and Ligase

41. Match List I with List II List-II

A.

Wind pollution

I.

Bees

B.

Water pollination

II.

Grass

C.

Major pollinator

III. Zostera

insect

D. Tallest flower

Choose the correct answer form the options given below: (A) A-I, B-III, C-II, D-IV (B) A-I, B-III, C-IV, D-II (C) A-II, B-I, C-III, D-IV (D) A-II, B-III, C-I, D-IV Ans. Option (D) is correct Explanation: Wind-pollination or Anemophily is a form of pollination whereby pollen is distributed by wind. Almost all gymnosperms are anemophilous and grasses too. Zostera, commonly called marine eelgrass or seagrass is a water pollinated plant. Bees are dominant biotic pollinating agents. Amorphophallus, plant produces only one petiole with a single umbrella-like compound leaf at the top. Amorphophallus titanum ‘Corpse flower’ has the world's largest unbranched inflorescence with a height of upto (6-7 feet).

42.Yucca plant shows .................... (A) Flower with non-sticky pollen grains (B) Water pollination (C) Large feather stigma (D) Moth is partner to complete life cycle Ans. Option (D) is correct Explanation: Yucca plant and moth show mutualistic relationship. Both are interconnected to each other for their survival. The moth's larvae depend on the seeds of the yucca plant for food, and the yucca plant can only be pollinated by the yucca moth.

43. .................... is aquatic plant showing insect pollination. (A) Hydrilla

(B) Vallisneria

(C) Zostera

(D) Water hyacinth

Ans. Option (D) is correct

Ans. Option (C) is correct Explanation: Clarification, is the removal process of suspended material, is an important step in the production of fruit juice due to its significant effect on the appearance, flavour and commercialization of juice. The enzymes, pectinases and proteases are helpful in clarifying fruit juices. Pectinase, breaks down the pectin polymer structure and reduces the undesirable turbidity. List-I

25

IV. Amarphophallus

Explanation: Pontedria crassipes, commonly known as water hyacinth is an aquatic plant showing insect pollination. Water hyacinth is not buried in the water, so it do not discharge its pollen grains into the water, hence it is not pollinated by water. Pollination takes place with the help of insects and wind.

44.Which of the following is NOT a feature of insect pollinated plant? (A) Light and non-sticky pollen grains. (B) Flowers are large and colourful. (C) Presence of nectar. (D) Flower with fragrance. Ans. Option (A) is correct Explanation: Insect-pollinated flowers are large, attractive, brightly colored petals, flowers with fragrance (which attract insects). Their pollens are often sticky or spiky which help them to get sticked to the legs and body of insects.

26

OSWAAL CUET (UG) Sample Question Papers, Biology

45. Grasses show pollination by

(A) Water (B) Insect (C) Wind (D) Animals Ans. Option (C) is correct Explanation: Grasses show pollination by wind, i.e., they are Anemophilous. PASSAGE Read the sentence and answer the questions. The process of replication requires a set of catalyst (enzymes). The main enzyme is referred to as DNA dependent DNA Polymerase. E.coli that has only 4.6×106 bp (compare it with human whose diploid content is 6.6×106 bp), completes the process of replication within 18 minutes, that means the average rate of polymerisation has to be approximately 2000 bp per second these polymerase also have catalyse the reaction with high degree of accuracy. Deoxyribonucleoside triphosphates serve dual purposes. In addition to DNA dependent DNA polymerases, many additional enzymes are required to complete the process of replication with high degree of accuracy. For long DNA molecules, since the two strands of DNA cannot be separated in its entire length (due to very high energy requirement), the replication occur with in a small opening of the DNA helix, referred to as replication fork. The DNAdependent DNA polymerase catalyse polymerisation only in one direction, that 5’-3’. This creates some additional complications at the replicating fork. Consequently, on one strand (the template with polarity 3’ → 5’) the replication is continuous, which on the others (the template with polarity 5’ → 3’), it is the discontinuous. The non-continuously synthesized fragments are later joined by the enzyme DNA ligase. 46. The DNA dependent-DNA polymerase catalyse polymerisation in one direction only that is .................... (A) 5’-3’ of DNA (B) 3’-5’ of DNA (C) 5’-3’ of RNA (D) 3’-5’ of RNA Ans. Option (A) is correct Explanation: The DNA dependent DNA polymerase catalyse polymerisation only in one direction that is 5’ → 3’ of DNA. 47. In human beings DNA polymerase complete the process of replication in .................... (A) 25 Minutes (B) 16 Minutes (C) 12 Minutes (D) 18 Minutes Ans. Option (D) is correct Explanation: In human beings DNA polymerase complete the process of replication within 18 minutes.

48. The number of nucleotides in E. coli is .................... .

(A) 1.2 × 106 bp (B) 2.3 × 106 bp 6 (C) 4.6 × 10 bp (D) 6.6 × 106 bp Ans. Option (C) is correct Explanation: The number of nucleotide in E.Coli is 4.6 × 106 bp. 49. In E. coli the main enzyme required to catalyze the polymerisation of deoxynucleotide is .................... . (A) DNA-Dependent DNA Polymerase (B) DNA-Dependent RNA Polymerase (C) RNA-Dependent DNA Polymerase (D) RNA-Dependent DNA Polymerase Ans. Option (A) is correct Explanation: In E.Coli the main enzyme required to catalyze the polymerisation of deoxynucleotide is DNA dependent DNA polymerase. 50. Which of the following statements are correct? A. DNA dependent DNA polymerase is a slow catalyst B. Deoxyribonucleoside triphosphate provide energy for polymerisation C. Replication fork is a small opening of the DNA helix D. The replication is discontinuous on DNA template with polarity 3’-5’ E. The replication is continuous on DNA template with polarity 3’-5’. Choose the correct answer form the options given below: (A) A and B only (B) B and C only (C) A and D only (D) C and E only Ans. Option (B) is correct Explanation: The two terminal phosphates in a deoxynucleoside triphosphates are high-energy phosphates, same as ATP. It is also a building block of DNA strand. Since, the two strands of DNA cannot be separated in its entire length (due to high energy requirement), the replication occurs within a small opening of the DNA helix, called replication fork. The replication is continuous on the template strand with polarity 3→5 while on the other strand replication is discontinuous with polarity 5→3. As per NTA option (B) is correct however as per NCERT, statements (B), (C) and (E) are correct but none of the option includes all three statements that are (B), (C) and (E).

❑❑

CUET Question Paper 2022 NATIONAL TESTING AGENCY 30th August 2022—Slot-2

Biology [This includes Questions pertaining to Domain Specific Subject]

SOLVED . Time Allowed: 3 hrs.

Maximum Marks: 200

General Instructions: 1.

The test is of 45 Minutes duration.

2.

The test contains is 50 Questions out of which 40 questions needs to be attempted.

3.

Marking Scheme of the test: a. b. c.

Correct answer or the most appropriate answer: Five marks (+5). Any incorrect option marked will be given minus one mark (–1). Unanswered/marked for review will be given no mark (0).

1. Match List I with List II

List-I

List-I

List-II

A.

Monoecious Plants

I.

Date palm

B.

Unisexual

II.

Leach

C.

Hermaphrodites

III. Cucurbita

D.

Dioecious Plants

IV. Cockroach

Choose the correct answer form the options given below: (A) A-III, B-IV, C-I, D-II (B) A-III, B-IV, C-II, D-I (C) A-I, B-IV, C-II, D-III

D.

Cytokine barrier

List-II IV. Skin

Choose the correct answer from the options given below: (A) A-I, B-IV, C-III, D-II (B) A-II, B-I, C-IV, D-III (C) A-III, B-IV, C-I, D-II (D) A-IV, B-II, C-III, D-I Ans. Option (D) is correct Explanation: Skin is the main physical barrier which prevents entry of the micro-organisms. Natural killer cells are the cellular barriers of innate immunity. Tears from eyes come under the category of physiological barriers.

(D) A-I, B-IV, C-III, D-II Ans. Option (B) is correct Explanation: Cucurbita is a monoecious plant. Plants bearing both male and female sex organs on the same body are called monoecious plants. Cockroach is unisexual. Unisexual organisms have either male or female sex organ on single body. Leach is hermaphrodite. Hermaphrodite are bisexual animals. Date palm is an example of dioecious plant. Plants bearing one of the sex organs i.e., either male or female are called dioecious plants.

2. Match List I with List II List-I

List-II

A.

Physical barrier

I.

Interferons

B.

Cellular barrier

II.

Natural Killer cells

C.

Physiological barrier

III. Tears from eyes

Interferons comes under cytokine barrier. Virus infected cells secrete proteins called interferons which protect non-infected cells from further viral infection.

3. A couple is unable to produce a child as the male partner has low sperm count. Infertility caused by this condition can be treated using: (A) ZIFT

(B) AI

(C) GIFT

(D) IUT

Ans. Option (B) is correct Explanation: Artificial insemination (AI) technique is useful for the male partner having inability to inseminate female or has low sperm counts. It is the method of transferring semen (sperm) collected from the husband or a healthy donor into the vagina or the uterus (IUI-intrauterine insemination) of the recipient Female.

OSWAAL CUET (UG) Sample Question Papers, Biology

28

Zygote Intra Fallopian Transfer (ZIFT) involves transfer of zygote or early embryos (with up to 8 blastomeres) into fallopian tube. Gamete Intra Fallopian Transfer (GIFT) is the method of transfer of gamete (ovum) from a donor into the fallopian tube of another female who is unable to produce ovum, but can provide right conditions for fertilization and development of an embryo. Intra Uterine Transfer (IUT') involves transfer of embryos with more than 8 blastomeres into the uterus. The babies thus produced from this method are known as test tube babies.

4. Which of these are the carriers of male gametes in some seed plants? (A) Microspore mother cells (B) Pollen Grains (C) Anthers

6. Arrange the following steps in order for the technique of DNA finger printing. A.

Isolation of DNA.

B.

Separation of DNA electrophoresis.

C.

Digestion of DNA by restriction endonucleases.

fragments

by

gel,

D. Blotting of separated DNA fragments to nylon membrane. E.

Hybridisation using VNTR probe followed by autoradiography.

Choose the correct answer from the options given below: (A) A,C,B,D,E

(B) A,B,C,D,E

(C) A,B,D,C,E

(D) A,E,C,B,D

Ans. Option (A) is correct

(D) Megaspore mother cells

Explanation: Different fingerprinting are:

Ans. Option (B) is correct Explanation: Pollen grain carries the male gametes which can be transferred to the female gamete via pollination. All the events - from pollen deposition on the stigma until pollen tubes enter the ovule - are together referred as pollen-pistil interaction.

5. Match List I with List II List-I

Filariasis is caused by the nematode, Wuchereria. It spreads by bite of female Culex mosquito.

List-II

A.

Malaria

I.

Salmonella

B.

Pneumonia

II.

Wuchereria

C.

Typhoid

III. Haemophilus

D.

Filariasis

IV. Plasmodium

Choose the correct answer from the options given below: (A) A-I, B-II, C-IV, D-III (B) A-IV, B-III, C-I, D-II (C) A-III, B-II, C-I, D-IV (D) A-II, B-IV, C-I, D-III Ans. Option (B) is correct Explanation: The causative agent of Malaria is Plasmodium, a protozoan. It is a vector borne disease that spreads by biting of the female Anopheles mosquito. The causative agent of Pneumonia is Haemophilus, a bacterium. It spreads by the sputum of the infected person. Typhoid is caused by a bacterium, Salmonella typhi. It spreads through contaminated food and water and migrates to other organs through blood from intestine.

steps

of

DNA

A- Isolation of DNA. (From any cells like blood stains, semen stains or hair roots) C- Digestion of DNA by restriction endonucleases. B- Separation of DNA fragments by gel electrophoresis. D- Transferring (blotting) of separated DNA fragments to synthetic membranes, such as nitrocellulose or nylon and then baked in a vacuum oven at 80°C for 3-5 hours (to fix the DNA fragment on the membrane). E-

Hybridization using labeled VNTR probe, followed by autoradiography.

7. Long term use of alcohol specially causes— (A) Liver cirrhosis (B) Arthritis (C) Pulmonary system damage (D) Premature baldness Ans. Option (A) is correct Explanation: Long term use of alcohol causes liver cirrhosis, as chronic use of alcohol damages nervous system and liver. Arthritis is associated with the inflammation of joints. Pulmonary system damage is associated with smoking tobacco or other forms of air pollutants. Premature baldness can occur due to Deficiency of biotin or growth hormone can cause premature boldness.

8. Select the appropriate options with reference to post fertilisation changes in angiosperms.

CUET Solved Paper, 2022 A.

Inner integument of ovule develops into tegmen.

B.

Ovary wall forms pericarp.

C.

Ovule forms fruit.

29

Ans. Option (D) is correct Explanation: The retroviral life cycle begins in the nucleus of an infected cell A retrovirus inserts its genome into the host’s genome.

D. Zygote forms endosperm.

D - Viral RNA is introduced into the cell.

E.

B - Enzyme reverse transcriptase acts on viral RNA.

Outer integument of ovule develops into Testa.

Choose the correct answer from the options given below: (A) B, E only

(B) A, B, E only

(C) C, D, B only

(D) A, C, B only

Ans. Option (B) is correct Explanation: Post fertilization events includes a.

Development of endosperm from primary endosperm cell.

b.

Development of embryo from zygote

c.

Formation of seed from ovule

d.

At the time of seed formation, an integument develops into seed coats. Outer integument forms testa (outer seed coat) while inner integument forms tegmen (inner seed coat).

e.

Formation of pericarp from ovary wall.

9. The part of the human sperm that helps in the fertilisation of the ovum is a cap-like structure called (A) Head

(B) Acrosome

(C) Middle piece

(D) Tail

Ans. Option (B) is correct Explanation: Acrosome is a special kind of organelle with a cap-like structure that covers the anterior portion of the head of the spermatozoon and contains digestive enzymes It contains lytic enzymes that help in fertilization. Head is oval shaped and consists of a haploid nucleus and a cap like acrosome.

E - Viral DNA produced is incorporated into the host genome. A - New viral RNA is produced by infected cell. C - New virus particles inject into other cells.

11. Secondary treatment of sewage involves (A) Physical removal of large and small particles from sewage through filtration (B) Anaerobic sludge digestion (C) Sedimentation to remove smaller and larger particles (D) Mechanical agitation coupled with addition of aerobic microbes Ans. Option (D) is correct Explanation: Sewage includes two stages: (i)

Primary treatment or plıysical treatment: It involves physical removal of particles from the sewage through filtration and sedimentation.

(ii) Secondary treatment or Biological treatment: The primary effluent is passed into large aeration tanks and constantly agitated to allow vigorous growth of useful aerobic microbes into flocs. Once the BOD of sewage water is reduced significantly, the effluent is then passed into a settling tank where the bacterial flocs are allowed to sediment. This sediment is called Activated sludge.

Middle piece is composed of axial filament surrounded by mitochondria and cytoplasm.

12. Given below two statements: one is labelled as

Tail consists of a central axial filament. The sperm moves in fluid medium of female genital tract by the undulating movement of the tail.

Assertion (A): In F2 generation of a Mendelian cross, the traits seen in the progeny are identical to their parents.

10. Rearrange the events of life cycle of a retrovirus. A.

New viral RNA produced by infected cell.

B.

Reverse transcription.

C.

New virus particles inject into other cells.

D. Viral RNA introduced into the cell. E.

Viral DNA incorporates into host genome.

Choose the correct answer from the options given below: (A) A, B, D, E, C

(B) B, E, D, A, C

(C) C, A, B, D, E

(D) D, B, E, A, C

assertion A and the other is labelled as Reason R

Reason R: The progeny of the F2 generation in a Mendelian cross shows no blending of traits. In the light of the above statements, chose the most appropriate answer from the options given below: (A) Both A and R are correct and R is the correct explanation of A (B) Both A and R are correct but R is NOT the correct explanation of A (C) A is correct but R is not correct (D) A is not correct but R is correct Ans. Option (A) is correct

OSWAAL CUET (UG) Sample Question Papers, Biology

30

Explanation: In monohybrid Mendelian cross, the F1 hybrid always showed one of the parental forms of the trait whereas both the parental forms of the trait (contrasting forms of the trait) appeared without any change in the F2 generation. This occurred because there is no blending of traits. The form of the trail that appeared in the F2 hybrids is called dominant form and it appeared in the F2 generation about three times in frequency as its alternate (recessive) form.

13. In flowering plants, the egg apparatus consists of: (A) Three antipodals and one egg cell (C) Two synergids and one egg cell (D) Three antipodals and two synergids Explanation: The egg apparatus consists of 2 synergids and one egg cell. The synergids have special cellular thickenings at the micropylar tip. These are together called the filiform apparatus. It helps to guide the pollen tubes into the synergid. Similarly, at the chalazal end, three out of the four nuclei differentiate as antipodal cells. The remaining two cells of the micropylar end and the chalazal end) move towards the center and are known as polar nuclei, which are situated in a large central cell. Hence, at maturity, a typical mature angiosperm embryo sac (the female gametophyte) appears as a 7-celled structure, though it has 8-nuclei.

14. Layers surrounding the ovum from outside to inside are B.

Vitelline membrane

C.

Corona radiata

15. The term ribosome is applicable to the following molecule in bacteria (A) mRNA

(B) 23S rRNA

(C) 5S rRNA

(D) hn RNA

Ans. Option (B) is correct Explanation: In prokaryotes, the larger subunit 50S of ribosome consists of 23S and 5S rRNA whereas in eukaryotes, the larger subunit 60S of ribosome has 28S, 5.8S and 5S rRNA.

hnRNA is the precursor of mRNA in eukaryotes.

Ans. Option (C) is correct

Zona pellucida

Corona radiata: It is the outer layer formed of follicle cells. These cells are bound together by hyaluronic acid.

mRNA copies DNA's instructions and carries them to the ribosome where proteins can be made.

(B) Two synergids and one central cell

A.

d.

16. Match List I with List II List-I

List-II

A.

Homo habilis

I.

Used hides to protect their body and buried their dead

B.

Dryopithecus

II.

Hunted with stone weapons but essentially ate fruits

C.

Australopithecines

III. First human-like being

D.

Neanderthal man

IV. Ape-like ancestor

Choose the correct answer from the options given below: (A) A - I, B - II, C - III, D - IV (B) A - III, B - IV, C - II, D - I (C) A - IV, B - III, C - I, D - II (D) A - II, B - I, C - IV, D - III

Choose the correct answer from the options given below: (A) A, C, B

(B) С, В, А

(C) C, A, B

(D) A, B, C

Ans. Option (C) is correct Explanation: Ovum is spherical surrounded by four membranes:

and

is

a.

Plasma membrane (Oolemma), which is the innermost layer.

b.

Vitelline membrane, which is attached to the plasma membrane.

c.

Zona pellucida. It is the transparent noncellular layer found outer to the Vitelline membrane.

Ans. Option (B) is correct Explanation: Homo habilis were the first humanlike being the hominid. They have fully erect posture. Dryopithecus were existing about 15 mya and they were more ape-like. They were hairy and walked like gorillas and chimpanzees. Australopithecines hunted with stone weapons, lived at trees, canines and incisors were small. Neanderthal man was cave dweller, used hides to protect their bodies, and buried their dead.

17. After an HIV infection, the body of the patient starts becoming immunodeficient mainly due to : (A) The damage caused to secondary lymphoid organs.

CUET Solved Paper, 2022 (B) Rapid multiplication of the virus outside the host cells. (C) Progressive decline in the number of helper Tlymphocytes. (D) Infection by a number of pathogens. Ans. Option (C) is correct Explanation: AIDS is caused by HIV transmitted via sexual or blood-blood contact. After entering the human body, the HIV virus attacks and enters the macrophages. Inside the macrophages, the RNA of the virus replicates with the help of enzyme reverse transcriptase and give rise to viral DNA. Then, this viral DNA incorporates into the host DNA and directs the synthesis of virus particles. At the same time. HIV enters the helper T-lymphocytes. It replicates and produces viral progeny here. These newly formed progeny viruses get released into the blood, attacking healthier helper T-lymphocytes in the body. As a result, the number of T-lymphocytes in the body of an infected person decreases progressively, thereby decreasing the Immunity of the body. T-helper cells stimulate B-cells to produce antibodies and induce killer-T-cells to destroy the non-self cells (foreign cells). As a result of the decline in the number of helper T-cells, the individual begins to experience opportunistic illnesses brought on by a decline in immunity.

18. Bioactive molecules like statins are produced by: (A) Trichoderma polysporum (B) Monascus purpureus (C) Acetobacter aceti (D) Streptococcus Ans. Option (B) is correct Explanation: Statins are produced by Monascus purpureus yeast. They are used as bloodcholesterol lowering agents. These bioactive molecules It inhibits the enzymes responsible for synthesis of cholesterol. Trichoderma polysporum a fungus produces an immunosuppressive agent cyclosporin A, which is used as an immunosuppressive agent in for organ transplant patients. Acetobacter aceti a bacterium produces acetic acid. Streptococcus produces the clot buster enzyme streptokinase. It is used to remove clots from the blood vessels of patients who have myocardial infarction.

19. The haplodiploid sex-determination system where all the males of the population are haploid and females are diploid, is seen in (A) Drosophila (B) Honey bees (C) Birds

(D) Grasshoppers

31

Ans. Option (B) is correct Explanation: The haplo-diploid sex determination system is seer in honey bees. In honeybee, the male have half the number of chromosome than that of female. The female are diploid having 32 chromosomes and males are haploid i.e., having 16 numbers of chromosomes. In Drosophila and grasshopper, sexdetermination is XX-XO type. Here, female is homogametic, i.e., XX (all gametes are with X chromosomes) while male is heterogametic, i.e., XO (Gametes with X and gametes without X) In birds, female heterogamety and male homogamety is seen. Female birds have two different sex chromosomes designated as Z and W whereas male birds have two similar sex chromosomes and called ZZ.

20. Match List I with List II List-I

List-II

A.

Genetic drift

I.

Change in allele frequency due to immigration and emigration

B.

Stabilising selection

II.

Change in gene frequency due to chance factor

C.

Gene flow

III. More individuals acquire mean character value

D.

Saltation

IV. Single step large mutation

Choose the correct answer form the options given below: (A) A - I, B – II, C - III, D - IV (B) A - III, B - 1, C - II, D - IV (C) A - I, B - III, C - IV, D - II (D) A - II, B - III, C - I, D - IV Ans. Option (D) is correct Explanation: Genetic drift is defined as sudden change in gene frequency by chance. It is the accidental gene flow causing change in frequency. Stabilizing selection: It is the selection that favours the intermediate phenotype. Hence, both extremes of the variation of a trait are eliminated. This selection results in favouring the average individuals in a population. Gene flow is the change in allele frequency due to immigration and emigration. Saltation is the single step large mutation.

OSWAAL CUET (UG) Sample Question Papers, Biology

32

21. Match List I with List II

Ans. Option (C) is correct

List-I

List-II

A.

Cowpea

I.

Tobacco virus

mosaic

B.

Chilli

II.

Leaf rust

C.

Brassica

III. Bacterial blight

D.

Wheat

IV. White rust

and

stripe

Choose the correct answer form the options given below: (A) A - II, B - III, C - IV, D - I (B) A - I, B - II, C - III, D - IV (C) A - III, B - I, C - IV, D - II (D) A - IV, B - II, C - III, D - I Ans. Option (C) is correct Explanation: The crop bred by hybridisation and selection for particular disease resistance are: Cowpea—Bacterial blight Chilli—Tobacco mosaic virus Brassica—White rust Wheat—Leaf and stripe rust

22. What happened when heat-killed S cells along with

Explanation: Oscillatoria are autotrophic microbes, which can fix atmospheric nitrogen. Trichoderma are free living fungi. They live in the roots of higher plants and protect them from various pathogens. They are effective biocontrol agents of several plant pathogens. Aspergillus are fungi, that forms citric acid. Rhizobium is a symbiotic bacteria found in the root nodules of leguminous plants that has the ability to fix atmospheric Nitrogen.

24. In humans females at the time of birth, the stage of cell cycle of oocyte is (A) Prophase-I

(B) Prophase-II

(C) Meiosis-II

(D) Mitosis

Ans. Option (A) is correct Explanation: Oogenesis is initiated in embryonic stage when millions of egg mother cells (oogonia) are formed within each foetal ovary. No more oogonia are formed and added after birth. These cells (Oogonia) undergo meiosis but get temporarily arrested at the prophase-I stage and are called primary oocytes.

25. Calotropis, a weed growing in abandoned fields,

live R cells were injected into mice in Griffith's experiment?

produces one of the following biochemicals as a defense against grazers and browsers—

(A) Mice died and showed live R cells in the blood.

(A) Nicotine

(B) Mice survived and showed live S cells in the blood.

(C) Cardiac glycosides (D) Strychnine

(C) Mice died and showed live S cells in the blood. (D) Mice survived and showed live R cells in the blood. Ans. Option (C) is correct Explanation: To test for the trait of pathogenicity, Griffith injected mice with mixes of the two strains •

S-strain → Inject into mice → Mice die



R-strain → Inject into mice → Mice live



S-strain (Heat killed) → Inject into mice → Mice live



S-strain (Heat killed) + R-strain (live) → Inject into mice → Mice die

He concluded that some 'transforming principle', transferred from heat-killed S-strain to R strain. It enabled R-strain to synthesize smooth polysaccharide coat and become virulent. This must be due to the transfer of some genetic material.

23. Autotrophic microbe for fixing atmospheric nitrogen: (A) Rhizobium

(B) Aspergillus

(C) Oscillatoria

(D) Trichoderma

(B) Opium

Ans. Option (C) is correct Explanation: Calotropis produces highly poisonous cardiac glycosides as a defense against grazers and browsers.

26. Calculate the birth rate if there were 60 sparrows in a garden last year and through reproduction 24 new sparrows are added this year (A) 0.4 offsprings per sparrow per year (B) 0.2 offsprings per sparrow per year (C) 0.6 offsprings per sparrow per year (D) 0.1 offsprings per sparrow per year Ans. Option (A) is correct Explanation: Birth rate Number of individuals born = Total number of individuals 24 = 60 = 0.4 off springs/ sparrow

27. Totipotency can be defined as— (A) Capacity to generate whole plant from any cell/explant. (B) Capacity to survive environment condition.

in

unfavourable

CUET Solved Paper, 2022

33

(C) Ability to conduct water in 100 meter tall tree.

C.

(D) The duration of dormancy of seeds and other propagules in angiosperms.

Barrier (physical)

III. Syphilis

D.

Venereal diseases

IV. Coitus interruptus

Ans. Option (A) is correct Explanation: Totipotency is the capacity to generate whole plant from any cell/explants. It is the genetic potential of a plant cell to produce the entire plant. In other words, totipotency is the cell characteristic in which the potential for forming all the cell types in the adult organism is retained.

28. Match List I with List II List-I

List-II

A.

Fig and wasp

I.

Competition

B.

Koel and crow

II.

Commensalism

C.

Balanus Chathamalus

D.

Egret and cattle

and

III. Mutualism IV. Brood parasitism

Choose the correct answer form the options given below: (A) A-III, B-IV, C-I, D-II (B) A-I, B-III, C-II, D-IV

Choose the correct answer form the options given below: (A) A-I, B-II, C-IV, D-III (B) A-II, B-I, C-IV, D-III (C) A-II, B-IV, C-I, D-III (D) A-I, B-IV, C-II, D-III Ans. Option (C) is correct Explanation: Sex determination is illegally done by amniocentesis so, there is ban on it in India. Coitus interruptus (withdrawal) involves withdrawal of penis from the vagina just before ejaculation, so as to avoid insemination into the vagina. Vault is a barrier method of contraception. They prevent conception by blocking the entry of sperms through the cervix. Syphilis is a venereal or sexually transmitted bacterial disease. Diseases transmitted through sexual intercourse are called sexually transmitted diseases (STDs).

30. Restriction endonucleases are used as molecular

(C) A-II, B-III, C-I, D-IV

_______ .

(D) A-III, B-II, C-I, D-IV

(A) molecular build up at nucleotides

Ans. Option (A) is correct Explanation: The Fig species is pollinated only by its partner wasp species and no other species. The female wasp pollinates the Fig inflorescence while searching for suitable egg-laying sites in fruits. The Fig offers the wasp some developing seeds as food for the wasp larvae. Hence. Fig and wasp show mutual relationship. Koel and crow show brood parasitism in which parasitic bird lays it’s eggs in the nest of its host and let the host incubate them. Connell's field experiments showed that on the rocky sea coasts of Scotland, the larger and competitively superior barnacle Balanus dominates intertidal area, and excludes the smaller barnacle Chathamalus from that zone. Hence, they show competition.

(B) molecular cement to join DNA sites (C) molecular knife cut DNA at specific site (D) molecular degradation to break up the DNA to nucleotides Ans. Option (C) is correct Explanation: Restriction enzymes are molecular scissors used in molecular biology for cutting DNA sequences from a specific site.

31. Given below is a simplified model of phosphorus cycle in terrestrial ecosystem with four blanks. Identify the blanks A, B, C and D.

The Egret and grazing cattle slow commensalism. The Egrets forage close to where the cattle are grazing. As the cattle move, the vegetation insects come out. Otherwise it is difficult for the Egrets to find and catch the insects.

29. Match List I with List II List-I

List-II

A.

Sex-determination

I.

Vault

B.

Withdrawal

II.

Amniocentesis

(A) A: Producer, B: Litter fall, C: Rock mineral, D: Detritus

OSWAAL CUET (UG) Sample Question Papers, Biology

34

(B) A: Rock minerals, B: Detritus, C: Litter fall, D: Producers (C) A: Litter fall, B: Producer, C: Rock minerals, D: Detritus (D) A: Detritus, B: Rock minerals, C: Producers, D: Litter fall Ans. Option (D) is correct Explanation: In the given simplified model of phosphorus cycling in a terrestrial ecosystem, the marked as “A” is Detritus, label ‘B’ is Rock minerals, label ‘C’ is Producers, and the label D represents Litter fall.

Explanation: Stanley Cohen and Herbert Boyer created the first recombinant DNA using a native plasmid of Salmonella typhimurium. This recombinant DNA was then transferred into Escherichia coli. Thermus aquaticus is a thermostable bacterium whose enzymes are used in PCR. Bacillus thuringiensis is used to make Bt cotton (pest-resistant crop). 34. Match List-I with List-II A. B. C. D.

32. Match List I with List II List-I (Genes in lac operon)

List-II (Products coded for)

A.

i-gene

I.

-galactosidase

B.

z-gene

II.

Transacetylase

C.

y-gene

III. Repressor protein

D.

a-gene

IV. Permease

Choose the correct answer from the options given below: (A) A-I, B-II, C-IV, D-III (B) A-III, B-I, C-IV, D-II (C) A-III, B-IV, C-II, D-I (D) A-I, B-II, C-III, D-IV Ans. Option (B) is correct Explanation: The ‘i’ gene codes for the repressor protein of the lac operon. The ‘z’ gene, -galactosidase hydrolyzes lactose to galactose and glucose. The ‘y’ gene codes for Permease, which increases the permeability of the cell to -galactosidase. 33. Cohen and Boyer created the first recombinant DNA using plasmid of (A) Escherichia coli (B) Thermus aquaticus (C) Salmonella typhimurium (D) Bacillus thuringiensis Ans. Option (C) is correct

List-I Catalytic converter Electrostatic precipitator Earmuffs Land fills

I.

List-II Solid wastes

II.

High noise level

III. Particulate matter IV. Carbon monoxide and nitrogen oxide

Choose the correct answer from the options given below: (A) A-II, B-III, C-I, D-IV (B) A-I, B-IV, C-III, D-II (C) A-IV, B-III, C-II, D-I (D) A-III, B-IV, C-II, D-I Ans. Option (C) is correct 35. Identify the correct statements. A. Oogenesis is initiated at puberty. B. Second meiotic division in secondary oocyte occurs in the ovary. C. Middle piece of sperm provides energy for movement. D. Fertilization of gametes occurs in the ampulla isthmic region of oviduct. E. Myometrium helps in muscular contractions of the uterus. Choose the correct answer from the options given below: (A) A, D, E only (B) A, B, D only (C) B, C, D only (D) C, D, E only Ans. Option (D) is correct Explanation: Catalytic converters are devices fitted in automobiles to reduce vehicular pollution. As the vehicular discharge passes through catalytic converter, carbon monoxide and nitric oxide released by catalytic converters are converted into carbon dioxide and nitrogen dioxide respectively. Electrostatic precipitator is the device widely used to remove particulate matter such as dust, smoke etc., from air using force of an electrostatic charge. It can remove over 99% particulate matter present in the exhaust from the thermal power plant. High noise level in an area is eliminated by using earmuffs. Solid wastes arc trashed in a depression or trench considered as landfills.

CUET Solved Paper, 2022 36. How many gametes will be formed in the pea plant having genotype RrTT? (A) One (B) Two (C) Four (D) Eight Ans. Option (B) is correct Explanation: Oogenesis is initiated in embryonic stage when millions of egg mother cells (oogonia) are formed within each foetal ovary. No more oogonia are formed and added after birth. 2nd meiotic division in secondary oocyte is completed in fallopian tube after entry of sperm into the ovum.3 The middle piece of sperm possesses numerous mitochondria, which produce energy for the movement of tail. Fertilisation can only occur if the ovum and sperms are transported simultaneously to the ampulla-isthmic region of oviduct. Myometrium exhibits strong uterine contractions during delivery of the baby. 37. The detritus food chain (DFC) begins with (A) Green plants (B) Carnivore (C) Dead Organic matter (D) Herbivore Ans. Option (C) is correct Explanation: Detritus food chain begins with detritus such as dead bodies of animals or fallen leaves, which are then eaten by decomposers or detrivores. These detrivores are in turn consumed by their predators. Grazing food chain begins with producers, present at the first trophic level. The plants biomass is then eaten by herbivores which in turn are consumed by a variety of consumers. Carnivores feed on consumers. Herbivores feed on green plants. 38. How many cycles in a PCR will be required for amplifying the genetic material to about one billion times? (A) 10 cycles (B) 15 cycles (C) 30 cycles (D) 35 cycles Ans. Option (C) is correct Explanation: If the process of DNA replication is repeated many times, the segment of DNA can be amplified to approximately billion times, i.e., 1 billion copies are made at the end of 30 PCR cycles. It is possible to generate 2n molecules after 'n' number of cycles. 39. Gene of which one of the following is present exclusively on the X-chromosome in human being (A) Thalassemia (B) Phenylketonuria (C) Red and green colour blindness (D) Sickle cell anaemia Ans. Option (C) is correct Explanation: Red-green colour blindness is an Xlinked recessive Mendelian disorder. Thalassemia, phenylketonuria and sickle cell anaemia are autosomal recessive Mendelian disorders. 40. Biofortification refers to: (A) Sources of proteins for animal and human nutrition.

35

(B) Production of somatic hybrids. (C) Use of mutation breeding. (D) Breeding crops with higher levels of vitamins and minerals, or higher protein and healthier fat. Ans. Option (D) is correct Explanation: Bio fortification is the process of breeding crops with higher levels of vitamins, minerals, proteins and fat content. This method is employed to improve public health. Single Cell Protein (SCP) is one of the sources of proteins for animal and human nutrition. Somatic hybrids are produced by somatic hybridization. Mutation breeding is one of the methods for making crops disease resistant. CASE STUDY Some strains of Bacillus thuringiensis produce protein that kill certain insects such as lepidopterans (tobacco budworm, armyworm), coleopterans (beetles) and dipterans (flies, mosquitoes). Bacillus thuringiensis forms protein crystals during a particular phase of their growth. These crystals contain a toxic insecticidal protein. The Bt toxin proteins exists as inactive protoxins but once an insect ingests the inactive toxin, it is converted into an active form due to alkaline pH of the gut which solubilise the crystals.

41. The Bt toxin acts on ______ of insect pest.

(A) Hind gut (B) Nervous system (C) Reproductive system (D) Midgut Ans. Option (D) is correct Explanation: The activated Bt toxin binds to the surface of midgut epithelial cells and creates pores that cell swelling and lysis and eventually cause death of the insect. 42. Some strains of Bacillus thuringiensis produce proteins that kill certain insects such as : (A) Grasshoppers and mosquitoes (B) Lepidopterans, coleopterans and dipterans (C) Coleopterans and nematodes (D) Meloidegyne incognitia Ans. Option (B) is correct Explanation: Some strains of Bacillus thuringiensis have proteins that kill insects like coleopterans (beetles), lepidopterans (tobacco budworm, armyworm) and dipterans (flies, mosquitoes, etc). 43. B. thuringiensis forms protein crystals during a particular phase of their growth. These crystals contain a toxic. (A) Proteolytic enzymes (B) Chitin (C) Insecticidal protein (D) Flavoprotein Ans. Option (C) is correct Explanation: Bacillus thuringeinsis forms a toxic protein crystals during a particular phase of their growth. These crystals contain a toxic insecticidal protein. 44. Bt toxin exists as inactive protoxins, it is converted into an active form of toxin due to: (A) Enzymes in insect’s gut. (B) Acidic pH of the insect’s gut. (C) Binding with collagen in insect’s gut (D) Alkaline pH of the insect’s gut.

36

OSWAAL CUET (UG) Sample Question Papers, Biology

Ans. Option (D) is correct Explanation: Bt toxin proteins are present in inactive protoxin form, but become active toxin in the alkaline pH of insect gut. This activated toxin binds to the surface of midgut epithelial cells and creates pores that cause cell swelling and lysis and eventually cause death of the insect. 45. The proteins encoded by the genes cryIAc and cryIIAb control (A) Corn borers (B) Cotton bollworms (C) Mosquitoes (D) Nematodes Ans. Option (B) is correct Explanation: The protein coded by genes cryIAc and cryIIAb control the cotton bollworms. The protein coded by gene cryIAb controls corn borer. PASSAGE The biological wealth of our planet has been declining rapidly and the accusing finger is clearly pointing to human activities. The colonisation of tropical pacific islands by humans is said to have led to extinction of more than 2000 species of native birds. The IUCN red list documents the extinction of 784 species. Since the origin and diversification of life on earth, there were five episodes of mass extinction of species. Current species extinction rate as estimated to be 100-1000 times faster. There are four major causes called “The Evil quartet” Habitat loss, over exploitation, Alien species invasion and co-extinction. Conservation efforts are necessary to preserve biodiversity and protect endangered species and their habitats. 46. Tissue culture and cryopreservation are the techniques used: (A) In in-situ conservation. (B) To control conservation of medicinal plants only. (C) In ex-situ conservation method. (D) Used in conventional breeding programme. Ans. Option (C) is correct Explanation: Tissue culture and cryopreservation are the techniques used in exsitu conservation method. It is the conservation of organisms outside their habitats. 47. Mr. X wants to understand the ‘Evil Quartet’ to know the reasons of Biodiversity loss. It is: (A) Habitat loss and fragmentation only. (B) Over exploitation and Alien species invasions only. (C) Co-extinctions and Alien-species invasion. (D) Habitat loss, over exploitation, Alien species invasions and co-extinctions

Ans. Option (D) is correct Explanation: Habitat loss and fragmentation, exploitation, alien species invasions and coextinctions are four major causes of biodiversity losses, which together are called “The Evil Quarter”. 48. Which one of the following is NOT the reason for decline in biodiversity? (A) Pollution (B) Agriculture (C) Sacred groves (D) Fishing Ans. Option (C) is correct Explanation: Sacred groves are forests which are regenerated around places of worship. Sacred groves help in protection of many rare, threatened and endemic species of plants and animals found in an area. Pollution, agriculture and fishing are the reasons for decline in biodiversity. 49. Select the one which is NOT an ex-situ conservation of biodiversity. (A) Zoological parks (B) Botanical gardens (C) National parks (D) Seed banks Ans. Option (C) is correct Explanation: National parks are in-situ conservation of biodiversity. Zoological parks, botanical gardens and seed banks are ex-situ conservation methods for the protection of species. 50. Identify the in-situ ways to conserve biodiversity from the following A. Zoological parks B. Eggs fertilised in-vitro C. Sacred groves D. Biosphere reserves Choose the correct answer from the options: (A) A, B only (B) B, C only (C) C, D only (D) A, D only Ans. Option (C) is correct. Explanation: Sacred groves and biosphere reserve are in-situ conservation strategies for protection of biodiversity. In situ conservation (on site) is the conservation of genetic resources within natural or human-made ecosystems in which they occur. Zoological parks are ex-situ conservation strategies for biodiversity protection. Ex situ conservation (off site) is the conservation of organisms outside their habitats. When the fertilization of egg is done outside the body, it is called in-vitro fertilization.

❑❑

CUET Question Paper 2021 National Testing Agency

UIQP02 23rd SEP 2021—Slot-2

BIOLOGY Solved

(This includes Questions pertaining to Domain Specific Subject only)

Max. Marks : 80

Time allowed : 30 Min.

General Instructions:

(i) This paper consists of 25 MCQs, attempt any 20 out of 25. (ii) Correct answer or the most appropriate answer: Four marks (+4) . (iii) Any incorrect option marked will be given minus One mark (–1) . (iv) Unanswered/Marked for Review will be given No mark (0) . (v) If more than one option is found to be correct then Four marks (+4) will be awarded to only those who have marked any of the correct options . (vi) If all options are found to be correct then Four marks (+4) will be awarded to all those who have attempted the question . (vii) If none of the options is found correct or a Question is found to be wrong or a Question is dropped then all candidates who have appeared will be given Four marks (+4). (viIi) Calculator / any electronic gadgets are not permitted . 1. Which of the following sequences fits into the organic evolution theory proposed by Charles Darwin and Alfred Wallace? (1) Variations, constancy of population size, overproduction, natural selection (2) Variations, overproduction, constancy of population size, natural selection (3) Overproduction, variations, constancy of population size, natural selection (4) Overproduction, constancy of population size, variations, natural selection Ans. Option (3) is correct. Explanation: The sequence proposed by Darwin and Wallace for organic evolution is overproduction, variations, the constancy of population size, natural selection. First, the species produces an enormous number of offsprings which contain variations. Due to overproduction, competition for survival will occur among the organisms of same or different species in a population. This results in a struggle for existence. This struggle for existence and heritable variations from parents to progeny results in the survival of the fittest or natural selection. The survival of the fittest results in a phenotype picked which is best suited for the environment. But with time even the environment changes, causing the species to change accordingly as well. After many years this changing environment along with the species changing to fit in it, results in the origin of a new species. 2. Which root zone is responsible for maximum water absorption? (1) Root cap (2) Region of elongation (3) Lateral root (4) Root hair Ans. Option (4) is correct. Explanation: Generally, plant absorbs water by the root system. Maximum absorption of water takes place in root hair zone. Root hair are present in the zone of cell maturation. They are unicellular extensions of root epidermis, called epiblema, which absorb water due to increased surface area for absorption. As a result of their presence, the absorptive surface area increases

substantially. Root cap, zone of cell division and zone of elongation generally lack root hairs. 3. Evolution occurs as a result of mutations in (1) somatic DNA (2) somatic RNA (3) germplasm DNA (4) germplasm RNA Ans. Option (3) is correct. Explanation: A mutation refers to the change in DNA of an organism, whereas evolution refers to change in the population over time. Mutation is one of the driving forces of evolution, which is responsible for the variation. Thus, evolution occurs as a result of mutations in Germplasm DNA. 4. Heliophytes are (1) salt-loving plants (3) shade-loving plants Ans. Option (2) is correct.

(2) sun-loving plants (4) water-loving plants

Explanation: Heliophytes are sun loving plants. They are also called sun plants because they grow in open in full sunlight. Prosopis, Betula, Teak, etc., are examples of heliophytes. 5. Which of the following viruses has the capacity to produce lysozymes? (1) Poliomyelitis virus (2) Bacteriophage (3) TMV (4) Vaccinia virus Ans. Option (2) is correct. Explanation: Chemically viruses are nucleoproteins, which are made up of central core of nucleic acid. The nucleic acid (DNA or RNA) represents the genetic characters of virus. TMV has RNA (like most plant viruses have). 10% RNA and 90% protein is present in influenza virus and PSTV (Potato Spindle Tuber Viroid) also has RNA but it does not have capsid (protein coat). Plant viruses contain RNA but cauliflower mosaic virus contain DNA. Bacteriophages contain DNA and almost half animal viruses contain RNA and half contain DNA. But it is said that often animal viruses contain DNA. Cancer causing viruses retrovirus contain both RNA and DNA.

38

OSWAAL CUET (UG) Sample Question Papers, BIOLOGY Some enzymes are detected in viruses such as lysozyme in bacteriophages, transcriptase in vaccinia virus, reverse transcriptase and DNA or RNA polymerase in retroviruses.

6. Silique is the fruit of the family (1) Cucurbitaceae (2) Malvaceae (3) Brassicaceae (4) Leguminosae Ans. Option (3) is correct. Explanation: Silique, which is also known as siliqua. It is a fruit having many seeds. When the length of fruit is less than three times the width of dried fruit it is reffered as silicle. Many species of Brassicaceae, posses siliqua fruit, but in some species silicles may present. 7. The percent crossing over is high when (1) genes are unlinked (2) genes are located on different chromosomes (3) genes are linked and are very close to each other (4) genes are linked and far away from each other Ans. Option (4) is correct. Explanation: Frequency of crossing over will increase if the distance between linked genes is longer. 8. Indole-3-acetic acid (auxin) was first isolated from (1) rice bran oil (2) Rhizopus (3) human urine (4) coleoptiles of oat seedlings Ans. Option (3) is correct. Explanation: Indole-3-acetic acid is the naturally occurring plant hormone of the auxin class. It was first isolated by Kogl and Haagen-Smit from human urine. 9. Sex in honeybee is determined by (1) X : A ratio (2) the presence of Y chromosome (3) environmental temperature (4) haplodiploidy Ans. Option (4) is correct. Explanation: Haplodiploidy or arrhenotoky type of sex determination is used by bees. It is a sex-determination system in which males develop from unfertilized eggs and are haploid, and females develop from fertilised eggs and are diploid. Hapto-diploid sex determination system Parents Female (2n = 32) (Diploid)

Male (n = 16) (Haploid)

Meiosis

Mitosis

n = 16 (Haploid)

without fertilisation (n = 16) (Haploid) Male

n = 16 (Haploid)

n = 16 (Haploid) Fertilisation

2n = 32 (Diploid) Female

10. Which of the following symbiotic microorganisms is nitrogen fixing? (1) Azotobacter (2) Rhizobium (3) Clostridium (4) Cyanobacteria Ans. Option (2) is correct. Explanation: Rhizobium is a symbiotic bacteria found in the root nodules of leguminous plants that has the ability to fix atmospheric Nitrogen. 11. Which one of the following is the correct match of the product and the producer? (1) Cyclosporin A : Trichoderma polysporum (2) Blood cholesterol-increasing statins : Monascus purpureus (3) Antibiotics : Acetobacter aceti (4) Red dye: Methanobacterium Ans. Option (1) is correct. Explanation: Cyclosporine A is produced by Trichoderma polysporum (fungus) and is used as an immunosuppressive agent in organ transplant patients. Statins are produced by Monascus purpureus (yeast). It is used as blood-cholesterol lowering agents. It inhibits the enzymes responsible for synthesis of cholesterol. Acetobacter aceti is a bacterium that produces acetic acid. Methanobacterium is a common methanogen. 12. Which of the following statements is correct with respect to cellular respiration? (1) Process of glycolysis occurs in cell membrane (2) Enzymes of TCA cycle are present in innermitochondrial membrane (3) NAD+ is the final electron donor for anaerobic respiration (4) Conversion of glucose-6-phosphate to fructose-6phosphate is a reversible reaction Ans. Option (4) is correct. Explanation: Glycolysis is a process of breakdown of glucose, which occurs in cytoplasm. In the presence of enzyme phosphoglucose isomerase, the Glucose-6phosphate converts into fructose-6-phosphate. This process is known as isomerisation reaction, which is a reversible process. So, conversion of glucose-6-phosphate to fructose-s-phosphate is a reversible reaction. 13. What is correct for test tube baby? (1) Fertilisation of ova and embryonic development takes place in test tube (2) Fertilisation of ova occurs in uterus while the development takes place in test tube (3) Fertilisation of ova occurs in test tube whereas development of embryo occurs in uterus (4) Unfertilised ova develops in test tube Ans. Option (3) is correct. Explanation: Test tube baby is a term used to refer to a child that is conceived outside the women’s body by a scientific process. The entire process is done in a laboratory. In this method, ova from the wife/donor and sperms from the husband/donor are collected and are fertilised to form zygote under simulated conditions in the laboratory. The fertilised ovum is then placed inside the uterus of a female. 14. Which of the following enzymes helps in fixing CO2 malic acid in Sorghum plant? (1) RuBP carboxylase (2) PEP carboxylase (3) Pentose phosphatase (4) Fructose phosphatase

CUET Board Paper 2021 Ans. Option (2) is correct. Explanation: Sorghum is a C4 plant. In Sorghum plant or C4 plants, PEP Carboxylase fixes CO2 in the mesophyll cells. It forms the 4-carbon compound oxaloacetic acid (OAA), and the other 4-carbon compounds malic acid. These compounds are transported to the bundle sheath cells, where C4 acid breaks down to form C3 acid and CO2. Carbon dioxide enters the C3 cycle. The C3 acid, so formed, is again transported to the mesophyll cells and is regenerated back into PEP. 15. Hormones responsible for menstrual cycle are produced from (1) uterus only (2) ovaries only (3) ovaries and anterior pituitary (4) uterus and anterior pituitary Ans. Option (3) is correct. Explanation: The menstrual cycle is regulated by the coordinated functions of the hypothalamus, pituitary, ovaries, and endometrium. Hypothalamus causes the nearby pituitary gland to produce certain chemicals, which prompt the ovaries to produce the sex hormones oestrogen and progesterone. 16. The vascular bundle where vascular cambium is present between xylem and phloem is called (1) collateral open (2) collateral closed (3) bicollateral (4) exarch Ans. Option (1) is correct. Explanation: When the vascular cambium is present between the xylem and phloem, the vascular bundle is called open collateral vascular bundle whereas when the vascular cambium is not present between the xylem and phloem, the vascular bundle is called closed collateral vascular bundle. The example of open collateral vascular bundle is dicot stem while of closed collateral is monocot stem. Phloem

Phloem

Cambium Xylem

Open Vascular Bundle

Xylem

Closed Vascular Bundle

17. Production of which of the following is stimulated by secretin hormone? (1) Bile juice (2) Pancreatic juice (3) Gastric juice (4) Intestinal juice Ans. Option (2) is correct. Explanation: Secretin hormone has digestive functions. It is produced by S-cells of duodenum and helps in neutralisation of acidic chyme by stimulating pancreas to secrete basic bicarbonate rich pancreatic juice. 18. Lateral conjugation in Spirogyra takes place in the cells of (1) same filament (2) two filaments of same species

39

(3) two filaments of different species (4) All of the above Ans. Option (1) is correct. Explanation: Lateral conjugation is a mode of sexual reproduction found in Spirogyra, which occurs in between the two adjacent cells of the same filament. These adjacent cells function as male and female gametes. The wall of contact between adjacent cells lead to protuberance, which is known as conjugation tube. 19. In which part of mitochondria is the Electron Transport System (ETS) located? (1) Inner membrane (2) Outer membrane (3) Intermembrane space (4) Cristae Ans. Option (1) is correct. Explanation: The metabolic pathway, through which the electron passes from one carrier to another, is called Electron transport system. It is located in the inner mitochondrial membrane. It helps in releasing and utilizing the energy stored in NADH + H+ and FADH2. 20. Diffusion of gases over the respiratory surface occurs because of (1) the pCO2 is more in alveoli than in blood (2) the pCO2 is more in blood than in tissues (3) the pO2 is more in alveoli than in blood (4) the pO2 is more in tissues than in blood Ans. Option (3) is correct. Explanation: The pO2 is high in alveoli and low in the blood of the pulmonary capillaries. So, O2 diffuses across the respiratory membrane from the alveoli into the blood. Also, the pCO2 is high in the pulmonary capillaries and low in the alveolus. So, CO2 diffuses to alveolus. 21. Seed dispersal in squirting cucumber occurs via (1) wind (2) water (3) ballistic mechanism (4) animal consumption Ans. Option (3) is correct. Explanation: Squirting cucumber got this name because of its unusual, rapid plant movement. In this, seed dispersal occurs via ballistic dispersal. In ballistic dispersal, when the fruit get ripened, the fruits pop off the end of their stems and forcibly eject their mucilage covered seeds in an explosive stream. 22. A patient suffering from diabetes mellitus will have (1) ketonuria and glycosuria (2) uremia and renal calculi (3) hyperglycemia (4) uremia and ketonuria Ans. Option (1) is correct. Explanation: Diabetes mellitus is a metabolic disorder in which the body does not produce enough insulin or doesnot respond normally to insulin, causing blood sugar (glucose) levels to be abnormally high. A patient suffering from diabetes mellitus will have ketonuria and glycosuria. Glycosuria is the condition in which glucose is present in urine while ketonuria is the condition in which ketone bodies are present in urine.

40

OSWAAL CUET (UG) Sample Question Papers, BIOLOGY

23. What does low hematocrit value represent? (1) More lymphocytes in blood than what's considered to be healthy (2) Less lymphocytes in blood than what's considered to be healthy (3) More RBCs in blood than what's considered to be healthy (4) Less RBCs in blood than what's considered to be healthy Ans. Option (4) is correct. Explanation: A low value of hematocrit represents the insufficient supply of healthy RBCs (Red blood cells) or few red blood cells. Due to lower than normal hematocrit, the symptoms observed are: Anaemia, fatigue weakness and low energy. 24. Which part of the brain extends to form spinal cord? (1) Medulla (2) Cerebrum

(3) Cerebellum Ans. Option (1) is correct.

(4) Hippocampus

Explanation: Medulla extends to form spinal cord, through a large opening in the bottom of the skull. 25. Which group features 'jointed appendages'? (1) Coelenterata (2) Porifera (3) Aschelminthes (4) Arthropoda Ans. Option (4) is correct. Explanation: The phylum Arthropoda include organisms with jointed appendages and hard exoskeleton. Insects, spiders, scorpions, centipedes, and millipedes on land; Crabs, crayfish, shrimp, lobsters, and barnacles in water are few examples of animals belonging to phylum arthropoda. 

SAMPLE

Question Paper Maximum Marks : 200

1

Time : 45 Minutes

General Instructions: (i) This paper consists of 50 MCQs, attempt any 40 out of 50 . (ii) Correct answer or the most appropriate answer: Five marks (+5) . (iii) Any incorrect option marked will be given minus one mark (– 1) . (iv) Unanswered/Marked for Review will be given no mark (0) . (v) If more than one option is found to be correct then Five marks (+5) will be awarded to only those who have marked any of the correct options . (vi) If all options are found to be correct then Five marks (+5) will be awarded to all those who have attempted the question . (vii) If none of the options is found correct or a Question is found to be wrong or a Question is dropped then all candidates who have appeared will be given five marks (+5). (viii) Calculator / any electronic gadgets are not permitted .

1. Flagellated, motile asexual reproductive structure

are called : (1) Megaspores (3) Zoospores

(2) Aplanospores (4) Microspores.

2. A cross between two tall plants resulted in offspring

having few dwarf plants. What would be the genotypes of both the parents? (1) TT and Tt (2) Tt and Tt (3) TT and TT (4) Tt and tt 3. Opium is obtained from: (1) Papaver somniferum (2) Cannabis sativa (3) Erythroxylum coca (4) Datura metel

4. Which of the following bacteria is not a source of

restriction endonuclease? (1) Haemophilus influenzae (2) Escherichia coli (3) Agrobacterium tumefaciens (4) Bacillius amyloliquefaciens

5. Amensalism is an association between two species

where (1) one species is harmed and other is benefitted. (2) one species is harmed and other is unaffected. (3) one species is benefitted and other is unaffected. (4) both the species are harmed.



6. In an embryo sac, the cells that degenerate after

fertilisation are: (1) Synergids and primary endosperm cell (2) Synergids and antipodals (3) Antipodals and primary endosperm cell

(4) Egg and antipodals.

7. The promoter site and the terminator site for

transcription are located at (1) 3’ (downstream) end and 5’ (upstream) end, respectively of the transcription unit. (2) 5’ (upstream) end and 3’ (downstream) end, respectively of the transcription unit. (3) the 5’ (upstream) end. (4) the 3’ (downstream) end.



Directions : In the following questions a statement of assertion (A) is followed by a statement of reason (R). Mark the correct choice as : (1) Both assertion (A) and reason (R) are true and reason (R) is the correct explanation of assertion (A). (2) Both assertion (A) and reason (R) are true but reason (R) is not the correct explanation of assertion (A). (3) Assertion (A) is true but reason (R) is false. (4) Assertion (A) is false but reason (R) is true.

8. Assertion (A) : Hybrids result from a cross between two genetically unlike parents.



Reason (R) : Hybrid vigour is the superiority of hybrid over either of the parents.

9. A pro-toxin is (1) a primitive toxin (2) a denatured toxin (3) toxin produced by protozoa (4) inactive toxin

2

OSWAAL CUET (UG) Sample Question Papers, BIOLOGY

10. How much of the net primary productivity of

a terrestrial ecosystem is eaten and digested by herbivores? (1) 1% (2) 10% (3) 40% (4) 90%

11. Which one of the following is not a male accessory

gland? (1) Seminal vesicle (2) Ampulla (3) Prostate (4) Bulbo-urethral gland

12. Starting from the innermost part, the correct

sequence of parts in an ovule is, (1) egg, nucellus, embryo sac, integument. (2) egg, embryo sac, nucellus, integument. (3) embryo sac, nucellus, integument, egg. (4) egg, integument, embryo sac, nucellus.

13. Which one of the following is not a nitrogen-fixing

organism? (1) Anabaena (3) Azotobacter

14. An antibiotic resistance gene in a vector usually

helps in the selection of (1) competent cells (3) recombinant cells

(2) Nostoc (4) Pseudomonas

(2) transformed cells (4) None of the above

15 Which one of the following shows maximum

genetic diversity in India ? (1) Rice (2) Maize (3) Mango (4) Groundnut

16 In case of a couple where the male is having a very



low sperm count which technique will be suitable for fertilisation ? (1) Intrauterine transfer (2) Gamete intracytoplasmic fallopian transfer (3) Artificial Insemination



(4) Intracytoplasmic sperm injection

Directions : In the following questions a statement of assertion (A) is followed by a statement of reason (R). Mark the correct choice as : (1) Both assertion (A) and reason (R) are true and reason (R) is the correct explanation of assertion (A). (2) Both assertion (A) and reason (R) are true but reason (R) is not the correct explanation of assertion (A). (3) Assertion (A) is true but reason (R) is false.



(4) Assertion (A) is false but reason (R) is true.

17 Assertion (1) : Sickle cell anaemia is an example of point mutation.



Reason (R) : It is caused by addition or deletion of nitrogenous bases in the DNA or mRNA.



18 The chemical test that is used for diagnosis of

typhoid is (1) ELISA-Test (3) PCR-Test

(2) ESR-Test (4) Widal-Test

19 Stirred-tank bioreactors have been designed for. (1) ensuring anaerobic conditions in culture vessel (2) purification of product (3) addition of preservatives to product (4) availability of oxygen throughout the process 20. Ecotone is (1) a polluted area. (2) the bottom of a lake. (3) a zone of transition between two communities. (4) a zone of developing community.

21. Which of the following statements does not support

the view that elaborates sexual reproductive process appeared much later in the organic evolution. (i) Lower groups of organisms have simpler body design. (ii) Asexual reproduction is common in lower groups. (iii) Asexual reproduction is common in higher groups of organisms. (iv) The high incidence of sexual reproduction in angiosperms and vertebrates.



Choose the correct answer from the options given below. (1) i, ii and iii (2) i, iii and iv (3) i, ii and iv (4) ii, iii and iv 22. ZZ / ZW type of sex determination is seen in (1) Platypus (2) snails. (3) cockroach (4) peacock

23. Which of the following toxic substance is responsible

for the malarial fever. (1) Haemocyanin (3) Haemoglobin

(2) Hemozoin (4) Haemoriden

24. Who among the following was awarded the Nobel

Prize for the development of PCR technique? (1) Herbert Boyer (2) Hargovind Khurana (3) Kary Mullis (4) Arthur Kornberg 25. Ecological niche is (1) the surface area of the ocean. (2) an ecologically adapted zone. (3) the physical position and functional role of a species within the community. (4) formed of all plants and animals living at the bottom of a lake.

26. A multicellular, filamentous alga exhibits a type of sexual life cycle in which the meiotic division occurs after the formation of zygote. The adult filament of this alga has

Sample Question Papers (1) haploid vegetative cells and diploid gametan­ gia. (2) diploid vegetative cells and diploid gametangia. (3) diploid vegetative cells and haploid gametan­ gia. (4) haploid vegetative cells and haploid game­ tangia. 27. A human female with Turner's syndrome. (1) has 45 chromosome with XO (2) has one additional X chromosome (3) exhibits male characters (4) is able to produce children with normal husband

28. A collection of all the alleles of all the genes of a crop



plant is called (1) germplasm collection. (2) protoplasm collection. (3) herbarium. (4) somaclonal collection. 29. ………….. is a first transgenic cow. (1) Dolly (2) Molly (3) Shelly (4) Rosie

30. Which one of the cell in an embryo-sac produce endosperm after double fertilisation? (1) Synergids cell (2) Antipodal cell (3) Central Cell (4) Egg



31. Which one of the cell in an embryo-sac produce

endosperm after double fertilisation? (1) Synergids cell (2) Antipodal cell (3) Central Cell (4) Egg 32. The net electric charge on DNA and histone is (1) both positive (2) both negative (3) negative and positive, respectively (4) zero 33. Big holes in Swiss cheese are made by (1) a machine (2) a bacterium that produces methane gas (3) a bacterium producing a large amount of carbon dioxide (4) a fungus that releases a lot of gases during its metabolic activities 34. ‘Restriction’ in Restriction enzyme refers to (1) cleaving of phosphodiester bond in DNA by the enzyme. (2) cutting of DNA at specific position only. (3) prevention of the multiplication of bacterio­ phage in bacteria. (4) All of the above

35. Which one of the following is not a major

characteristic feature of biodiversity hot-spots? (1) Large number of species

3

(2) Abundance of endemic species (3) Large number of exotic species (4) Mostly located in polar regions

36. The correct surgical procedure as a contraceptive

method is (1) ovariectomy. (3) vasectomy.

(2) hysterectomy. (4) castration.

37. Synthetic drugs structurally related to adrenaline

are (1) Hallucinogens (3) Amphetamines

(2) Analgesics (4) Barbiturates

38. Protoplast is (1) another name for protoplasm. (2) an animal cell. (3) a plant cell without a cell wall. (4) a plant cell.

39. Which of the following is a post-fertilisation event

in flowering plants? (1) Transfer of pollen grains (2) Embryo development (3) Formation of flower (4) Formation of pollen grains

40. From among the situations given below, choose the

one that prevents both autogamy and geitonogamy. (1) Monoecious plant bearing unisexual flowers. (2) Dioecious plant bearing only male or female flowers. (3) Monoecious plant with bisexual flowers. (4) Dioecious plant with bisexual flowers.

41. Which type of immune response is responsible for

the rejection of tissues/organs in the patient's body post transplantation ? (1) auto-immune response (2) humoral immune response (3) physiological immune response (4) cell-mediated immune response

42. A nitrogen-fixing microbe associated with Azolla in rice fields is : (1) Spirulina (3) Frankia

(2) Anabaena (4) Tolypothrix

43. Non-biodegradable pollutants are created by (1) nature (2) excessive use of resources (3) humans (4) natural disasters

44. Which of the following is not a cause for loss of

biodiversity? (1) Destruction of habitat (2) Invasion by alien species (3) Keeping animals in zoological parks (4) Over-exploitation of natural resources

4

OSWAAL CUET (UG) Sample Question Papers, BIOLOGY

45. Pyramid of numbers is (1) always upright. (2) always inverted. (3) either upright or inverted. (4) neither upright nor inverted.





Read the following text and answer the following questions on the basis of the same : Down syndrome (sometimes called Down’s syndrome) is a condition in which a child is born with an extra copy of their 21st chromosome hence its other name, trisomy 21. The affected individual mental retarded, short statured with small round, head, furrowed tongue and partially open mouth, Physical, psychomotor and mental development is retarded.

46. The number of chromosomes a child with Down

syndrome has is (1) 45 (3) 47

47. Down syndrome is (1) Sex-linked (3) dominant

48. One of this trait is seen in a person with Down

syndrome (1) Upward slant eye (3) Short stature

(2) Chromosomal (4) recessive

(2) Baldness (4) Long neck

49. Down Syndrome is an extra copy of which

chromosome (1) 22nd chromosome (3) 45th chromosome

(2) 21st chromosome (4) 47th chromosome

50. Down syndrome is caused due to (1) bacterial infection (2) lack of oxygen supply to the brain during birth (3) Viral infection (4) a chromosomal abnormality

(2) 46 (4) 48

qqq

SAMPLE

Question Paper Maximum Marks : 200

2

Time : 45 Minutes

General Instructions: (i) This paper consists of 50 MCQs, attempt any 40 out of 50 . (ii) Correct answer or the most appropriate answer: Five marks (+5) . (iii) Any incorrect option marked will be given minus one mark (– 1) . (iv) Unanswered/Marked for Review will be given no mark (0) . (v) If more than one option is found to be correct then Five marks (+5) will be awarded to only those who have marked any of the correct options . (vi) If all options are found to be correct then Five marks (+5) will be awarded to all those who have attempted the question . (vii) If none of the options is found correct or a Question is found to be wrong or a Question is dropped then all candidates who have appeared will be given five marks (+5). (viii) Calculator / any electronic gadgets are not permitted .

1. The number of chromosomes in the shoot tip cells

of a maize plant is 20. The number of chromosomes in the microspore mother cells of the same plant shall be (1) 20. (2) 10. (3) 40. (4) 15. 2. Intensely lactating mothers do not generally conceive due to the (1) suppression of gonadotropins. (2) hyper-secretion of gonadotrophins. (3) suppression of gametic transport. (4) suppression of fertilisation. 3. Person having genotype IA IB would show the blood group as AB. This is because of (1) pleiotropy. (2) co-dominance. (3) segregation. (4) incomplete dominance. 4. Diseases are broadly grouped into infectious and non-infectious diseases. In the list given below, identify the infectious diseases. (i) Cancer (ii) Influenza (iii) Allergy (iv) Small pox (1) (i) and (ii) (2) (ii) and (iii) (3) (iii) and (iv) (4) (ii) and (iv) 5. The correct order of step in polymerase chain reaction (PCR) is : (1) Extension, Denaturation, Annealing (2) Denaturation, Annealing, Extension (3) Denaturation, Extension, Annealing (4) Annealing, Extension, Denaturation

6. Animal husbandry and plant breeding programmes

are the examples of (1) reverse evolution. (3) mutation.

(2) artificial selection. (4) natural selection.



7. Which one of the following microbes forms



8. Which of the following statements does not hold



9. The function of copper ions in copper releasing



10. A woman has an X-linked condition on one of her

symbiotic association with plants and help them in their nutrition ? (1) Azotobacter (2) Aspergillus (3) Glomus (4) Trichoderma true for restriction enzyme? (1) It recognises a palindromic nucleotide sequence. (2) It is an endonuclease. (3) It is isolated from viruses. (4) It produces the same kind of sticky ends in different DNA Molecules. IUD's : (1) They suppress sperm motility and fertilising capacity of sperms. (2) They inhibit gametogenesis (3) They make uterus unsuitable for implantation (4) They inhibit ovulation

X-chromosomes. This chromosome can be inherited by (1) only daughters (2) both sons and daughters (3) only grandchildren (4) only sons 11. A person with sickle cell anaemia is (1) more prone to malaria. (2) more prone to typhoid. (3) less prone to malaria. (4) less prone to typhoid.

12. Which of the following has popularised the PCR (polymerase chain reactions)?

6

OSWAAL CUET (UG) Sample Question Papers, BIOLOGY

(1) Easy availability of DNA template (2) Availability of synthetic primers (3) Availability of cheap deoxyribonucleotides (4) Availability of ‘Thermostable’ DNA polymerase

13. Which type of selection is industrial melanism



14. The primary treatment of waste water involves the



15 Which type of restriction endonucleases is used

observed in moth, Biston betularia? (1) Stabilising (2) Directional (3) Disruptive (4) Artificial

removal of (1) dissolved impurities (2) stable particles (3) toxic substances (4) harmful bacteria most in genetic engineering? (1) Type I (2) Type II (3) Type III (4) Type IV

16 Biosphere is (1) a component in the ecosystem. (2) composed of the plants present in the soil. (3) life in the outer space. (4) composed of all living organisms present on earth which interact with the physical environment. 17 Prostate glands are located below (1) gubernaculum (2) seminal vesicles (3) epididymis (4) bulbourethral glands



Directions : In the following questions a statement of assertion (A) is followed by a statement of reason (R). Mark the correct choice as : (1) Both assertion (A) and reason (R) are true and reason (R) is the correct explanation of assertion (A). (2) Both assertion (A) and reason (R) are true but reason (R) is not the correct explanation of assertion (A). (3) Assertion (A) is true but reason (R) is false. (4) Assertion (A) is false but reason (R) is true.

18 Assertion (1) : Termination codons or stop codons



Reason (R) : Stop codons represent termination of translation.

are UAA, UAG and UGA.

19 Which one is correctly matched (1) Cocaine – Opiate narcotic (2) Bhang – Analgesic (3) Reserpine – Tranquilizer (4) Morphine – Hallucinogen 20. α-1 antitrypsin is (1) an antacid. (2) an enzyme (3) used to treat arthritis (4) used to treat emphysema

21. Variations

during mutations of meiotic recombinations are (1) random and directionless. (2) random and directional. (3) random and small. (4) random, small and directional. 22. Which gases are produced in anaerobic sludge digesters ?

(1) Methane and CO2 only (2) Methane, hydrogen sulphide and CO2 (3) Methane, hydrogen sulphide and O2 (4) Hydrogen Sulphide and CO2 23. The polio vaccine was tested on : (1) Mice (2) Monkey (3) Rabbit (4) Dog 24. A population has more young individuals compared to the older individuals. What would be the status of the population after some years? (1) It will decline (2) It will stabilize (3) It will increase (4) It will first decline and then stabilize 25. The membranous cover of the ovum at ovulation is (1) corona radiata. (2) zona radiata. (3) zona pellucida. (4) chorion. 26. To initiate translation, the mRNA first binds to (1) the smaller ribosomal sub-unit. (2) the larger ribosomal sub-unit. (3) the whole ribosome. (4) No such specificity exists. 27. All genes located on the same chromosome (1) form different groups depending upon their relative distance. (2) form one linkage group. (3) will not form any linkage groups. (4) form interactive groups that affect the phenotype. 28. Germplasm storing centre in India is : (1) NEERI (2) FRI (3) CDRI (4) ICRISAT. 29. The site of production of ADA in the body is (1) bone marrow (2) lymphocytes (3) blood plasma (4) monocytes

30. Which one of the following organisms reproduces

sexually only once in its life time? (1) Banana plant (2) Mango (3) Tomato (4) Eucalyptus

31. Fossils are generally found in (1) sedimentary rocks. (2) igneous rocks. (3) metamorphic rocks. (4) any type of rock. 32. Fungicides and antibiotics are chemicals that (1) enhance yield and disease resistance. (2) kill pathogenic fungi and bacteria, respectively. (3) kill all pathogenic microbes. (4) kill pathogenic bacteria and fungi, respectively.

33. What is common to the following plants : Nepenthes,



34. If a genetic disease is transferred from a

Psilotum, Rauwolfia and Aconitum? (1) All are ornamental plants. (2) All are phylogenic link species. (3) All are prone to over-exploitation. (4) All are exclusively present in the eastern Himalayas.

phenotypically normal but carrier female to only some of the male progeny, the disease is (1) autosomal dominant. (2) autosomal recessive.

7

Sample Question Papers



35. Which drug is used as medicine to help patients

42. A nucleoside differs from a nucleotide. It lacks the (1) base (2) sugar (3) phosphate group (4) hydroxyl group



36. The species-area relationship is represented in



44. The genotype of a plant showing the dominant



45. Catalytic converters are fitted into automobiles



Read the following text and answer the following questions on the basis of the same :



Gynoecium, is the female reproductive part of the flower. It may consist of a single or more than one pistil. These pistil may be free or fuse. Each pistils has three parts, stigma, style and ovary. Ovary has an ovarian cavity, which has one or many chambers or locules. The placenta is located inside the ovarian cavity. Megasporangia or ovules arise from the placenta.



46. In which of the following plants the number of

(3) sex-linked dominant. (4) sex-linked recessive. cope with depression and insomnia  ? (1) Morphine (2) Amphetamines (3) Codeine (4) LSD

……………..form on a log scale. (1) linear (2) parabolic (3) straight (4) linear and parabolic



Directions : In the following questions a statement of assertion (A) is followed by a statement of reason (R). Mark the correct choice as : (1) Both assertion (A) and reason (R) are true and reason (R) is the correct explanation of assertion (A). (2) Both assertion (A) and reason (R) are true but reason (R) is not the correct explanation of assertion (A). (3) Assertion (A) is true but reason (R) is false. (4) Assertion (A) is false but reason (R) is true.

37. Assertion (A) : In Griffith’s experiment, the



38. Both deoxyribose and ribose belong to a class of

43. Which of the following is not a producer? (1) Spirogyra (2) Agaricus (3) Volvox (4) Nostoc phenotype can be determined by : (1) test cross (2) dihybird cross (3) pedigree analysis (4) back cross.

to reduce emission of harmful gases. Catalytic converters change unburnt hydrocarbons into (1) carbon dioxide and water. (2) carbon monoxide. (3) methane. (4) carbon dioxide and methane.

dead R strain bacteria was capable of causing the transformation of the live S-strain bacteria. Reason(R) : The S-strain is virulent strain.

sugars called (1) trioses (3) pentoses

(2) hexoses (4) polysaccharides

39. Lysine and tryptophan are (1) proteins. (2) non-essential amino acids. (3) essential amino acids. (4) aromatic and no acids.

40. The historic convention on Biological Diversity held

in Rio de Janeiro in 1992 is known as (1) The CITES Convention (2) The Earth Summit (3) The G-16 Summit (4) The MAB Programme

41. Decomposers like fungi and bacteria are (i) autotrophs. (ii) heterotrophs. (iii) saprotrophs. (iv) chemo-autotrophs.



Choose the correct answer. (1) i and iii (2) i and iv (3) ii and iii (4) i and ii

ovules in an ovary is one ? (1) Mango (3) Water melon

(2) Orchids (4) Papaya

47. A multicarpellary, syncarpous gynoecium is found

in : (1) Papaver (3) Tomato

(2) Brinjal (4) All

48. 82% of ovules found in angiosperms are ............. . (1) Anatropous (2) Amphitropous (3) Orthotropous (4) Circinotropous

49. Which among the following cell is binucleate in an

embryo sac ? (1) Antipodal cell (3) Synergid

(2) Central cell (4) Female gamete

50. Flowers with both androecium and gynoecium are

called : (1) Bisexual flowers (3) Unisexual flowers

(2) Anther (4) Androgynous

qqq

SAMPLE

Question Paper Maximum Marks : 200

3

Time : 45 Minutes

General Instructions: (i) This paper consists of 50 MCQs, attempt any 40 out of 50 . (ii) Correct answer or the most appropriate answer: Five marks (+5) . (iii) Any incorrect option marked will be given minus one mark (– 1) . (iv) Unanswered/Marked for Review will be given no mark (0) . (v) If more than one option is found to be correct then Five marks (+5) will be awarded to only those who have marked any of the correct options . (vi) If all options are found to be correct then Five marks (+5) will be awarded to all those who have attempted the question . (vii) If none of the options is found correct or a Question is found to be wrong or a Question is dropped then all candidates who have appeared will be given five marks (+5). (viii) Calculator / any electronic gadgets are not permitted .

1. Among the terms listed below, those that of are not

technically correct names for a floral whorl are : (i) Androecium (ii) Carpel (iii) Corolla (iv) Sepal (1) (i) and (iv) (2) (iii) and (iv) (3) (ii) and (iv) (4) (i) and (ii) 2. What is the genetic disorder in which an individual has an overall masculine development, Gynaecomastia, and is sterile ? (1) Turner's syndrome (2) Klinefelter's syndrome (3) Down's syndrome (4) Edward syndrome 3. Tobacco consumption is known to stimulate secretion of adrenaline and nor-adrenaline. The component causing this could be (1) nicotine (2) tannic acid (3) curamin (4) catechin 4. Which of the given statement is correct in the context of observing DNA separated by agarose gel electrophoresis? (1) DNA can be seen in visible light. (2) DNA can be seen without staining in visible light. (3) Ethidium bromide stained DNA can be seen in visible light. (4) Ethidium bromide stained DNA can be seen under exposure to UV light. 5. Embryo sac is related to ovule as _______ is related to an anther. (1) Stamen (2) Filament (3) Pollen grain (4) Androecium 6. In XO type of sex determination (1) males produce two different types of gametes (2) females produce two different types of gametes (3) males produce gametes with Y chromosome.

(4) females produce gametes with Y chromosome

7. The chronic intake of ……………. causes Cirrhosis

of liver. (1) opium (3) cocaine

(2) alcohol (4) tobacco

8. Which of the following steps in transcription is

catalysed by RNA polymerase? (1) Initiation (2) Elongation (3) Termination (4) All of the above 9. Pathophysiology is the (1) study of physiology of pathogen. (2) study of normal physiology of host. (3) study of altered physiology of host. (4) none of the above.

10. In some viruses, DNA is synthesised by using RNA



11. Which of the following ions are used by restriction

as template. Such a DNA is called (1) A-DNA (2) B-DNA (3) cDNA (4) rDNA

endonucleases for restriction? (1) Mg²+ ions (2) Mn2+ ions (3) Na+ions (4) K+ions

12. During microsporogenesis, meiosis occurs in: (1) Endothecium (2) Microspore mother cells (3) Microspore tetrads (4) Pollen grains.

13. Which among the following is not an application of



14. Match the scientists listed under column ‘A’ with

PCR ? (1) Gene amplification (2) Diagnosis of pathogens (3) DNA fingerprinting (4) In palaeontology. ideas listed column ‘B’.

Sample Question Papers

(ii) Use and disuse of organs

(1) a single stranded RNA (2) a single stranded DNA (3) either RNA or DNA (4) can be ss-DNA but not ss RNA

C Lamarck

(iii) Continental drift theory



D Wegener

(iv) Evolution by natural selection

Column A



Column B

A Darwin

(i)

B

Oparin

Abiogenesis

A

B

C

D

(1)

(i)

(iv)

(ii)

(iii)

(2)

(iv)

(i)

(ii)

(iii)

(3)

(ii)

(iv)

(iii)

(i)

(4)

(iv)

(iii)

(ii)

(i)

15 What triggers activation of protoxin to active Bt

toxin of Bacillus thuringiensis in bollworm ? (1) Body temperature (2) Moist surface of midgut (3) Alkaline pH of gut (4) Acidic pH of stomach

16 (p+q)2 = p2 + 2pq + q2 = 1 represents an equation

used in (1) population genetics. (2) Mendelian genetics. (3) biometrics. (4) molecular genetics. 17 Bt cotton is not (1) a genetically modified (GM) plant. (2) insect resistant. (3) a bacterial gene expressing system. (4) resistant to all pesticides.

18 In human adult females oxytocin (1) stimulates pituitary to secrete vasopressin (2) causes strong uterine contractions during parturition (3) is secreted by anterior pituitary (4) stimulates growth of mammary glands.

19 What will happen to a well growing herbaceous

24. Lichens are the associations of (1) bacteria and fungus (2) algae and bacterium (3) fungus and algae (4) fungus and virus 25. Acrosomal reaction of the sperm occurs due to (1) its contact with zona pellucida of the ova. (2) reactions within the uterine environment of the female. (3) reactions within the epididymal environment of the male. (4) androgens produced in the uterus. 26. The most important cause of biodiversity loss is : (1) Over exploitation of economic species (2) Habitat loss and fragmentation (3) Invasive species (4) Breakdown of plant-pollinator relationships

29. Sacred groves are specially useful in : (1) preventing soil erosion (2) year-round flow of water in rivers (3) conserving rare and threatened species (4) generating environmental awareness





(2) Cocaine (4) Marijuana

22. A probe which is a molecule used to locate specific

sequences in a mixture of DNA or RNA molecules could be

(2) Sweden (4) South Africa

28. The first genetic material could be (1) protein (2) carbohydrates (3) DNA (4) RNA

20. Analogous organs arise due to (1) divergent evolution. (2) artificial selection. (3) genetic drift. (4) convergent evolution. morphine ? (1) Heroin (3) Tobacco

27. World Summit on Sustainable Development (2002)

was held in : (1) Brazil (3) Argentina



21. Which compound is formed by acetylation of

23. If a population of 50 Paramecium present in a pool

increases to 150 after an hour, what would be the growth rate of population? (1) 50 per hour (2) 200 per hour (3) 5 per hour (4) 100 per hour

plant in the forest if it is transplanted outside the forest in a park? (1) It will grow normally. (2) It will grow well because it is planted in the same locality. (3) It may not survive because of change in its microclimate. (4) It grows very well because the plant gets more sunlight.



9

30. The major source of noise pollution, worldwide is

due to (1) office equipment. (2) transport system. (3) sugar, textile and paper industries. (4) oil refineries and thermal power plants.

31. Gastrula is the embryonic stage in which (1) cleavage occurs (2) blastocoel form (3) germinal layers form (4) villi form.

32. If the sequence of nitrogeneous bases of the

coding strand of DNA in a transcription unit is : 5'-ATGAATG-3', the sequence of bases in its RNA transcript would be (1) 5'-AUGAAUG-3' (2) 5'-UACUUAC-3' (3) 5'-CAUUCAU-3' (4) 5'-GUAAGUA-3' 33. Compressed Natural Gas (CNG) is (1) propane. (2) methane. (3) ethane. (4) butane.



Directions : In the following questions a statement of assertion (A) is followed by a statement of reason (R). Mark the correct choice as :

10

OSWAAL CUET (UG) Sample Question Papers, BIOLOGY

(1) Both assertion (A) and reason (R) are true and reason (R) is the correct explanation of assertion (A). (2) Both assertion (A) and reason (R) are true but reason (R) is not the correct explanation of assertion (A). (3) Assertion (A) is true but reason (R) is false. (4) Assertion (A) is false but reason (R) is true.

34. Assertion (1) : Pathogens are disease causing



35. Assertion (1) : Statin is produced by a yeast called



Reason (R) : It lowers the blood cholesterol level in the body.



36. Seminal plasma, the fluid part of semen, is

organisms. Reason (R) : Salmonella typhi is a pathogenic virus causing tuberculosis. Trichoderma polysporum.

contributed by (i) Seminal vesicle (iii) Urethra (1) i and ii (3) ii, iii and iv

(ii) Prostate (iv) Bulbo-urethral gland (2) i, ii and iv (4) i and iv



37. The process of mineralisation by microorganisms



38. Emergency contraceptives are effective if used



39. If both parents have sickle cell trait, then there



40. The technology of biogas production from cow

helps in the release of (1) inorganic nutrients from humus. (2) both organic and inorganic nutrients from detritus. (3) organic nutrients from humus. (4) inorganic nutrients from detritus and formation of humus. within (1) 72 hrs of coitus. (2) 72 hrs of ovulation. (3) 72 hrs of menstruation. (4) 72 hrs of implantation.

is _______________of the child having sickle cell anaemia. (1) 25 % risk (2) 50 % risk (3) 75% risk (4) No risk dung was developed in India largely due to the efforts of (1) Gas Authority of India (2) Oil and Natural Gas Commission (3) Indian Agricultural Research Institute and Khadi and Village Industries Commission (4) Indian Oil Corporation 41. Choose the incorrect statement from the following. (1) In birds and mammals internal fertilisation takes place. (2) Colostrum contains antibodies and nutrients. (3) Polyspermy is prevented by the chemical changes in the egg surface. (4) In the human female implantation occurs almost seven days after fertilisation.

42. Which of the following type of ecosystem is expected

in an area where evaporation exceeds precipitation, and mean annual rainfall is below 100 mm. (1) Grassland (2) Shrubby forest (3) Desert (4) Mangrove 43. From the sexually transmitted diseases mentioned below, identify the one which does not specifically affect the sex organs. (1) Syphilis (2) AIDS (3) Gonorrhoea (4) Genital warts 44. Substitution of a purine with another purine is known as ............... : (1) Transversion (2) Inversion (3) Transition (4) Translocation 45. Methanogenic bacteria are not found in (1) rumen of cattle (2) gobar gas plant (3) bottom of water-logged paddy fields (4) activated sludge Read the following and answer the question Q.46 to Q.50. given below: The lac operon consists of a regulation gene and three structural gene. The lactose acts as inducer. In the presence of an Inducer such as lactose, the repressor is inactivated during the interaction. This allows RNA polymerase access to the promoter and transcription proceeds. The repressor is synthesized which in turn binds with the operator region of the operon and prevents RNA polymerase from transcribing the operon. 46. When the process of Lac operon is blocked by a repressor it represents : (1) Positive regulation (2) Negative regulation (3) sometimes positive sometimes negative (4) both positive and negative regulation 47. Identify the correct sequence of the structural genes in the lac operon. (1) lacA-lacZ-lacY (2) lacZ-lacA-lacY (3) lacZ-lacY-lacA (4) lacA-lacY-lacZ 48. Which of the following statement is true in reference to the lac operon process in E.coli? (i) Galactosidase is the only enzyme produced in large quantities when lac operon is turned on (ii) The messenger RNA in lac operon is a polycistronic mRNA (1) Only i is correct (2) Only ii is correct (3) Both (i) and (ii) are correct (4) None of them are correct 49. What provides binding site to RNA polymerase? (1) Exons (2) Promoter (3) Inducer (4) Repressor 50. The lac operon of E. coli contains genes involved in lactose metabolism. It's expressed only when lactose is____________(1) and glucose is _________(2). (1) 1: Present, 2: Absent (2) 1: Absent, 2: Present (3) 1: More, 2: less (4) 1: repressed, 2: promoted

qqq

SAMPLE

Question Paper Maximum Marks : 200

4

Time : 45 Minutes

General Instructions: (i) This paper consists of 50 MCQs, attempt any 40 out of 50 . (ii) Correct answer or the most appropriate answer: Five marks (+5) . (iii) Any incorrect option marked will be given minus one mark (– 1) . (iv) Unanswered/Marked for Review will be given no mark (0) . (v) If more than one option is found to be correct then Five marks (+5) will be awarded to only those who have marked any of the correct options . (vi) If all options are found to be correct then Five marks (+5) will be awarded to all those who have attempted the question . (vii) If none of the options is found correct or a Question is found to be wrong or a Question is dropped then all candidates who have appeared will be given five marks (+5). (viii) Calculator / any electronic gadgets are not permitted .

1. A few statements describing certain features of

reproduction are given below. (i) Gametic fusion takes place. (ii) Transfer of genetic material takes place. (iii) Reduction division takes place. (iv) Progeny have some resemblance with parents. Select the options that are true for both asexual and sexual reproduction from the options given below. (1) i and ii (2) ii and iii (3) ii and iv (4) i and iii

2. In a dihybrid cross, if you get 9 : 3 : 3 : 1 ratio it

denotes that (1) the alleles of two genes are interacting with each other. (2) it is a multigenic inheritance. (3) it is a case of multiple allelism. (4) the alleles of two genes are segregating independently.

3. Which of the following should be chosen for best

yield if one were to produce a recombinant protein in large amounts? (1) Laboratory flask of largest capacity (2) A stirred-tank bioreactor without in-lets and out-lets (3) A continuous culture system (4) Any of the above

4. Which of the following would necessarily decrease



5. The term ‘clone’ cannot be applied to offspring

the density of a population in a given habitat? (1) Natality > mortality (2) Immigration > emigration (3) Mortality and emigration (4) Natality and immigration

formed by sexual reproduction because (1) offsprings do not possess exact copies of parental DNA. (2) DNA of only one parent is copied and passed on to the offspring. (3) offsprings are formed at different times. (4) DNA of parent and offspring is completely different.

6. It is said that, Mendel proposed that the



7. An antibiotic resistance gene in a vector usually

factor controlling any character is discrete and independent. This proposition was based on the (1) results of F3 generation of a cross. (2) observations that the offspring of a cross made between the plants having two contrasting characters shows only one character without any blending. (3) self-pollination of F1 off springs. (4) cross pollination of parental generations. helps in the selection of (1) competent cells. (3) recombinant cells.

(2) transformed cells. (4) None of the above

8. In a DNA strand, the nucleotides are linked together by (1) glycosidic bonds. (3) peptide bonds.

(2) phosphodiester bonds. (4) hydrogen bonds.



9. What parameters are used for tiger census in our



10. In a fertilised embryo sac, the haploid, diploid and

country’s national parks and sanctuaries? (1) Pug marks only. (2) Pug marks and faecal pellets. (3) Faecal pellets only. (4) Actual head counts. triploid structures are

12

OSWAAL CUET (UG) Sample Question Papers, BIOLOGY

(1) synergids, zygote and primary endosperm nucleus. (2) synergids, antipodal and polar nuclei. (3) antipodal, synergids and primary endosperm nucleus. (4) synergids, polar nuclei and zygote.

11. In a certain taxon of insects, some have 17

chromosomes and the others have 18 chromosomes. The 17 and 18 chromosome-bearing organisms are (1) males and females, respectively. (2) females and males, respectively. (3) all males. (4) all females.

12. The transfer of genetic material from one bacterium

to another through the mediation of a vector like virus is termed as (1) transduction. (2) conjugation. (3) transformation. (4) translation.

13. An inverted pyramid of biomass can be found in

which ecosystem? (1) Forest (3) Grass land

(2) Marine (4) Tundra

14. The outermost and innermost wall layers of

microsporangium in an anther are respectively, (1) endothecium and tapetum. (2) epidermis and endodermis. (3) epidermis and middle layer. (4) epidermis and tapetum.

15 Climax community is in a state of (1) non-equilibrium. (2) equilibrium. (3) disorder. (4) constant change. 16 With regard to mature mRNA in eukaryotes (1) exons and introns do not appear in the mature RNA. (2) exons appear, but introns do not appear in the mature RNA. (3) introns appear but exons do not appear in the mature RNA. (4) both exons and introns appear in the mature RNA.

17 The most important feature in a plasmid to be used

as a vector is (1) origin of replication (ori). (2) presence of a selectable marker. (3) presence of sites for restriction endonuclease. (4) its size.

18 Which of the following countries has the highest

biodiversity? (1) Brazil (3) Russia

(2) South Africa (4) India

19 The phenomenon observed in some plants wherein parts of the sexual apparatus are used for forming embryos without fertilisation is called (1) parthenocarpy. (2) apomixis. (3) vegetative propagation. (4) sexual reproduction. 20. C-peptide of human insulin is (1) a part of mature insulin molecule.

(2) responsible for formation of disulphide bridges. (3) removed during maturation of pro-insulin to insulin. (4) responsible for its biological activity.

21. Which of the following is not an invasive alien

species in the Indian context? (1) Lantana (2) Cynodon (3) Parthenium (4) Eichhornia

22. Function of filiform apparatus is to [AIPMT 2014] (1) Recognise the suitable pollen at stigma (2) Stimulate division of generative cell (3) Produce nectar (4) Guide the entry of pollen tube

23. While analysing the DNA of an organism, a total

number of 5,386 nucleotides were found out of which the proportion of different bases were : Adenine = 29%, Guanine = 17%, Cytosine = 32% and Thymine = 17%. Considering the Chargaff ’s rule, it can be concluded that (1) it is a double-stranded circular DNA. (2) it is single-stranded DNA. (3) it is a double-stranded linear DNA. (4) no conclusion can be drawn.

24. The vas deferens receives duct from the seminal vesicle and opens into urethra as (1) epididymis. (2) ejaculatory duct. (3) efferent ductule. (4) ureter.

25. Who amongst the following scientists had no

contribution in the development of the double helix model for the structure of DNA? (1) Rosalind Franklin (2) Maurice Wilkins (3) Erwin Chargaff (4) Meselson and Stahl

26. Amongst the animal groups given below, which one has the highest percentage of endangered species? (1) Insects (2) Mammals (3) Amphibians (4) Reptiles

27. Which of the following forests is known as the ‘lungs of the planet Earth’? (1) Taiga forest (2) Tundra forest (3) Amazon rain forest (4) Rain forests of north-east India

28. Identify the odd one from the following. (1) Labia minora (3) Infundibulum

(2) Fimbriae (4) Isthmus

29. In E. coli, the lac operon gets switched on when (1) lactose is present and it binds to the repressor. (2) repressor binds to operator. (3) RNA polymerase binds to the operator. (4) lactose is present and it binds to RNA polymerase.

30. Which among the following is a set of bacterial

diseases ? (1) Cholera, typhoid and mumps (2) Malaria, mumps and poliomyelitis (3) Diphtheria, leprosy and plague (4) Tetanus, Tuberculosis and measles.

Sample Question Papers 31. Golden rice is (1) a variety of rice grown along the yellow river in china. (2) long stored rice having yellow colour tint. (3) a transgenic rice having gene for β-carotene . (4) wild variety of rice with yellow coloured grains.

32. Increased IMR and decreased MMR in a population

will (1) cause rapid increase in growth rate. (2) result in decline in growth rate. (3) not cause significant change in growth rate. (4) result in an explosive population/exp.

33. The human chromosome with the highest and least

number of genes in them are respectively, (1) chromosome 21 and Y. (2) chromosome 1 and X. (3) chromosome 1 and Y. (4) chromosome X and Y.

34. What provides binding site to RNA polymerase? (1) Exons (2) Promoter (3) Inducer (4) Repressor 35. GEAC stands for (1) Genome Engineering Action Committee. (2) Ground Environment Action Committee. (3) Genetic Engineering Approval Committee. (4) Genetic and Environment Approval committee.

36. Following statements are given regarding MTP.

13



39. Assertion (A) : Totipotent cells are found in spores



40. Syphilis is an infectious disease caused by Treponema



(a) Sore or sores at the original site of infection (b) Skin rash, swollen lymph nodes and fever.



41. The most accepted line of descent in human

and zygotes. Reason (R) : Totipotency is the ability of a single cell to divide and produce all of the differentiated cells into an organism. pallidum with 3 stages :

(c) Severe medical problems and can affect the heart brain and other organs of the body. (1) a-I, b-II, c-III (3) a-I, b-III, c-II (2) a-III, b-I, c-II (4) a-II, B-III, c-I evolution is (1) Australopithecus → Ramapithecus → Homo sapiens → Homo habilis. (2) Homo erectus → Homo habilis → Homo sapiens. (3) Ramapithecus → Homo habilis → Homo erectus → Homo sapiens. (4) Australopithecus → Ramapithecus → Homo erectus → Homo habilis → Homo sapiens. 42. Cytidine is a : (1) Nucleoside (2) Nitrogen base (3) Nucleotide (4) Common dinucleotide in DNA and RNA.

Choose the correct options given below. (i) MTPs are generally advised during first trimester. (ii) MTPs are used as a contraceptive method. (iii) MTPs are always surgical. (iv) MTPs require the assistance of qualified medical personnel. (1) ii and iii (2) i and iii (3) i and iv (4) i and ii

43. Biogas is composed of majorly: (1) methane, CO2 and O2 (2) CO2, H2S, and H (3) methane, CO2 (4) H2S, H and O2

37. Viviparity is considered to be more evolved because (1) the young ones are left on their own. (2) the young ones are protected by a thick shell. (3) the young ones are protected inside the mother's body and are looked after they are born leading to more chances of survival. (4) the embryo takes a long time to develop.

45. Choose the incorrect statement. (1) The Montreal protocol is associated with the control of emission of ozone depleting substances. (2) Methane and carbon dioxide are Greenhouse gases. (3) Dobson units are used to measure oxygen content. (4) Use of incinerators is crucial for disposal of hospital wastes.



Directions : In the following questions a statement of assertion (A) is followed by a statement of reason (R). Mark the correct choice as : (1) Both assertion (A) and reason (R) are true and reason (R) is the correct explanation of assertion (A). (2) Both assertion (A) and reason (R) are true but reason (R) is not the correct explanation of assertion (A). (3) Assertion (A) is true but reason (R) is false. (4) Assertion (A) is false but reason (R) is true. 38. Assertion (A) : Opioids, also called opiates used to treat pain. Reason (R) : Opioids are the drugs which bind to specific opioid receptors present in our central nervous system.



44. During DNA replication, the breaking of H-bonds is

performed by : (1) Topoisomerase (3) Helicases



(2) Gyrase (4) None.

Read the following text and answer any four questions on the basis of the same : Ecological Indicators : The presence of dragonflies can reveal changes in the water ecosystems more quickly than studying other animals or plants. In fact, from the nymph to the adult stage, the dragonfly has a significant, positive ecological impact. Dragonfly eggs are laid and hatched in or near water, so their lives show on impact on both water and land ecosystems. Once hatched, dragonfly nymphs can breathe in the air or underwater which enables them to eat mosquito larvae, other aquatic insects and worms,

OSWAAL CUET (UG) Sample Question Papers, BIOLOGY

14

and even small aquatic vertebrates like tadpoles and small fish. Adult dragonflies capture and eat adult mosquitoes.





Community wide mosquito control programs that spray insecticides to kill adult mosquitoes also kill dragonflies.

46. The approach to biological control includes :

(1) Import and release of an insect pest to a new area to provide hosts for natural enemies. (2) Import and release of natural enemies from the native home of an alien insect pest that has invaded a new area. (3) Preservation of natural enemies (predators & parasites) that are already established in an area. (4) Use of insecticides to reduce alien insect pests to establish new equilibrium position.

47. Two diseases less likely to occur in a region with

plenty of dragonflies are_____ (1) Yellow fever and amoebic dysentery (2) Malaria and Yellow fever (3) Anthrax and typhoid (4) Cholera and typhoid

48. Dragonflies indicate positive ecological impact as :

(1) The presence of dragonflies indicates polluted water. (2) Dragonfly nymphs selectively eat mosquito larvae. (3) They help to decrease the probability of diseases spread by vectors. (4) Dragonfly do not cause any harm to beneficial species.

49. The most effective stages in the life cycle of



50. Assertion : Releasing dragonflies in areas where



Reason : Dragonflies are dominant species and will not allow mosquitoes to reproduce. (1) Both assertion and reason are true, and the reason is the correct explanation of the assertion.

dragonfly that eradicates mosquitoes are : (1) Larvae and adult (2) Caterpillar and adult (3) Nymph and adult (4) Pupa and adult



there is an outbreak of malarial diseases can be an environment friendly method of control.

(2) Both assertion and reason are true, but the reason is not the correct explanation of the assertion. (3) Assertion is true but reason is false. (4) Both assertion and reason are false.

qqq

SAMPLE

Question Paper Maximum Marks : 200

5

Time : 45 Minutes

General Instructions: (i) This paper consists of 50 MCQs, attempt any 40 out of 50 . (ii) Correct answer or the most appropriate answer: Five marks (+5) . (iii) Any incorrect option marked will be given minus one mark (– 1) . (iv) Unanswered/Marked for Review will be given no mark (0) . (v) If more than one option is found to be correct then Five marks (+5) will be awarded to only those who have marked any of the correct options . (vi) If all options are found to be correct then Five marks (+5) will be awarded to all those who have attempted the question . (vii) If none of the options is found correct or a Question is found to be wrong or a Question is dropped then all candidates who have appeared will be given five marks (+5). (viii) Calculator / any electronic gadgets are not permitted .

1. Match the items in column I and column II and choose the correct option. Column I

Column II

A

UV

(i)

Bio-magnification

B

Biodegradable (ii) Eutrophication organic matter

C

DDT

(iii) Snow blindness

D

Phosphates

(iv) BOD

The correct matches are (1) A – (ii), B – (i), C – (iv), D – (iii) (2) A – (iii), B – (ii), C – (iv), D – (i) (3) A – (iii), B – (iv), C – (i), D – (ii) (4) A – (iii), B – (i), C – (iv), D – (ii)

(1) SO2 (3) DDT

7. Bacillus thuringiensis is widely used as : (1) Insecticide (2) Weedicides (3) Rodenticide (4) All of the above 8. The RNA polymerase holoenzyme transcribes (1) the promoter, structural gene and the terminator region. (2) the promoter and the terminator gene. (3) the structural gene and the terminator regions. (4) the structural gene only.

molecular or biological scissors because: (1) they cleave base pairs of DNA only at their terminal ends (2) they cleave one or both the strands of DNA (3) they act only on single stranded DNA (4) none of these



3. The free-living fungus Trichoderma can be used for (1) killing insects (2) biological control of plant diseases (3) controlling butterfly caterpillars (4) producing antibiotics

4. The unequivocal proof of DNA as the genetic

material came from studies on a (1) Viroid (2) Bacteriophages (3) Bacterium (4) Fungus

5. Which of the following causes bio-magnification?

6. The cutting of DNA at specific locations became

possible with the discovery of : (1) Ligases (2) Restriction enzymes (3) Probes (4) Selectable markers.

2. Restriction endonucleases are also called as



(2) Mercury (4) Both (2) & (3)

9. Which one of the following is not a major

characteristic feature of biodiversity hot-spots? (1) Large number of species (2) Abundance of endemic species (3) Large number of exotic species (4) Mostly located in polar regions

10. What would happen if oxygen availability to

activated sludge flocs is reduced? (1) It will slow down the rate of degradation of organic matter. (2) The centre of flocs will become anoxic, which would cause death of bacteria and eventually breakage of flocs. (3) Flocs would increase in size as anaerobic bacteria would grow around flocs. (4) Protozoa would grow in large numbers. 11. Which one of the following pairs of codons is correctly matched with their function or the signal for the particular amino acid ?

16

OSWAAL CUET (UG) Sample Question Papers, BIOLOGY

(1) GUU, GCU – Alanine (2) UAG, UGA – stop (3) AUG, ACG – Start/methionine (4) UUA, UCA – Leucine

12. In sickle cell anaemia, glutamic acid is replaced by

valine. Which one of the following triplets codes for valine? (1) G G G (2) A A G (3) G A A (4) G U G

13. Which one of the following is an endangered plant

species of India? (1) Rauwolfia serpentina (2) Santalum album (Sandal wood) (3) Cycas beddonei (4) All of the above

14. _______ is a CNS stimulant as it interferes with the

transport of the neuro-transmitter, dopamine. (1) Valium (2) Barbiturate (3) Cocaine (4) Opium

15 Which one of the following is not a part of

transcription unit in DNA ? (1) The inducer (2) A terminator (3) A promoter (4) The structural gene

16 A particular species of plant produces light, non-



sticky pollen in large numbers and its stigmas are long and feathery. These modifications facilitate pollination by: (1) Insects (2) Water (3) Wind (4) Animals



17 Among the following, where do you think the



18 One of the following is not the causal organism for

process of decomposition would be the fastest? (1) Tropical rain forest (2) Antarctic (2) Dry arid region (4) Alpine region ringworm. (1) Microsporum (3) Epidermophyton

(2) Trichophyton (4) Macrosporum



19 Occasionally, a single gene may express more than



20. The zone at the edge of a lake or ocean which is



21. MALT constitutes about .............. per cent of the



22. Aneuploidy which occurs due to loss of a complete



23. Attractants and rewards are required for :

one effect. The phenomenon is called (1) multiple allelism. (2) mosaicism. (3) pleiotropy. (4) polygeny.

alternatively exposed to air and immersed in water is called (1) pelagic zone. (2) benthic zone (3) lentic one. (4) littoral zone. lymphoid tissue in human body. (1) 50% (2) 20% (3) 70% (4) 10%

[NEET 2017]

homologous pair of chromosomes is  : (1) Trisomy (2) Nullisomy (3) Tetrasomy (4) Euploidy (1) Anemophily (3) Hydrophily

24. One of the following is true with respect to AUG. (1) It codes for methionine only. (2) It is also an initiation codon. (3) It codes for methionine in both prokaryotes and eukaryotes. (4) All of the above

(2) Entomophily (4) Cleistogamy



25. Change in sequence of nucleotide in DNA is known



34. The reservoir for the gaseous type of bio-geo

as ...... . (1) Translation (2) Recombination (3) Mutation (4) Transcription. 26. A protozoan reproduces by binary fission. What will be the number of protozoans in its population after six generations? (1) 128 (2) 24 (3) 64 (4) 32 27. The fact that a purine base always paired through hydrogen bonds with a pyrimidine base leads to, in the DNA double helix (1) the antiparallel nature (2) the semi-conservative nature (3) uniform width throughout DNA (4) uniform length in all DNA 28. 33 percent of India’s (Gross Domestic Product) comes from (1) industry. (2) agriculture. (3) export. (4) small-scale cottage industries. 29. In the embryos of a typical dicot and a grass, true homologous structures are : (1) Coleorhiza and coleoptile (2) Coleoptile and scutellum (3) Cotyledons and scutellum (4) Hypocotyl and radicle. 30. Which of the following is a partial root parasite? (1) Sandal wood (2) Mistletoe (3) Orobanche (4) Ganoderma 31. The biggest constraint of plant breeding is (1) availability of desirable gene in the crop and its wild relatives. (2) infrastructure. (3) trained manpower. (4) transfer of genes from unrelated sources. 32. The immature male germ cells undergo division to produce sperms by the process of spermatogenesis. Choose the correct one with reference to above. (1) Spermatogonia have 46 chromosomes and always undergo meiotic cell division. (2) Primary spermatocytes divide by mitotic cell division. (3) Secondary spermatocytes have 23 chromosomes and undergo second meiotic division. (4) Spermatozoa are transformed into spermatids. 33. Choose the incorrect statement from the following. (1) In birds and mammals internal fertilisation takes place. (2) Colostrum contains antibodies and nutrients. (3) Polyspermy is prevented by the chemical changes in the egg surface. (4) In the human female, implantation occurs almost seven days after fertilisation. chemical cycle exists in

17

Sample Question Papers (1) stratosphere. (3) ionosphere.

(2) atmosphere. (4) lithosphere.

35. Sonalika and Kalyan Sona are varieties of (1) wheat. (2) rice. (3) millet. (4) tobacco.

36. Give one word for the following :



37. Layers of an ovum from outside to inside is

Type of mutation that arise due to change in a single base pair of DNA. (1) Point mutation (2) Somatic mutation (3) Deletion (4) Insertion (1) corona radiata, zona pellucida and vitelline membrane (2) zona pellucida, corona radiata and vitelline membrane (3) vitelline membrane, zona pellucida and corona radiata (4) zona pellucida, vitelline membrane and corona radiata.



43. Assertion (A) : DNA fingerprinting is applied in



44. Assertion (A) : Haemophilia is an autosomal

paternity testing in case of disputes. Reason (R) : It employs the principle of polymorphism in DNA sequences as polymorphisms are inheritable from parent to children. disorder. Reason (R) : A haemophilic father can never pass the gene for haemophilia to his son. 45. The ectoderm cells will form the.......................... . (1) Skeletal system (2) Nervous system (3) Excretory system (4) Respiratory system



Read the following and answer questions from Q.1. to Q.5. given below:



Plants having foreign genes in their genome through genetic engineering are called transgenic plants. Genes can be incorporated either through a vector or through direct introduction of DNA. Bt cotton is a genetically modified organism which is pest resistant. It contains gene cry I Ac and cry II Ab of Bacillus thuringiensis. It is used to control lepidopterans, coleopterans and dipterans. Bt cotton can resist cotton bollworm and produce higher yields. Cry gene produces cry protein or Bt toxin. It is an endotoxin which remains as protoxin in plants and converted to active toxin after getting ingested by the insects. Alkaline pH of the insect gut solubilizes the protein crystals, the activated toxin creates pores in the mid gut wall of the insects which cause them to die.

38. A national level approach to build up a



reproductively healthy society was taken up in our country in (1) 1950s (2) 1960s (3) 1980s (4) 1990s



39. Among the following bio-geochemical cycles which



40. Site of fusion of sperm with an ovum.



41. There are various types of reproduction. The types

one does not have losses due to respiration? (1) Phosphorus (2) Nitrogen (3) Sulphur (4) All of the above (1) Infundibulum (2) Ampullary isthmic junction of oviduct (3) Cervix of uterus (4) Ovary

of reproduction adopted by an organism depend on: (1) the habitat and morphology of the organism. (2) morphology of the organism. (3) morphology and physiology of the organism. (4) the organism’s habitat, physiology and genetic make-up. 42. Micro-propagation is (1) propagation of microbes in vitro. (2) propagation of plants in vitro. (3) propagation of cells in vitro. (4) growing plants on smaller scale.



Directions : In the following questions a statement of assertion (A) is followed by a statement of reason (R). Mark the correct choice as : (1) Both assertion (A) and reason (R) are true and reason (R) is the correct explanation of assertion (A). (2) Both assertion (A) and reason (R) are true but reason (R) is not the correct explanation of assertion (A). (3) Assertion (A) is true but reason (R) is false. (4) Assertion (A) is false but reason (R) is true.



46. What is true about Bt cotton crops?



(1) They are fungal resistant (2) They are insect resistant (3) They are drought resistant (4) All of these

47. Cotton bollworms are killed by the protein encoded

by the gene (1) cry I Ac (3) cry II Ab

(2) cry I Ab (4) cry both (1) and (3)

48. Identify which of the following is not an advantage

of GM crops? (1) GM plants enhance nutritional value of food. (2) GM plants are more tolerant to abiotic stresses. (3) GM plants have helped to reduce post-harvest losses. (4) GM plants can cause gene transfer to non-target plant species. 49. Bacillus thuringiensis is a (1) air borne bacteria. (3) soil borne bacteria

(2) soil borne fungus (4) food borne bacteria

50. Some strains of Bacillus thuringiensis can kill certain

insects such as _______ (1) lepidopterans (3) fruit fly

(2) scorpion (4) dragonfly

qqq

SAMPLE

Question Paper Maximum Marks : 200

6

Time : 45 Minutes

General Instructions: (i) This paper consists of 50 MCQs, attempt any 40 out of 50 . (ii) Correct answer or the most appropriate answer: Five marks (+5) . (iii) Any incorrect option marked will be given minus one mark (– 1) . (iv) Unanswered/Marked for Review will be given no mark (0) . (v) If more than one option is found to be correct then Five marks (+5) will be awarded to only those who have marked any of the correct options . (vi) If all options are found to be correct then Five marks (+5) will be awarded to all those who have attempted the question . (vii) If none of the options is found correct or a Question is found to be wrong or a Question is dropped then all candidates who have appeared will be given five marks (+5). (viii) Calculator / any electronic gadgets are not permitted . 1. Which condition describes the sex correctly ? (1) XO condition as in Turner's syndrome determines the female sex (2) XX sex chromosomes produce male in Drosophila (3) ZZ sex chromosomes determine female sex in birds (4) XO sex chromosomes determine male sex in Grasshopper

2. There is no natural death in single-celled organisms,

like Amoeba and bacteria, because (1) they cannot reproduce sexually. (2) they reproduce by binary fission. (3) parental body is distributed amongst the offsprings. (4) they are microscopic.

3. The term ‘Health’ is defined in many ways. The

most accurate definition of the health would be (1) health is the state of body and mind in a balanced condition. (2) health is the reflection of a smiling face. (3) health is a state of complete physical, mental, and social well-being. (4) health is the symbol of economic prosperity.



4. Which of the following is commonly used as a

vector for introducing a DNA fragment in human lymphocytes? (1) X phage (2) Ti plasmid (3) Retrovirus (4) PBR322

5. The number of chromosomes a child with Down

syndrome has is (1) 45 (3) 47

(2) 46 (4) 48

6. Identify the incorrect statement.

(1) In asexual reproduction, the offsprings produced are morphologically and genetically identical to the parent. (2) Zoospores are sexual reproductive structures. (3) In asexual reproduction, a single parent produces offspring with or without the formation of gametes. (4) Conidia are asexual structures in Penicillium. 7. The disease chikungunya is transmitted by (1) house flies. (2) Aedes mosquitoes. (3) cockroach. (4) female Anopheles. 8. Identify the wrong statement from the following. (1) High levels of oestrogen triggers the ovulatory surge. (2) Oogonial cells start to proliferate and give rise to functional ova in regular cycles from puberty onwards. (3) Sperms released from seminiferous tubules are poorly motile/non-motile. (4) Progesterone level is high during the post-ovulatory phase of menstrual cycle. 9. The site of production of ADA in the body is (1) bone marrow. (2) lymphocytes. (3) blood plasma. (4) monocytes.

10. Aneuploidy which occurs due to loss of a complete



11. Which of the following are the reason(s) for

homologous pair of chromosomes is  : (1) Trisomy (2) Nullisomy (3) Tetrasomy (4) Euploidy

Rheumatoid arthritis? Choose the correct option. (i) Lymphocytes become more active. (ii) Body attacks self-cells. (iii) More antibodies are produced in the body. (iv) The ability to differentiate pathogens or foreign

Sample Question Papers molecules from self-cells is lost. (1) (i) and (ii) (2) (ii) and (iv) (3) (iii) and (iv) (4) (i) and (iii)

12. The first clinical gene therapy was done for the

treatment of (1) AIDS. (2) cancer. (3) cystic fibrosis. (4) SCID (Severe Combined Immuno Deficiency resulting from deficiency of ADA).

13. Theory of “Continuity of Germplasm” was given

by August : (1) G. Mendel (3) Haeckel

(2) Lamarck (4) August Weismann.

14. A typical angiosperm embryo sac at maturity is (1) 8-nucleate and 8-celled (2) 8-nucleate and 7-celled (3) 7-nucleate and 8-celled (4) 7-nucleate and 7-celled 15 Which of the following is not a lymphoid tissue? (1) Spleen (2) Tonsils (3) Appendix (4) Thymus

16 Which group represents minimum species diversity



17 What is the genetic disorder in which an

among vertebrates in India? (1) Birds (2) Mammals (3) Reptiles (4) Amphibians

individual has an overall masculine development, Gynaecomastia, and is sterile ? (1) Turner's syndrome (2) Klinefelter's syndrome (3) Down's syndrome (4) Edward syndrome 18 Which one is not applicable to RNA? (1) Complementary base pairing (2) 5’phosphoryl and 3’ hydroxyl ends (3) Heterocyclic nitrogenous bases (4) Chargaff ’s rule 19 More than 70% of livestock population is in (1) Denmark. (2) India. (3) China. (4) India and China.

20. Directions : In the following questions a statement

of assertion (A) is followed by a statement of reason (R). Mark the correct choice as : (1) Both assertion (A) and reason (R) are true and reason (R) is the correct explanation of assertion (A). (2) Both assertion (A) and reason (R) are true but reason (R) is not the correct explanation of assertion (A). (3) Assertion (A) is true but reason (R) is false. (4) Assertion (A) is false but reason (R) is true.



Assertion (A) : Mutations are non-directional. Reason (R) : Darwin's small variation are directional.

21. Miller's experiment provided evidence for the

theory ? (1) Chemical evolution (2) Abiogensis (3) Biogenesis (4) Special creation.



19

22. Genera like Selaginella and Salvinia produce two

kinds of spores. Such plants are known as (1) Heterosporous (2) Homosorus (3) Heterosorus (4) Homosporous

23. Which of the following forest plants controls the

light conditions at the ground? (1) Climbers (2) Shrubs (3) Tall trees (4) Herbs

24. The fact that a purine base always paired through

hydrogen bonds with a pyrimidine base leads to, in the DNA double helix (1) the antiparallel nature (2) the semi-conservative nature (3) uniform width throughout DNA (4) uniform length in all DNA

25. In some plants, thalamus contributes to fruit

formation. Such fruits are termed as : (1) False fruits (2) Aggregate fruits (3) True fruits (4) Parthenocarpic fruit

26. A bacterial cell was transformed with a recombinant

DNA that was generated using a human gene. However, the transformed cells did not produce the desired protein. Reasons could be: (1) Human gene may have intron which bacteria cannot process. (2) Amino acid codons for humans and bacteria are different. (3) Human protein is formed but degraded by bacteria. (4) All of the above

27. Which of the following has proved helpful in

preserving pollen as fossils? (1) Oil content (2) Cellulosic intine (3) Pollenkitt (4) Sporopollenin

28. Which one of the following is not a fungal disease? (1) Rust of wheat (2) Smut of Bajra (3) Black rot of crucifers (4) Red rot of sugarcane

29. Which of the following is not a feature of the

plasmid? (1) Single stranded (2) Independent replication (3) Circular structure (4) Small, circular double-stranded

30. One of the following is true with respect to AUG. (1) It codes for methionine only. (2) It is also an initiation codon. (3) It codes for methionine in both prokaryotes and eukaryotes. (4) All of the above



31. The phenomenon wherein, the ovary develops into



32. The vitamin whose content increases following the

a fruit without fertilisation is called (1) parthenocarpy. (2) apomixis. (3) asexual reproduction. (4) sexual reproduction.

conversion of milk into curd by lactic acid bacteria is

20

OSWAAL CUET (UG) Sample Question Papers, BIOLOGY

(1) vitamin C. (3) vitamin B12.

(2) vitamin D. (4) vitamin E.

33. Significance of ‘heat shock’ method in bacterial

transformation is to facilitate (1) binding of DNA to the cell wall. (2) uptake of DNA through membrane transport proteins. (3) uptake of DNA through transient pores in the bacterial cell wall. (4) expression of antibiotic resistance gene.

34. Which of the following taxon shows maximum species diversity ? (1) Fishes (3) Orchids

(2) Beetles (4) Ants.

36. In a flower, if the megaspore mother cell forms

megaspores without undergoing meiosis and if one of the megaspores develops into an embryo sac, its nuclei would be (1) haploid. (2) diploid. (3) a few haploids and a few diploids. (4) with varying ploidy. 37. Which of the following is not used as a biopesticide? (1) Xanthomonas campestris (2) Bacillus thuringiensis (3) Trichoderma harzianum (4) Nucleopolyhedrovirus

38. ___________is the first major cause of species

extinction. (1) Co-extinction (3) Habitat destruction

44. Directions : In the following questions a statement

of Assertion (A) is followed by a statement of reason (R). Mark the correct choice as : (1) Both Assertion (A) and reason (R) are true and reason (R) is the correct explanation of Assertion (A). (2) Both Assertion (A) and reason (R) are true but reason (R) is not the correct explanation of Assertion (A). (3) Assertion (A) is true but reason (R) is false. (4) Assertion (A) is false but reason (R) is true.



35. Which of the following steps in transcription is

catalysed by RNA polymerase? (1) Initiation (2) Elongation (3) Termination (4) All of the above

(3) enhancing its resistance to root pathogens. (4) increasing its resistance to insects.

(2) Over exploitation (4) Alien species invasion

39. For the MN-blood group system, the frequencies

of M and N alleles are 0.7 and 0.3, respectively. The expected frequency of MN-blood group bearing organisms is likely to be (1) 42%. (2) 49%. (3) 9%. (4) 58%.

40. Which one of the following products of apiculture is used in cosmetics and polishes? (1) Honey (2) Oil (3) Wax (4) Royal jelly

41. Rising of dough is due to (1) multiplication of yeast. (2) production of CO2. (3) emulsification. (4) hydrolysis of wheat flour starch into sugars.

42. Appearance of antibiotic-resistant bacteria is an example of (1) adaptive radiation. (2) transduction. (3) pre-existing variation in the population. (4) divergent evolution. 43. Mycorrhiza does not help the host plant in (1) enhancing its phosphorus uptake capacity. (2) increasing its tolerance to drought.

Assertion (A) : Sacred groves are highly protected.



Reason (R) : They are of religious importance to the communities.

45. Select the option including all sexually transmitted

diseases. (1) Gonorrhoea, Malaria, Genital herpes (2) AIDS, Malaria, Filaria (3) Cancer, AIDS, Syphilis (4) Gonorrhoea, Syphilis, Genital herpes

46. Read the following text and answer the following questions on the basis of the same:



Growth of a population with time shows specific and predictable patterns. Two types of growth pattern of population are exponential and logistic growth. When resources in the habitat are unlimited each species has the ability to realise fully its innate potential to grow in number. Then the population grows in exponential fashion. When the resources are limited growth curve shows an initial slow rate and then it accelerates and finally slows giving the growth curve which is sigmoid. (1) Exponential growth occurs in organism such as lemmings. (2) Logistic growth is more realistic. (3) Exponential growth has two phases lag and log. (4) In logistic growth, population passes well beyond the carrying capacity of ecosystem.

47. Identify the correct equation that represents the exponential population growth curve: (1) dN/dt = IN (2) dN/dt = rN (K-N/K) (3) Nt =N0ert (4) Both (1) and (3)

48. The equations correctly represents Verhulst-Pearl logistic growth is: (1) dN/dt = rN(K-N)/K (3) dN/dt = N(K-N)/K

(2) dN/dt = rN/K (4) dN/dt = r(K-N)/K

49. The population growth is generally described by the following equation: dN/dt = rN(K-N)/K What does 'r' represent in the given equation? (1) Population density at time Y (2) Intrinsic rate of natural increase (3) Carrying capacity (4) The base of natural logarithm

50. Study the population growth curves (A and B) in the given graph and select the incorrect statement:

Sample Question Papers

21

(1) Curve 'A' shows exponential growth, represented by equation dN/dt=rN (2) Curve 'B' shows logistic growth, represented by equation dN/dt = r(K-N)/K (3) Exponential growth curve is considered as more realistic than the logistic growth curve (4) Curve 'A' can also be represented by equation Nt = N0ert

qqq

SAMPLE

Question Paper Maximum Marks : 200

7

Time : 45 Minutes

General Instructions: (i) This paper consists of 50 MCQs, attempt any 40 out of 50 . (ii) Correct answer or the most appropriate answer: Five marks (+5) . (iii) Any incorrect option marked will be given minus one mark (– 1) . (iv) Unanswered/Marked for Review will be given no mark (0) . (v) If more than one option is found to be correct then Five marks (+5) will be awarded to only those who have marked any of the correct options . (vi) If all options are found to be correct then Five marks (+5) will be awarded to all those who have attempted the question . (vii) If none of the options is found correct or a Question is found to be wrong or a Question is dropped then all candidates who have appeared will be given five marks (+5). (viii) Calculator / any electronic gadgets are not permitted . 1. Snails escape from stressful time in summers by (1) Hibernation (2) Aestivation (3) Diapause (4) Homeostasis

2. Non-specific host defence that exists prior to the

exposure of an antigen is called : (1) acquired immunity (2) passive immunity (3) innate immunity (4) active immunity

3. Viruses grown in the presence of radioactive

phosphorus contained radioactive ________ but not radioactive ________ . (1) DNA, protein (2) Protein, DNA (3) RNA, Nucleoside (4) mRNA, Protein 4. A few statements describing certain features of reproduction are given below. (i) Gametic fusion takes place. (ii) Transfer of genetic material takes place. (iii) Reduction division takes place. (iv) Progeny have some resemblance with parents. Select the options that are true for both asexual and sexual reproduction from the options given below. (1) i and ii (2) ii and iii (3) ii and iv (4) i and iii 5. Biosphere is (1) a component in the ecosystem. (2) composed of the plants present in the soil. (3) life in the outer space. (4) composed of all living organisms present on earth which interact with the physical environment. 6. Marijuana is extracted from (1) dried leaves and flowers of hemp plant (2) ergot fungus (3) roots of hemp plant (4) cocoa plant.

7. DNA replication is : (1) Semi-conservative, continuous (2) Semi-continuous, conservative (3) Semi-conservative, semi-discontinuous (4) Conservative. 8. Which one of the following is correctly matched? (a) Chlamydomonas-Conidia (b) Yeast-Zoospores (c) Onion-Bulb (d) Ginger-Sucker

9. Regulatory proteins are the accessory proteins that



10. A protozoan reproduces by binary fission. What



11. _______ is a CNS stimulant as it interferes with the



12. Choose the correct statement from amongst the

interact with RNA polymerase and affect its role in transcription. Which of the following statements is correct about regulatory protein? (1) They only increase expression. (2) They only decrease expression. (3) They interact with RNA polymerase, but do not affect the expression. (4) They can act both as activators and as repressors. will be the number of protozoans in its population after six generations? (1) 128 (2) 24 (3) 64 (4) 32 transport of the neuro-transmitter, dopamine. (1) Valium (2) Barbiturate (3) Cocaine (4) Opium

following. (1) Dioecious organisms are seen only in animals. (2) Dioecious organisms are seen only in plants.

Sample Question Papers (3) Dioecious organisms are seen in both plants and animals. (4) Dioecious organisms are seen only in vertebrates. 13. Wastewater treatment generates a large quantity of sludge, which can be treated by (1) digesters (2) activated sludge (3) chemicals (4) oxidation pond 14. A dicotyledonous plant bears flowers but never produces fruits and seeds. The most probable cause for the above situation is (1) plant is dioecious and bears only pistillate flowers. (2) plant is dioecious and bears both pistillate and staminate flowers. (3) plant is monoecious. (4) plant is dioecious and bears only staminate flowers. 15 Select the statement which explains best parasitism. (1) One organism is benefited (2) Both the organisms are benefited (3) One organism is benefited, other is not affected (4) One organism is benefited, other is affected 16 Autogamy can occur in a chasmogamous flower if (1) pollen matures before maturity of ovule. (2) ovules mature before maturity of pollen. (3) Both pollen and ovules mature simultaneously. (4) Both anther and stigma are of equal lengths. 17 PCR technique is best for : (1) DNA synthesis (2) Protein amplification (3) DNA amplification (4) DNA ligation. Directions : In the following questions a statement of assertion (1) is followed by a statement of reason (R). Mark the correct choice as : (1) Both assertion (A) and reason (R) are true and reason (R) is the correct explanation of assertion (A). (2) Both assertion (A) and reason (R) are true but reason (R) is not the correct explanation of assertion (A). (3) Assertion (A) is true but reason (R) is false. (4) Assertion (A) is false but reason (R) is true. 18 Assertion (A) : Bone marrow and thymus are primary lymphoid organs. Reason (R) : It is the organs, to which matured lymphocytes migrate, interact with antigens and then proliferate to become effector cells. 19 Assertion (A) : Bottled fruit juices bought from the market are clearer as compared to those made at home. Reason (R) : They are clarified by pectinases and proteases. 20. Where among the following will you find pitcher plant? (1) Rain forest of North-East India (2) Sundarbans (3) Thar desert (4) Western ghats

21. Which of the following are the functions of RNA?

(1) It is a carrier of genetic information from DNA to ribosomes synthesising polypeptides.



23

(2) It carries amino acids to ribosomes. (3) It is a constituent component of ribosomes. (4) All of the above

22. The ‘Origin of Species’ was written by (1) Oparin. (2) Weismann. (3) Lamarck. (4) Darwin. 23. In majority of angiosperms (1) there are numerous antipodal cells (2) reduction division occurs in the megaspore mother cells (3) a small central cell is present in the embryo sac (4) egg has a filiform apparatus

24. Which of the following countries has the highest

biodiversity? (1) Brazil (3) Russia

(2) South Africa (4) India

25. Which of the following pairs is wrongly matched. (1) Starch synthesis in pea : Multiple alleles (2) ABO blood grouping : Co-dominance (3) Flower colour in Snapdragon : Incomplete dominance (4) T.H. Morgan : Linkage



26. Spot the odd one out from the following structures

with reference to the male reproductive system. (1) Rete testis (2) Epididymis (3) Vasa efferentia (4) Isthmus 27. Mendel’s Law of independent assortment holds good for genes situated on the (1) non-homologous chromosomes. (2) homologous chromosomes. (3) extra nuclear genetic element. (4) same chromosome. 28. What is common to the techniques (i) in-vitro fertilisation, (ii) Cryopreservation and (iii) tissue culture? (1) All are in-situ conservation methods. (2) All are ex-situ conservation methods. (3) All require ultra-modern equipment and large space. (4) All are methods of conservation of extinct organisms. 29. Variations during mutations of meiotic recombinations are (1) random and directionless (2) random and directional (3) random and small (4) random, small and directional

30. Match the assistant reproductive technique given in column A with their description given in column B. S. No. 1.

Column A GIFT

Column B (i)

Both eggs and sperm are transferred into the Fallopian tubes in a procedure called laparoscopy.

OSWAAL CUET (UG) Sample Question Papers, BIOLOGY

24 2.

ZIFT

(ii)

It is used when blockage in uterine tube prevents the normal binding of sperm to egg.

3.

ICSI

(iii) A single sperm is injected directly into each egg to carry out fertilisation.

4.

IUI

(iv) It involves placing sperm inside a woman’s uterus to facilitate fertilisation.

(1) a – (i), b – (ii), c – (iii), d – (iv) (2) a – (ii), b – (iii), c – (i), d – (iv) (3) a – (iii), b – (ii), c – (iv), d – (i) (4) a – (iv), b – (i), c – (ii), d – (iii)

31. Approximately how much of the solar energy that

falls on the leaves of a plant is converted to chemical energy by photosynthesis? (1) Less than 1% (2) 2–10% (3) 30% (4) 50%

(1) Enzyme P-Exonuclease and Enzyme QPermease (2) Enzyme P- Exonuclease and Enzyme Q- Ligase (3) Enzyme P-Endonuclease and Enzyme QPermease (4) Enzyme P-Restriction endonuclease and Enzyme Q- Ligase

32. F2 generation in a Mendelian cross showed that



33. The theory of spontaneous generation stated that (1) life arose from living forms only. (2) life can arise from both living and non-living. (3) life can arise from non-living things only. (4) life arises spontaneously, neither from living nor from the non-living.





both genotypic and phenotypic ratios are same as 1 : 2 : 1. It represent a case of : (1) Co-dominance (2) Dihybrid cross (3) Monohybrid cross with incomplete dominance (4) Monohybrid cross with complete dominance



34. Mutation theory on organic evolution was given by

Hugo de Vries while working on (1) Althea rosea. (2) Pisum sativum. (3) Drosophila melanogaster. (4) Oenothera lamarckiana.

35. Which of the following represent maximum number

of species among global biodiversity? (1) Mosses and ferns (2) Algae (3) Lichens (4) Fungi 36. Urethral meatus refers to the (1) urinogenital duct. (2) opening of vas deferens into urethra. (3) external opening of the urinogenital duct. (4) muscles surrounding the urinogenital duct.

37. Name the enzymes ‘P’ and ‘Q’ that are involved in the processes given below

38. Which of the following is the correct group of

vestigial organs in man? (1) Nictitating membrane, ear muscles, eyelids and coccyx. (2) Appendix, coccyx, ear muscles and elbow joint. (3) Wisdom tooth, coccyx, body hair and ear muscles. (4) Wisdom tooth, body hairs, nictitating membrane and vermiform appendix.

39. The one-horned rhinoceros is specific to which of

the following sanctuary? (1) Bhitar Kanika (2) Bandipur (3) Kaziranga (4) Corbett park 40. Somaclones are obtained by : (1) Tissue culture (2) Genetic Engineering (3) Plant breeding (4) Irradiation.

41. Which one of the following changes involved is not

correct regarding the evolution of man? (1) Absence of tail (2) Perfection of hand for making tool (3) Change of diet from hard nuts and hard roots to soft food (4) Increase in the ability to communicate with others and develop community behaviour 42. Golden rice is (1) a variety of rice grown along the yellow river in China. (2) long stored rice having yellow colour tint. (3) a transgenic rice having gene for β-carotene . (4) wild variety of rice with yellow coloured grains.

Sample Question Papers

43. In vitro fertilisation is a technique that involves

transfer of which one of the following into the fallopian tube? (1) Embryo only up to 8 celled stage (2) Either zygote or early embryo up to 8 celled stage (3) Embryo of 32 celled stage (4) Zygote only

45. Directions : In the following questions a statement

of assertion (A) is followed by a statement of reason (R). Mark the correct choice as : (1) Both assertion (A) and reason (R) are true and reason (R) is the correct explanation of assertion (A). (2) Both assertion (A) and reason (R) are true but reason (R) is not the correct explanation of assertion (A). (3) Assertion (A) is true but reason (R) is false. (4) Assertion (A) is false but reason (R) is true.



Assertion (A) : Members of the genus Glomus form mycorrhiza.



Reason (R) : Mycorrhiza is an association of plants and fungi.

46. Assertion (A) : Blue green algae are popular as biofertilisers.



Reason (R) : Blue green algae cause algal bloom in polluted water bodies.



Read the following text and answer the following questions on the basis of the same:



Transgenic animals have their DNA manipulated to possess and express an extra gene. The genome of these animals has been changed and they can carry genes from other species. The first transgenic sheep was Dolly. The reason behind producing these transgenic animals are for specific economic trait, some are produced as disease models (animals genetically manipulated to exhibit disease symptoms so that effective treatment can be studied).

44. Weismann cut off tails of mice generation after

generation but tails neither disappeared nor shortened. According to this, which of the following statement is correct? (1) Darwin was correct. (2) Tail is an essential organ. (3) Mutation theory is wrong. (4) Lamarckism was wrong in inheritance of acquired characters.



25

47. ___ were the first mammals to be transgenic animals. (1) Sheep (2) Mice (3) Fruit flies (4) None of these 48. 95% transgenic animal are : (1) Mice (2) Cat (3) Dog (4) Sheep

49. 1st cloned animal is :



50. The 1st transgenic cow is :

(1) Rat (3) Pig (1) Mosie (3) Dosie

(2) Rabbit (4) Sheep. (2) Rosie (4) Cowsie.

qqq

SAMPLE

Question Paper Maximum Marks : 200

8

Time : 45 Minutes

General Instructions: (i) This paper consists of 50 MCQs, attempt any 40 out of 50 . (ii) Correct answer or the most appropriate answer: Five marks (+5) . (iii) Any incorrect option marked will be given minus one mark (– 1) . (iv) Unanswered/Marked for Review will be given no mark (0) . (v) If more than one option is found to be correct then Five marks (+5) will be awarded to only those who have marked any of the correct options . (vi) If all options are found to be correct then Five marks (+5) will be awarded to all those who have attempted the question . (vii) If none of the options is found correct or a Question is found to be wrong or a Question is dropped then all candidates who have appeared will be given five marks (+5). (viii) Calculator / any electronic gadgets are not permitted .

1. Following statements are given regarding MTP.

Choose the correct options given below. (i) MTPs are generally advised during first trimester. (ii) MTPs are used as a contraceptive method. (iii) MTPs are always surgical. (iv) MTPs require the assistance of qualified medical personnel. (1) ii and iii (2) i and iii (3) i and iv (4) i and ii

2. Occasionally, a single gene may express more than

one effect. The phenomenon is called (1) multiple allelism. (2) mosaicism. (3) pleiotropy. (4) polygeny.

3. The organisms which cause diseases in plants and

animals are called (1) pathogens. (3) insects.

(2) vectors. (4) worms.

4. The role of DNA ligase in the construction of a

recombinant DNA molecule is (1) formation of phosphodiester bond between two DNA fragments. (2) formation of hydrogen bonds between sticky ends of DNA fragments. (3) ligation of all purine and pyrimidine bases. (4) None of the above

5. Increased IMR and decreased MMR in a population

will (1) cause rapid increase in growth rate. (2) result in decline in growth rate. (3) not cause significant change in growth rate. (4) result in an explosive population/exp.



6. In a certain taxon of insects some have 17

chromosomes and the others have 18 chromosomes. The 17 and 18 chromosome-bearing organisms are (1) males and females, respectively. (2) females and males, respectively. (3) all males. (4) all females. 7. A person with sickle cell anaemia is (1) more prone to malaria. (2) more prone to typhoid. (3) less prone to malaria. (4) less prone to typhoid. 8. Bt cotton is not (1) a genetically modified (GM) plant. (2) insect resistant. (3) a bacterial gene expressing system. (4) resistant to all pesticides. 9. The extinction of passenger pigeon was due to (1) increased number of predatory birds. (2) over-exploitation by humans. (3) non-availability of the food. (4) bird flu virus infection. 10. Sterilisation techniques are generally foolproof methods of contraception with least side effects. Yet, this is the last option for the couples because : (i) It is almost irreversible (ii) Of the misconception that it will reduce sexual urge/drive (iii) It is a surgical procedure (iv) Of lack of sufficient facilities in many parts of the country Choose the correct option : (1) i and iii (2) ii and iii (3) ii and iv (4) i, ii, iii and iv

Sample Question Papers 11. With regard to mature mRNA in eukaryotes (1) exons and introns do not appear in the mature RNA. (2) exons appear, but introns do not appear in the mature RNA. (3) introns appear but exons do not appear in the mature RNA. (4) both exons and introns appear in the mature RNA.

12. The substance produced by a cell in viral infection

that can protect other cells from further infection is (1) serotonin. (2) colostrum. (3) interferon. (4) histamine.

13. Choose the correct option regarding Retrovirus (1) an RNA virus that can synthesise DNA during infection. (2) a DNA virus that can synthesise RNA during infection. (3) a ss DNA virus. (4) a dsRNA virus. 14. Biodiversity is affected by (1) latitudinal gradients (2) species-area relationship (3) Both (1) and (2) (4) None of the above

27

19 Morula is a developmental stage (1) between the zygote and blastocyst. (2) between the blastocyst and gastrula. (3) after the implantation. (4) between implantation and parturition.

20. Who amongst the following scientists had no



21. Inbreeding is carried out in animal husbandry



22. Which of the following material takes the longest

contribution in the development of the double helix model for the structure of DNA? (1) Rosalind Franklin (2) Maurice Wilkins (3) Erwin Chargaff (4) Meselson and Stahl because it (1) increases vigour. (2) improves the breed. (3) increases heterozygosity. (4) increases homozygosity.

time for biodegradation? (1) Cotton (2) Paper (3) Bone (4) Jute

23. Which of the following hormones is not secreted



by human placenta? (1) hCG (3) Progesterone

(2) Estrogens (4) LH



24. The human chromosome with the highest and least

coding strand of DNA in a transcription unit is : 5′-ATGAATG-3′, the sequence of bases in its RNA transcript would be (1) 5′-AUGAAUG-3′. (2) 5′-UACUUAC-3′. (3) 5′-CAUUCAU-3′. (4) 5′-GUAAGUA-3′.



26. Among the following edible fishes, which one is

17 Antibodies present in colostrum which protect the



27. The given figure is the diagrammatic representation

15 Acrosomal reaction of the sperm occurs due to (1) its contact with zona pellucida of the ova. (2) reactions within the uterine environment of the female. (3) reactions within the epididymal environment of the male. (4) androgens produced in the uterus.



16 If the sequence of nitrogeneous bases of the

new born from certain diseases is of (1) Ig G type. (2) Ig A type. (3) Ig D type. (4) Ig E type.

number of genes in them are respectively, (1) chromosome 21 and Y. (2) chromosome 1 and X. (3) chromosome 1 and Y. (4) chromosome X and Y. 25. Which one of the following is a marine fish? (1) Rohu (2) Hilsa (3) Catla (4) Common Carp

a marine fish having rich source of omega-3-fatty acids? (1) Mystus (2) Mangur (3) Mrigala (4) Mackerel of E.coli vector pBR322. EcoR I

Pvu I Pst I

18 Match correctly the following and choose the correct option.

Column I Environment Protection Act

(i)

1974

B

Air Prevention and Control of Pollution

(ii)

1987

C

Water Act

(iii)

1986

Amendment of Air Act (iv) to include noise The correct matches are (1) A – (iii), B – (iv), C – (i), D – (ii) (2) A – (i), B – (iii), C – (ii), D – (iv) (3) A – (iv), B – (i), C – (ii), D – (iii) (4) A – (iii), B – (iv), C – (ii), D – (i)

1981

Hind III BamH I

ampR

Column II

A

D

Cla I

tetR

pBR322 ori

Sal I

rop Pvu II



Which one of the given options correctly identifies its certain component(s)? (1) Ori – original restriction enzyme (2) Rop – Reduced osmotic pressure (3) Hind III, EcoRI – selectable markers (4) AmpR, tetR – antibiotic resistance genes

28

OSWAAL CUET (UG) Sample Question Papers, BIOLOGY

28. The phenomenon observed in some plants wherein



37. Among the following sets of examples for divergent



38. Which one of the following alcoholic drinks is

parts of the sexual apparatus are used for forming embryos without fertilisation is called (1) parthenocarpy. (2) apomixis. (3) vegetative propagation. (4) sexual reproduction.

evolution, select the incorrect option : (1) Brain of bat, man and cheetah (2) Heart of bat, man and cheetah (3) Forelimbs of man, bat and cheetah (4) Eye of octopus, bat and cheetah.

29. Which of the following evidences does not favour

produced without distillation? (1) Wine (2) Whisky (3) Rum (4) Brandy



the Lamarckian concept of inheritance of acquired characters? (1) Absence of limbs in snakes (2) Melanisation in peppered moth (3) Presence of webbed toes in aquatic birds (4) Lack of pigment in cave-dwelling animals 30. Artificial insemination means (1) Transfer of sperms of a healthy donor to a test tube containing ova (2) Transfer of sperms of husband to a test tube containing ova (3) Artificial introduction of sperms of a healthy donor into the vagina (4) Introduction of sperms of healthy donor directly into the ovary 31. In agarose gel electrophoresis, DNA molecules are separated on the basis of their (1) charge only. (2) size only. (3) charge to size ratio. (4) All of the above

32. In a typical complete, bisexual and hypogynous

flower the arrangement of floral whorls on the thalamus from the outermost to the innermost is (1) calyx, corolla, androecium and gynoecium. (2) calyx, corolla, gynoecium and androecium. (3) gynoecium, androecium, corolla and calyx. (4) androecium, gynoecium, corolla and calyx.

33. Which of the following is used as an atmospheric

pollution indicator? (1) Lepidoptera (2) Lichens (3) Lycopersicon (4) Lycopodium 34. Analogous structures are a result of : (1) Convergent evolution(2) Shared ancestry (3) Stabilizing selection (4) Divergent evolution

35. There is a restriction endonuclease called EcoRI.

What does ‘co’ part in it stand for? (1) Colon (2) Coelom (3) Coenzyme (4) Coli

36. Given below are a few statements related to external



fertilisation. (i) The male and female gametes are formed and released simultaneously. (ii) Only a few gametes are released into the medium. (iii) Water is the medium in a majority of organisms exhibiting external fertilisation. (iv) Offsprings formed as a result of external fertilisation have better chances of survival than those formed inside an organism.

Choose the correct statements from the given options. (1) (iii) and (iv) (2) (i) and (iii) (3) (ii) and (iv) (4) (i) and (iv)

39. Control of gene expression takes place at the level of (1) DNA-replication. (2) Transcription. (3) Translation. (4) None of the above

40. Cryopreservation of gametes of threatened species in viable and fertile condition can be referred to as : (1) In-situ conservation of biodiversity (2) advanced ex-situ conservation of biodiversity. (3) In-situ conservation by sacred groves. (4) In-situ cryo-conservation of biodiversity 41. Which one of the following is correctly matched? (1) Chlamydomonas-Conidia (2) Yeast-Zoospores (3) Onion-Bulb (4) Ginger-Sucker 42. Palaeontological evidences for evolution refer to the (1) development of embryo. (2) homologous organs. (3) fossils. (4) analogous organs.

43. BOD of waste water is estimated by measuring the amount of (1) total organic matter. (2) biodegradable organic matter. (3) oxygen evolution. (4) oxygen consumption.

44. Which of the following steps are catalysed by Taq



45. Evolution of life shows that life forms had a trend of



Read the following text and answer the following questions on the basis of the same:



All biomes of the earth constitute the earth's ecosystem. All flora and fauna occur in the lithosphere, atmosphere and hydrosphere along with their environments. Air, water, soil, biota, temperature and light are the major environmental factors that directly affect the flora and fauna in different geological areas of the earth.

polymerase in a PCR reaction? (1) Denaturation of template DNA (2) Annealing of primers to template DNA (3) Extension of primer end on the template DNA (4) All of the above moving from (1) land to water. (2) dry land to wet land. (3) fresh water to sea water. (4) water to land.

46. Climate of an area depends upon : (1) Altitude of the area (2) Latitude of the area (3) Topography (4) Both (1) and (2).

Sample Question Papers 47. Hydrological cycle is concerned with : (1) Lithosphere (2) Hydrosphere (3) Atmosphere (4) All of these. 48. Soil aeration is inversely proportional to : (1) Soil pore spaces (2) Diameter of soil particles (3) Water holding capacity of soil (4) None of the above. 49. Edaphology is concerned with study of : (1) Soil (2) Climate (3) Biota (4) Topography



Directions : In the following questions a statement of Assertion (A) is followed by a statement of reason

29

(R). Mark the correct choice as : (1) Both Assertion (A) and reason (R) are true and reason (R) is the correct explanation of Assertion (A). (2) Both Assertion (A) and reason (R) are true but reason (R) is not the correct explanation of Assertion (A). (3) Assertion (A) is true but reason (R) is false. (4) Assertion (A) is false but reason (R) is true.

50. Assertion (A) : Some organisms like Salmon are

known to be Euryhaline. Reason (R) : Euryhalines can tolerate a wide range of salinities.

qqq

SAMPLE

Question Paper Maximum Marks : 200

9

Time : 45 Minutes

General Instructions: (i) This paper consists of 50 MCQs, attempt any 40 out of 50 . (ii) Correct answer or the most appropriate answer: Five marks (+5) . (iii) Any incorrect option marked will be given minus one mark (– 1) . (iv) Unanswered/Marked for Review will be given no mark (0) . (v) If more than one option is found to be correct then Five marks (+5) will be awarded to only those who have marked any of the correct options . (vi) If all options are found to be correct then Five marks (+5) will be awarded to all those who have attempted the question . (vii) If none of the options is found correct or a Question is found to be wrong or a Question is dropped then all candidates who have appeared will be given five marks (+5). (viii) Calculator / any electronic gadgets are not permitted .

1. The number of chromosomes in the shoot tip cells

of a maize plant is 20. The number of chromosomes in the microspore mother cells of the same plant shall be (1) 20. (2) 10. (3) 40. (4) 15. 2. ZZ / ZW type of sex determination is seen in (1) Platypus. (2) snails. (3) cockroach. (4) peacock.

3. AIDS is caused by HIV. Among the following, which



4. According to Allen's rule, the mammals from colder

one is not a mode of transmission of HIV? (1) Transfusion of contaminated blood. (2) Sharing the infected needles. (3) Shaking hands with infected persons. (4) Sexual contact with infected persons. climates have (1) shorter ears and longer limbs (2) longer ears and shorter limbs (3) longer ears and longer limbs (4) shorter ears and shorter limbs





(1) (ii) and (iv) (3) (iii) and (iv)

Choose the correct answer from the options given below.

6. The inheritance pattern of a gene over generations

among humans is studied by the pedigree analysis. Character studied in the pedigree analysis is equivalent to (1) quantitative trait. (2) Mendelian trait. (3) polygenic trait. (4) maternal trait. Directions : In the following questions a statement of assertion (A) is followed by a statement of reason (R). Mark the correct choice as : (1) Both assertion (A) and reason (R) are true and reason (R) is the correct explanation of assertion (A). (2) Both assertion (A) and reason (R) are true but reason (R) is not the correct explanation of assertion (A). (3) Assertion (A) is true but reason (R) is false. (4) Assertion (A) is false but reason (R) is true.

7. Assertion (A) : Ti plasmid (tumour-inducing

plasmid) of Agrobacterium tumefaciens is used as a cloning vector. Reason (R) : Ti plasmid integrates a segment of its DNA, termed T-DNA into the chromosomal DNA of its host plant cells.

5. Which of the following situations correctly describe

the similarity between an angiosperm egg and a human egg? (i) Eggs of both are formed only once in a lifetime. (ii) Both the angiosperm egg and human egg are stationary. (iii) Both the angiosperm egg and human egg are motile transported. (iv) Syngamy in both results in the formation of zygote.

(2) (iv) only (4) (i) and (iv)



8. A group of animals, which are related by descent

and share many similarities, are referred to as (1) breed. (2) race. (3) variety. (4) species.

9. A multicellular, filamentous alga exhibits a type of

sexual life cycle in which the meiotic division occurs after the formation of zygote. The adult filament of this alga has

Sample Question Papers

31

(1) haploid vegetative cells and diploid gametan­ gia. (2) diploid vegetative cells and diploid gametangia. (3) diploid vegetative cells and haploid gametangia. (4) haploid vegetative cells and haploid game­ tangia.



18. The outermost and innermost wall layers of

10. Which one of the following cannot be explained on



19. In a DNA strand, the nucleotides are linked



the basis of Mendel's law of dominance? (1) Alleles do not show any blending and both the characters recover as such in F2 generation (2) Factors occur in pairs (3) The discrete unit controlling a particular character is called a factor (4) Out of one pair of factors, one is dominant and the other is recessive

together by (1) glycosidic bonds. (3) peptide bonds.

12. To isolate protoplast, one needs (1) pectinase. (2) cellulase. (3) both pectinase and cellulase. (4) chitinase. 13. Autecology is the (1) relation of a population to its environment. (2) relation of an individual to its environment. (3) relation of a community to its environment. (4) relation of a biome to its environment. 14. Embryo sac is to ovule as _______ is to an anther. (1) stamen (2) filament (3) pollen grain (4) androecium

15. Conditions of a karyotype 2n ± 1 and 2n ± 2 are

called (1) aneuploidy. (3) allopolyploidy.

(2) polyploidy. (4) monosomy.

16. The term ‘totipotency’ refers to the capacity of a (1) cell to generate whole plant. (2) bud to generate whole plant. (3) seed to germinate. (4) cell to enlarge in size. Directions : In the following questions a statement of assertion (A) is followed by a statement of reason (R). Mark the correct choice as : (1) Both assertion (A) and reason (R) are true and reason (R) is the correct explanation of assertion (A). (2) Both assertion (A) and reason (R) are true but reason (R) is not the correct explanation of as­ sertion (A). (3) Assertion (A) is true but reason (R) is false. (4) Assertion (A) is false but reason (R) is true.

17. Assertion (A) : It is essential to have few cloning



Reason(R) : It helps in identifying and eliminating non-transformants and selectively permitting the growth of the transformants.

sites in cloning vector.

(2) phosphodiester bonds. (4) hydrogen bonds.

20. Match the following list of bacteria and their commercially important products. Column A (Bacterium)

11. Several South Indian states raise 2–3 crops of rice

annually. The agronomic feature that makes this possible is because of (1) shorter rice plant. (2) better irrigation facilities. (3) early yielding rice variety. (4) disease resistant rice variety.



microsporangium in an anther are respectively, (1) endothecium and tapetum. (2) epidermis and endodermis. (3) epidermis and middle layer. (4) epidermis and tapetum.

Column B (Product)

A Aspergillus niger

(i)

Lactic acid

B

Acetobacter aceti

(ii)

Butyric acid

C

Clostridium butylicum

(iii) Acetic acid

D Lactobacillus

(iv) Citric acid



Choose the correct match. (1) A – (ii), B – (iii), C – (iv), D – (i) (2) A – (ii), B – (iv), C – (iii), D – (i) (3) A – (iv), B – (iii), C – (ii), D – (i) (4) A – (iv), B – (i), C – (iii), D – (ii)

21. The process of separation and purification of

expressed protein before marketing is called : (1) Upstream processing (2) Downstream processing (3) Bio processing (4) Post production processing 22. World’s most problematic aquatic weed is (1) Azolla. (2) Wolffia. (3) Eichhornia. (4) Trapa.

23. A particular species of plant produces light, non-

sticky pollen in large numbers and its stigmas are long and feathery. These modifications facilitate pollination by (1) insects. (2) water. (3) wind. (4) animals.

24. Which Mendelian idea is depicted by a cross in

which F1 generation resembles both the parents? (1) Co-dominance (2) Incomplete dominance (3) Law of dominance (4) Inheritance of one gene

25. Which one of the following diseases is not due to

contamination of water? (1) Hepatitis-B (2) Jaundice (3) Cholera (4) Typhoid

26. Discontinuous synthesis of DNA occurs in one

strand, because (1) DNA molecule being synthesised is very long. (2) DNA-dependent DNA polymerase catalyses polymerisation only in one direction (5′→3′). (3) it is a more efficient process. (4) DNA ligase has to have a role.

32

OSWAAL CUET (UG) Sample Question Papers, BIOLOGY

27. Edaphology is concerned with study of : (1) Soil (2) Climate (3) Biota (4) Topography 28. The first organisms were (1) autotrophs. (2) eukaryotes. (3) chemoautotrophs. (4) chemo-heterotrophs.

29. While planning for an artificial hybridisation programme involving dioecious plants, which of the following steps would not be relevant? (1) Bagging of female flower (2) Dusting of pollen on stigma (3) Emasculation (4) Collection of pollen

30. Removal of introns and joining of exons in a defined

order during transcription is called : (1) Slicing (2) Splicing (3) Looping (4) Inducing

31. Diseases are broadly grouped into infectious and

non-infectious diseases. In the list given below, identify the infectious diseases. (i) Cancer (ii) Influenza (iii) Allergy (iv) Small pox (1) (i) and (ii) (2) (ii) and (iii) (3) (iii) and (iv) (4) (ii) and (iv)

32. Which of the following is commonly used as a

vector for introducing a DNA fragment in human lymphocytes ? (1) Retrovirus (2) pBR322 (3) l Phage (4) Ti plasmid

33. Seminal plasma, the fluid part of semen, is contributed by



(i) Seminal vesicle (iii) Urethra (1) i and ii (3) ii, iii and iv

(ii) Prostate (iv) Bulbo-urethral gland (2) i, ii and iv (4) i and iv

34. If the base sequence of a codon in mRNA is 5′-AUG-

3′, the sequence of tRNA pairing with it must be (1) 5’-UAC-3’. (2) 5’-CAU-3’. (3) 5’-AUG-3’. (4) 5’-GUA-3’.

35. Use of bio-resources by multinational companies

and organisations without authorisation from the concerned country and its people is called (1) Bio-infringement (2) Bioexploitation (3) Biodegradation (4) Biopiracy 36. Haemozoin is (1) a precursor of haemoglobin. (2) a toxin from Streptococcus. (3) a toxin from Plasmodium species. (4) a toxin from Haemophilus species.

37. Which of the following layer of uterus is involved in

the implantation of embryo and menstrual cycle? (1) Clitoris (2) Isthmus (3) Mon pubis (4) Endometrium

38. Which of the following steps in transcription is catalysed by RNA polymerase ? (1) Initiation (2) Elongation (3) Termination (4) All of the above 39. Transgenic animals are (1) Animals whose DNA is manipulated to possess and express an extra (foreign) gene (2) Animals whose RNA is manipulated to possess and express an extra (foreign) gene (3) Animals whose both DNA and RNA are manipulated to possess and express an extra (foreign) gene. (4) None of the above 40. Which of the following is not a lymphoid tissue? (1) Spleen (2) Tonsils (3) Appendix (4) Thymus

41. Which of the following statements is the most

appropriate for sickle cell anaemia? (1) It cannot be treated with iron supplements. (2) It is a molecular disease. (3) It confers resistance to acquiring malaria. (4) All of the above.

42. Intensely lactating mothers do not generally

conceive due to the (1) suppression of gonadotropins. (2) hyper-secretion of gonadotrophins. (3) suppression of gametic transport. (4) suppression of fertilisation.

43. Transplantation of tissues/organs to save certain

patients often fails due to rejection of such tissues/ organs by the patient. Which type of immune response is responsible for such rejections? (1) Auto-immune response (2) Humoral immune response (3) Physiological immune response (4) Cell-mediated immune response

44. Crystals of Bt toxin produced by some bacteria do

not kill the bacteria themselves because (1) bacteria are resistant to the toxin. (2) toxin is immature. (3) toxin is inactive. (4) bacteria enclose toxin in a special sac.

45. Which was absent in the atmosphere at the time of

origin of life? (1) NH3 (3) O2

(2) H2 (4) CH4



Read the following text and answer the following questions on the basis of the same:



Pollution, climate changes, over-exploitation by human etc., are responsible for changes in the ecosystem. Change in an ecosystem is responsible for depletion in biodiversity which directly or indirectly affects human beings and their surrounding. So, it is important to conserve the

33

Sample Question Papers biodiversity. There are various methods like in-situ conservation, ex situ conservation by which we can conserve our biodiversity.

46. How many biodiversity hot-spots are identified

globally ? (1) 20 (3) 34

(2) 3 (4) 50

47. Main cause of extinction of animals and plants is : (1) Habitat loss and fragmentation (2) Competition between species (3) Over-exploitation (4) Alien species invasion.

48. Protected areas are example of (1) In-situ conservation (2) Ex-situ conservation (3) Cryopreservation (4) Green houses. 49. Name the National Aquatic Animal of India ? (1) Sea-horse (2) Gangetic shark (3) Blue whale (4) River dolphin

50. Species at the high risk of extinction in the future is

called  : (1) Vulnerable (3) Endemic

(2) Extinct (4) Critically Endangered

qqq

SAMPLE

Question Paper Maximum Marks : 200

10

Time : 45 Minutes

General Instructions: (i) This paper consists of 50 MCQs, attempt any 40 out of 50 . (ii) Correct answer or the most appropriate answer: Five marks (+5) . (iii) Any incorrect option marked will be given minus one mark (–1) . (iv) Unanswered/Marked for Review will be given no mark (0) . (v) If more than one option is found to be correct then Five marks (+5) will be awarded to only those who have marked any of the correct options . (vi) If all options are found to be correct then Five marks (+5) will be awarded to all those who have attempted the question . (vii) If none of the options is found correct or a Question is found to be wrong or a Question is dropped then all candidates who have appeared will be given five marks (+5). (viii) Calculator / any electronic gadgets are not permitted .

1. Nuisance growth of aquatic plants and bloom-



2. The trigger for activation of toxin of Bacillus

use of (1) short interfering RNA (RNAi). (2) antisense RNA. (3) by both (1) and (2). (4) none of the above.

3. Distance between the genes and percentage of



forming algae in natural waters is generally due to high concentrations of (1) carbon (2) sulphur (3) calcium (4) phosphorus

thuringiensis is (1) acidic ph of stomach. (2) high temperature. (3) alkaline ph of gut. (4) mechanical action in the insect gut.

recombination shows (1) a direct relationship. (2) an inverse relationship. (3) a parallel relationship. (4) no relationship.

4. The method of directly injecting a sperm into ovum

is assisted by reproductive technology called (1) GIFT. (2) ZIFT. (3) ICSI. (4) ET.

5. The bones of forelimbs of whale, bat, cheetah and

man are similar in structure, because (1) one organism has given rise to another. (2) they share a common ancestor. (3) they perform the same function. (4) they have bio-chemical similarities.

6. Which one of the following impurities is easiest to

remove from waste water? (1) Bacteria (2) Colloids (3) Dissolved solids (4) Suspended solids

7. Silencing of a gene could be achieved through the

8. ‘Smack’ is a drug obtained from the (1) latex of Papaver somniferum. (2) leaves of Cannabis sativa. (3) flowers of Dhatura. (4) fruits of Erythroxylum coca.

9. Which of the following will not result in variations

among siblings? (1) Independent assortment of genes (2) Crossing over (3) Linkage (4) Mutation

10. Choose the correct statement regarding the ZIFT

procedure. (1) Ova collected from a female donor are transferred to the Fallopian tube to facilitate zygote formation. (2) Zygote is collected from a female donor and transferred to the Fallopian tube. (3) Zygote is collected from a female donor and transferred to the uterus. (4) Ova collected from a female donor and transferred to the uterus. 11. The green scum seen in the fresh water bodies is (1) blue green algae. (2) red algae. (3) green algae. (4) both (1) and (3)

Sample Question Papers

12. ADA is an enzyme which is deficient in a genetic



13. Match the following and choose the correct options.

disorder SCID. What is the full form of ADA? (1) Adenosine deoxyaminase (2) Adenosine deaminase (3) Aspartate deaminase (4) Arginine deaminase Column I

Column II

A.

Trophoblast

i.

Embedding of blastocyst in the endometrium

B.

Cleavage

ii.

Group of cells that would differentiate as embryo

C.

Inner cell mass

iii. Outer layer of blastocyst attached to the endometrium

D.

Implantation

iv. Mitotic division of zygote

(1) A-ii, B-i, C-iii, D-iv (3) A-iii, B-i, C-ii, D-iv

(2) A-iii, B-iv, C-ii, D-i (4) A-ii, B-iv, C-iii, D-i

14. In 1953, S.L. Miller created primitive earth conditions

in the laboratory and gave experimental evidence for origin of first form of life from pre-existing non-living organic molecules. The primitive earth conditions created include (1) low temperature, volcanic storms and atmosphere rich in oxygen. (2) low temperature, volcanic storms and reducing atmosphere. (3) high temperature, volcanic storms and nonreducing atmosphere. (4) high temperature, volcanic storms, reducing atmosphere containing CH4, NH3, etc. 15. The active chemical drug reserpine is obtained from (1) datura. (2) Rauwolfia. (3) Atropa. (4) Papaver.

19. What is common to the techniques (a) in-vitro fertilisation, (b) Cryopreservation and (c) tissue culture? (1) All are in-situ conservation methods. (2) All are ex-situ conservation methods. (3) All require ultra-modern equipment and large space. (4) All are methods of conservation of extinct organisms.

21. What is the correct sequence of sperm formation ? (1) Spermatogonia, spermatozoa, spermatocyte, spermatid (2) Spermatogonia, spermatocyte, spermatid, spermatozoa (3) Spermatid, spermatocyte, spermatogonia, spermatozoa (4) Spermatogonia, spermatocyte, spermatozoa, spermatid 22. Edaphic factor refers to (1) Water. (3) Relative humidity.

23. 1st cloned animal is :



24. Which condition of zygote cell will lead to birth of a

(1) Rat (3) Pig

(2) Rabbit (4) Sheep.

normal human female child ? (1) One X-chromosome (2) One X and one Y chromosome (3) Two X chromosome (4) One Y chromosome

25. Match between the following representing parts

of the sperm and their functions and choose the correct option. Column A

17. A woman has an X-linked condition on one of her

18. Choose the incorrect statement from the following. (1) In birds and mammals, internal fertilisation takes place. (2) Colostrum contains antibodies and nutrients. (3) Polyspermy is prevented by the chemical changes in the egg surface. (4) In the human female, implantation occurs almost seven days after fertilisation.

(2) Soil. (4) Altitude.



16. A recombinant DNA molecule can be produced in

X-chromosomes. This chromosome can be inherited by (1) only daughters (2) both sons and daughters (3) only grandchildren (4) only sons

20. While isolating DNA from bacteria, which of the

following enzymes is not used? (1) Lysozyme (4) Ribonuclease (3) Deoxyribonuclease (5) Protease

the absence of the following (1) Restriction endonuclease (2) DNA ligase (3) DNA fragments (4) Escherichia coli

35

A.

Head

i.

Enzymes

B.

Middle piece

ii

Sperm motility

C.

Acrosome

iii.

Energy

D.

Tail

iv.

Genetic material

(1) A-ii, B-iv, C-i, D-iii (3) A-iv, B-i, C-ii, D-iii

Column B

(2) A-iv, B-iii, C-i, D-ii (4) A-ii, B-i, C-iii, D-iv

26. The sequence of communities of primary succession

in water is (1) phytoplankton, sedges, free-floating hydrophytes, rooted hydrophytes, grasses and trees. (2) phytoplankton, free-floating hydrophytes, rooted hydrophytes, sedges, grasses and trees. (3) free-floating hydrophytes, sedges, phytoplankton, rooted hydrophytes, grasses and trees. (4) phytoplankton, rooted submerged hydrophytes, floating hydrophytes, reed swamp, sedges, meadow and trees.

36

OSWAAL CUET (UG) Sample Question Papers, BIOLOGY

27. Heaviest molecule of protoplasm is : (1) Lipids (2) Proteins (3) DNA (4) RNA.

28. If both parents have sickle cell trait, then there is

_______________of the child having sickle cell trait. (1) 25 % risk (2) 50 % risk (3) 75% risk (4) No risk





37. In 2005, for each of the 14 million people present in

29. Which of the following is an ecosystem service

a country, 0.028 were born and 0.008 died during the year. Using exponential equation, the number of people present in 2015 is predicted as : (1) 25 million (2) 17 million (3) 20 million (4) 18 million

31. Which among the following does not confer stability

38. What does ‘Bt’ in Bt toxin represent ? (1) Bioterrorism (2) Blue tooth toxin (3) Bleeding toxin (4) Bacillus thuringiensis Directions : In the following questions a statement of assertion (A) is followed by a statement of reason (R). Mark the correct choice as : (1) Both assertion (A) and reason (R) are true and reason (R) is the correct explanation of assertion (A). (2) Both assertion (A) and reason (R) are true but reason (R) is not the correct explanation of assertion (A). (3) Assertion (A) is true but reason (R) is false. (4) Assertion (A) is false but reason (R) is true.

provided by a natural ecosystem? (1) Cycling of nutrients. (2) Prevention of soil erosion. (3) Pollutant absorption and reduction of the threat of global warming. (4) All of the above. 30. DPT vaccine is used for the immunization of : (1) Cholera (2) Bubonic plague (3) Typhoid fever (4) Whooping cough.

developed ears, eyes, hair and other phenotypic characters? (1) Cope’s Law (2) Dollo’s Law (3) Allen’s Law (4) Bergmann’s Law

to the helical structure of DNA ? (1) Phosphodiester bond (2) H-bond (3) N-glycosidic linkage (4) All of these.

32. From the statements given below, choose the options

that are true for a typical female gametophyte of a flowering plant. (i) It is 8-nucleate and 7-celled at maturity. (ii) It is free-nuclear during the development. (iii) It is situated inside the integument but outside the nucellus. (iv) It has an egg apparatus situated at the chalazal end. (1) (i) and (iv) (2) (ii) and (iii) (3) (i) and (ii) (4) (ii) and (iv)

39. Assertion (A) : Diphtheria is caused by Corynebacterium diphtheriae.



Reason (R) : Corynebacterium diphtheriae produce a potent exotoxin that inhibits protein synthesis in epithelial cells.



40. Assertion (A) : Aminoacylation is an essential step

compared to the older individuals. What would be the status of the population after some years? (1) It will decline. (2) It will stabilise. (3) It will increase. (4) It will first decline and then stabilise.



for the synthesis of protein. Reason (R) : It is the process of adding an activated amino acid to the acceptor arm of a transfer RNA.



41. Non-specific host defence that exists prior to the

34. Regulatory proteins are the accessory proteins that



42. Which compound is formed by acetylation of





33. A population has more young individuals

interact with RNA polymerase and affect its role in transcription. Which of the following statements is correct about regulatory protein? (1) They only increase expression. (2) They only decrease expression. (3) They interact with RNA polymerase, but do not affect the expression. (4) They can act both as activators and as repressors.

35. Among the terms listed below, those that of are not technically correct names for a floral whorl are :



i. Androecium ii. Carpel iii. Corolla iv. Sepal (1) i and iv (2) iii and iv (3) ii and iv (4) i and ii.

36. Which of the following law shows that the animals of cold countries have relatively shorter and poorly

exposure of an antigen is called : (1) acquired immunity (2) passive immunity (3) innate immunity (4) active immunity morphine ? (1) Heroin (3) Tobacco

43. Which of the following is an example for link

species? (1) Lobe fish (3) Sea weeds

(2) Cocaine (4) Marijuana

(2) Dodo bird (4) Chimpanzee

44. The statements given below describe certain

features that are observed in the pistil of flowers. (i) Pistil may have many carpels. (ii) Each carpel may have more than one ovule. (iii) Each carpel has only one ovule. (iv) Pistils have only one carpel.



Choose the statements that are true from the options below. (1) (i) and (ii) (2) (i) and (iii) (3) (ii) and (iv) (4) (iii) and (iv)

37

Sample Question Papers

45. Which of the following is a post-fertilisation event

in flowering plants? (1) Transfer of pollen grains (2) Embryo development (3) Formation of flower (4) Formation of pollen grains Read the following text and answer the these questions on the basis of the same: Large quantities of sewage is generated every day in cities and towns, which is treated in Sewage Treatment Plants (STPs) to make it less polluted. Given below is the flow diagram of one of the stages of STP.

Observe the given flow diagram and answer the questions accordingly.

Primary effluent is passed into large aeration tank.

Effluent passed into settling tank to form the sediment. 46. Primary effluent is passed into large aeration tanks: (1) To allow bacterial flocs to settle down (2) To allow fast growth of useful aerobic microbes into flocs (3) To allow anaerobic sludge digestion (4) None of these



47. Technical term used for sediment formed in

sewage treatment is: (1) Flocs (2) Effluents (3) Activated sludge (4) Anerobic sludge

48. The significance of the above sediment formed is

that: (1) It acts as inoculum (2) It serves in formation of flocs (3) It helps in formation of natural water bodies (4) It helps in anerobic sludge digestion

49. Identify the correct set of gases produced during

digestion of sludge: (1) methane, hydrogen sulphide and carbon dioxide (2) methane, sulphur and carbon dioxide (3) carbon mono-oxide, methane, hydrogen sulphide (4) methane, hydrogen sulphide and nitrogen

50. The technology of biogas production from cow

dung was developed in India largely due to the efforts of : (1) Gas authority of India (2) Oil and Natural Gas Commission (3) Indian Agriculture Research Institute, Khadi and Village Industries Commission. (4) Indian Oil Corporation

qqq

SOLUTIONS OF Question Paper 1. Option (3) is correct. Explanation : Zoospore is a motile, a sexual spore, which uses its flagella for locomotion Example : Euglena.

2. Option (2) is correct. Explanation : The genotypes of both the parents are Tt and Tt. Refer the given cross between true breeding tall plants and true breeding dwarf plants.

1

plasmid which transforms normal plant cells into tumour cells to produce chemicals required by pathogens. The restriction enzyme EcoRl, is isolated from E. coli RY13. The first restriction enzymes Hind II was isolated from bacterium Haemophilus influenzae. The restriction enzyme Bam HI is isolated from Bacillus amyloliquefaciens.

5. Option (2) is correct. Explanation : Amensalism is an association between two organisms in which one species is unaffected while the other organism is destroyed or inhibited.

6. Option (2) is correct. Explanation : In an unfertilised embryo sac, the antipodals and synergids are distinctly present at chalazal end and micropylar end respectively while, in fertilised embryo sac, antipodals and synergids gradually degenerate after the formation of zygote.

7. Option(2) is correct.

When true breeding plants were crossed to each other, this is called a parental cross and offspring comprise the first filial or F1 - generation. When the members of the F1 - generation were crossed, this produced the F2-generation or second filial generation. A cross between true breeding tall and dwarf plants of the parent generation yield phenotypically tall plants. The cross between TT and Tt is called back cross, which results into two homozygous and two heterozygous dominant gametes. The cross between Tt and tt is called test cross which results into 1 : 1 ratio of gametes.

3. Option (1) is correct. Explanation : Opium, narcotic drug, is obtained from the unripe seedpods of the opium poppy (Papaver somniferum).

4. Option (3) is correct. Explanation : Agrobacterium tumefaciens is a pathogen of several dicot plants. It delivers a piece of DNA known as T-DNA in the Ti

Explanation : The promoter site and the terminator site for transcription are located at 5' (upstream) end and 3' (downstream) end, respectively of the transcription unit. The promoter is the binding site for RNA polymerase for initiation of transcription.

8. Option(2) is correct. Explanation : Hybrid vigour is superior in characteristic such as size, growth rate, fertility, etc., of a hybrid organism over it's parents. Hybrid, on the other hand is a result of cross between parents of two different types.

9. Option (4) is correct. Explanation : Bt toxin protein exist as inactive protoxins but once an insect ingests the inactive toxin, it is converted into an active form of toxin due to the alkaline pH of the gut which solubilise the crystals.

10. Option (2) is correct. Explanation : About 10% of the net primary productivity of a terrestrial ecosystem is eaten and digested by herbivores.

Solutions 11. Option (2) is correct. Explanation : Ampulla is a part of Fallopian tube where fertilisation occurs. Fallopian tube is part of female reproductive system.

12. Option (2) is correct. Explanation : Starting from the innermost part, the correct sequence of parts in an ovule is egg, embryo sac, nucellus, and integument. This sequence can be seen in following diagrammatic view of an ovule.

39

16. Option (3) is correct. Explanation : Artificial insemination (AI) is a technique in which the semen collected from the husband or a healthy donor is artificially introduced either into the vagina or the uterus.

17. Option (3) is correct. Explanation : Sickle-cell anaemia is caused by a single point mutation in the beta haemoglobin gene which converts a GAG, codon into GUG, which code for valine amino acid rather than glutamic acid.

18. Option (4) is correct.

13. Option (4) is correct. Explanation : Pseudomonas is a denitrifying bacterium, which converts ammonia and nitrates into free nitrogen. These bacteria are responsible to release free nitrogen in the environment through nitrogen cycle. Anabaena, Nostoc and Azotobacter are nitrogen fixing organisms. Anabaena and Nostoc are types of cyanobacteria which are widely distributed in aquatic and terrestrial environments and can fix atmospheric nitrogen. Azotobacter is free-living bacteria, which absorb free nitrogen from soil, air and convert it into salts of nitrogen like amino acids and enrich soil nutrients.

14. Option (2) is correct. Explanation : Selectable markers help in identifying and eliminating non-transformants and selectively permitting the growth of the transformants. The normal E. coli cells do not carry resistance against any antibiotic. Competent bacterial cells are made capable to take foreign DNA with chemical treatment (e.g., calcium chloride).

15. Option (1) is correct. Explanation : Rice has more than 50,000 genetically different strains while mango has 1500 varieties in India.

Explanation : Widal test is used to help make a presumptive diagnosis of enteric fever, also known as typhoid fever. Typhoid is caused by Salmonella typhi. It is spread through food, milk, and water contaminated with intestinal discharges either directly or through flies and personal hygiene. Enzyme-linked immunosorbent assay (ELISA) is used in immunology to detect the presence of an antibody or an antigen in a sample. Erythrocyte sedimentation rate test (ESR) is used to detects inflammation that may be caused by infection and some autoimmune diseases. Polymerase chain reaction (PCR) test is used in molecular biology. It is used to make numerous copies of a specific segment of DNA quickly and accurately.

19. Option (4) is correct. Explanation : The stirred tank bioreactor is well suited for large scale production of micro organism under aseptic condition for a number of days. It can be used easily in research laboratory and main advantage is an oxygen delivery system which provides oxygen without any interruption.

20. Option (3) is correct. Explanation : Ecotone is a transition zone between two communities. The adjacent biotic (natural) communities generally do not possess a fine demarcation line between them. Therefore, the adjacent of two communities is represented by population of both the communities, and this transition zone between two communities is referred as ecotone.

21. Option (3) is correct. Explanation : Statements (i), (ii) and (iv) support the view that elaborates sexual reproductive process, appeared much later in the organic evolution. Asexual reproduction is not common in higher group of organisms. It is common amongst single-celled organisms, and also in plants and animals with relatively simple organisations.

40

OSWAAL CUET (UG) Sample Question Papers, BIOLOGY

22. Option (4) is correct. Explanation : Z Z / ZW type of sex determination is seen in birds, reptiles and fish. Thus, peacock shows ZZ/ZW type sex determination. In this type, female has heteromorphic (ZW) sex chromosomes and the male has homomorphic (ZZ) sex chromosomes. In Platypus the sex determination is of XX-XY type in which both male and females has ten sex chromosome each. The male has XY, XY, XY, XY, XY and female has XX, XX, XX, XX, XX. In snails, the sex determination is environmentally induced, while in cockroaches it is of XX-XO types. In this type Y-chromosome is absent. In this the presence of unpaired X-chromosomes determines the masculine sex.

23. Option (2) is correct. Explanation : The rupture of RBCs is associated with release of a toxic substance, haemozoin, which is responsible for the chill and high fever recurring every three to four days.

24. Option (3) is correct. Explanation : Polymerase chain reaction technique was developed by Kary Mullis in 1985, and for this he received Nobel Prize for chemistry in 1993. It is a reaction in which amplification of specific DNA sequences is carried out in vitro.

25. Option (3) is correct. Explanation : Ecological niche is ecologically adapted zone, that is, the particular place of habitat occupied by individual of a species within its ecosystem. Ecological niche have specific characteristics, such as availability of nutrients, temperature, terrain, sunlight, and predators, which influences how populations affect and are affected by resources and enemies.

26. Option (4) is correct. Explanation : Adult filament of multicellular alga have haplontic life cycle in which the meiotic division occurs after the formation of zygote. So, the filament of this alga has haploid vegetative cells and haploid gametangia. A multicellular gametophyte or gametangia which is haploid (n) alternates with a multicellular sporophyte which is diploid (2n). A mature sporophyte produces spores (haploid cells) by meiosis, a process which reduces the number of chromosomes to half, from 2n to n.

27. Option (1) is correct. Explanation : It is a disorder caused due to the absence of one of the X-chromosomes, i.e., 45 with XO.

28. Option (1) is correct. Explanation : The entire collection of plants/ seeds which have all the diverse alleles for all genes in a given crop is called germplasm collection. Protoplasm collection refers to collection of protoplast (i.e., plant cell without cell wall). Herbarium is a museum of preserved plants that are used for botanical research, mainly in identification and classification of plants. Soma-clonal collection consists of those plants that are produced from a single cell which are genetically variable from their parents.

29. Option (4) is correct. Explanation : In 1997, Rosie , the first transgenic cow produced human protein-enriched milk (2.4 gm per litre).

30. Option (3) is correct. Explanation : Productivity refers to the rate of generation of biomass in an ecosystem. It is usually expressed in terms of g m–2 d–1 or kcal m–2 yr–1 to compare the different ecosystem.

31. Option (3) is correct Explanation : In female gametophyte,central cell is involved in the double fertilisation that help in the endosperm development. While antipodal cells provides nourishment to the egg cell, and synergid cell help in pollen tube growth.

32. Option (3) is correct. Explanation : DNA consists of a nitrogenous base, pentose sugar and a phosphate group. DNA has negative charge due to the presence of phosphate group. Histone is rich in the basic amino acids lysine and arginine, which carry positive charges in their side chains. Therefore, histone is positively charged.

33. Option (3) is correct. Explanation : The large holes in ‘Swiss cheese’ are made due to production of a large amount of CO2 by Propionibacterium shermanii. It releases carbon dioxide when it consumes the lactic acid and forms bubbles. These bubbles form little air pockets, resulting in the holes of the Swiss cheese.

34. Option (2) is correct. Explanation : Restriction enzymes (also called molecular scissors) are responsible for cutting DNA. These enzymes belong to a class of enzymes called nucleases and are of two types : (i) Exonuclease which cut DNA at the ends and (ii) endonucleases which make cuts at specific positions within the DNA. The

Solutions term ‘restriction’ refers to the function of these enzymes in restricting the propagation of foreign DNA of bacteriophage in host bacterium, that is, cutting of DNA, at specific position only.

35. Option (4) is correct. Explanation : Biodiversity hot-spots specifically refer to 25 biologically rich areas around the world that have lost at least 70% of their original habitat. They are characterised by large number of flora and fauna, abundance of endemic species and also large number of alien or exotic species. They are mostly found in tropical and temperate regions. There are no biodiversity hot-spots in polar regions.

36. Option (3) is correct. Explanation : Vasectomy is the correct surgical procedure as a contraceptive method in male. During this procedure, vas deferens is cut down to prevent sperm from entering into the urethra and thereby preventing fertilisation. In female, this procedure is called tubectomy in which Fallopian tubes are held and blocked or severed and sealed, either of which prevents eggs from reaching the uterus for implantation. Removal of ovary, uterus and testes are called as ovariectomy, hysterectomy and castration respectively.

37. Option (3) is correct. Explanation : Amphetamines (stimulants) mimic the effect of the naturally produced hormone adrenaline.

38. Option (3) is correct. Explanation : A protoplast is a plant, bacterial or fungal cell whose cell wall is completely or partially removed using either mechanical or enzymatic means.

39. Option (2) is correct. Explanation : During the sexual reproduction, the events which take place after the formation of zygote are called post-fertilisation events. The process of embryo development from the zygote (called embryogenesis) takes place after the fertilisation. In all flowering plants, the zygote is formed inside the ovule. In ovule, the zygote divides several times to form an embryo.

40. Option (2) is correct. Explanation : Dioecious plants (bearing only male or female flowers) prevent both autogamy and geitonogamy. Autogamy is a method of self-pollination in which the transfer of pollen grains from anther to stigma of the same flower

41

takes place. Geitonogamy is the transfer of pollen grains from anther to stigma of another flower of the same plant. It is ecologically cross - pollination which is supposed to be equivalent to self-pollination because all flowers on a plant are genetically identical.

41. Option (4) is correct. Explanation : The body is able to differentiate self and non-self and the cell-mediated immune response is responsible for the graft rejection.

42. Option (2) is correct. Explanation : The fern, Azolla and the bluegreen alga Anabaena azollae maintain a symbiotic relationship. The alga provides nitrogen to the fern, and the fern provides a habitat for the alga. This property of nitrogen fixation has made Azolla extremely important economically in the cultivation of rice.

43. Option (3) is correct. Explanation : Non-biodegradable pollutants are those pollutants which are not broken down into simpler substances by natural biological processes. These pollutants include plastics, tin container, heavy metals, radioactive substances, etc. These are created by human activities, like industrialisation. Whereas biodegradable pollutant created by humans (for example paper, household waste like peel of vegetables fruits, etc.) are degraded or disposed quickly by biological processes.

44. Option (3) is correct. Explanation : Keeping animals in zoological parks is not a cause for loss of biodiversity. The important factors causing loss of biodiversity are as follows : (i) Destruction of natural habitat (primary cause) (ii) Introduction of exotic (alien species) and indigenous species (iii) Over-exploitation of natural resources (iv) Co-extinction of species

45. Option (3) is correct. Explanation : Number of producers is high in grassland or forest or pond and therefore, pyramid of number is upright in these ecosystems while it is inverted in a parasitic food chain.

46. Option (3) is correct. Explanation : The affected person inherited with one extra copy of 21st chromosome that forms trisomy condition.

42

OSWAAL CUET (UG) Sample Question Papers, BIOLOGY

47. Option (2) is correct. Explanation : Down syndrome is an autosomal recessive disorder which can be inherited through normal parents in the child.

48. Option (3) is correct. Explanation : The person affected with Down syndrome has symptoms like mental retarded, short statured with small round, head, furrowed tongue, etc.

49. Option (2) is correct. Explanation : Down Syndrome is due to extra copy of 21st chromosome forming trisomy condition.

50. Option (4) is correct. Explanation : Down syndrome is due to autosomal chromosome abnormality.

qqq

SOLUTIONS OF Question Paper 1. Option (1) is correct. Explanation : The whole plant body of maize plant including shoot tip cells remains in diploid (2n) condition. As the microspore mother cell is a part of reproductive organ, the chromosome number in these cells will remain same as the individual, that is, 2n = 20. These microspore mother cells are further responsible for producing male gametes, that is, haploid (n) by reduction division.

2. Option (1) is correct. Explanation : Breast-feeding is one of the temporary and natural contraceptive methods which reduce the secretion of oestrogen hormone and thereby suppress ovulation (release of egg) and cause amenorrhoea. The contraceptive effect of the lactational amenorrhoea method is a result of increased levels of prolactin. In this condition, production and secretion of gonadotrophin releasing hormones are inhibited and resulted in low secretion of oestrogen. As ovulation cannot occur without a surge in oestrogen levels, and if a woman does not ovulate, pregnancy is prevented.

3. Option (2) is correct. Explanation : ABO blood grouping in humans is an example of co-dominance. ABO blood groups are controlled by gene I. Gene I has three alleles IA, IB, and Ii. When IA, and IB are present together, both express equally and produce the surface antigens A and B, whereas 'i' is the recessive allele and does not produce any antigen. Pleiotropy referred the genetic effect of a single gene on multiple phenotypic traits. Incomplete dominance does not completely dominate another allele. Segregation is the separation of allele during the process of gametogenesis. This is the basis of reappearance of recessive character in F2 generation.

4. Option (4) is correct. Explanation : In the given list, influenza and small pox are infectious diseases. Infectious diseases can be spread from one person to another, through contact with body fluids,

2

by aerosols (through coughing and sneezing), or via a vector. Whereas cancer and allergy are non-communicable diseases and cannot spread from infected persons to healthy person.

5. Option (2) is correct. Explanation : A single PCR amplification cycle involves three basic steps : denaturation, annealing and extension. PCR stands for polymerase chain reaction in which multiple copies of the gene, for DNA of interest is synthesised in vitro.

6. Option (2) is correct. Explanation : Animal husbandry and plant breeding programmes are the examples of artificial selection. Artificial selection is a process in which human choose only those traits or characters which are transferred into their offspring. Mutation is a sudden change in DNA sequence due to mutagenic agents like chemicals and radiations. Natural selection is a slow process by which biological traits become either more or less common in a population as a function of the effect of the changing environment. Reverse evolution or devolution is a concept in which species can change into more primitive forms over time.

7. Option(3) is correct. Explanation : Glomus is a genus of arbuscular mycorrhizal fungi and, all species form symbiotic relationships with plant roots. They help the plant in the absorption of nutrients, especially phosphorus from soil.

8. Option(3) is correct. Explanation : Restriction enzymes are a protein produced by bacteria that cleaves DNA at specific sites. These are not found in viruses. They are present in bacteria to provide a type of defence mechanism (called the restriction modification system) against bacterial viruses. They are of two types : Endonuclease and exonuclease. They are indispensable tools in recombinant DNA technology and genetic engineering.

9. Option (1) is correct.

44

OSWAAL CUET (UG) Sample Question Papers, BIOLOGY Explanation : Cu++ in Cu releasing IUDs suppresses sperm motility and fertilising capacity of sperm.

10. Option (2) is correct. Explanation : Here is this case, the women is a carrier. Both the son and daughter will inherit the X-chromosome but only the son will be diseased.

11. Option (3) is correct. Explanation : A person with sickle cell anaemia is less prone to malaria because in sickle cell anaemia red blood cells becomes abnormally shaped, if they inherit two faulty copies of the gene for the oxygen-carrying protein haemoglobin. The faulty gene persists because even carrying one copy of it confers some resistance to malaria. Therefore, people with normal haemoglobin A are susceptible to death from malaria but people with sickle cell trait who have one gene for haemoglobin A and other for haemoglobin S have a greater chance of surviving malaria and do not suffer adverse consequences from the haemoglobin S gene.

12. Option (4) is correct. Explanation : The polymerase chain reaction (PCR) is a reaction in which amplification of specific DNA sequences is carried out in vitro. Such repeated amplification is achieved by using thermostable DNA polymerase (isolated from a bacterium, Thermus aquaticus) enzyme which remains active and stable during high temperature and induced denaturation of double-standard DNA.

13. Option (2) is correct. Explanation : Directional type of selection is industrial melanism observed in moth, Biston betularia. In directional selection, the population changes towards one particular direction. Industrial melanism refer to the evolution of dark body colour in animal species that live in habitats blackened by industrial soot. Under this condition, individuals at one end of the frequency distribution do well than light and so more individuals of that type will be present in next generation.

14. Option (2) is correct. Explanation : The primary treatment of waste water involves the physical removal of both larger and smaller particles (stable particles) from the sewage with the help of filtration and sedimentation. These removals occur in the following stages: (i) Initially, floating debris is removed by sequential filtration. (ii) After that

grit (made of soil and small pebbles) are removed by sedimentation. All these solids particles that settled form the primary sludge and the supernatant forms the effluent. The effluent is taken for the secondary treatment. The primary treatment does not remove the dissolved impurities, toxic substances and harmful bacteria.

15. Option (2) is correct. Explanation : Type I and Type III are complex and have only a limited role in genetic engineering. Type II restriction endonucleases are used mostly as the cutting enzymes in gene cloning.

16. Option (4) is correct. Explanation : Biosphere, one of the four layers that surround the Earth, is composed of all living organisms present on earth which interact with their physical environment. It is collectively used for all the ecosystems of world.

17. Option (2) is correct. Explanation : Prostate gland is the part of male reproductive system that is located just below the bladder. Small and walnut shaped, the prostate surrounds the beginning of the urethra.

18. Option (1) is correct. Explanation : Both assertion and reason are true. Stop codons are also known as non-sense codons or termination codons as they do not code for an amino acid. If they are not present, then protein synthesis will continue and result in defective protein.

19. Option (3) is correct. Explanation : Cocaine is a natural stimulant. It stimulates nervous system and delays fatigue. Bhang is a hallucinogen. Morphine is a natural opiate derived from opium. Reserpine is a tranquiliser. It has sleep inducing properties.

20. Option (4) is correct. Explanation : Transgenic animals that produce useful biological products can be created by the introduction of the portion of DNA (or genes) which codes for a particular product such as human protein (α-1-antitrypsin) used to treat emphysema.

21. Option (1) is correct. Explanation : Variations during mutations of meiotic recombinations are random and directionless. Hugo de Vries proposed mutation theory on the basis of his work on evening primrose. He stated that mutations are

Solutions sudden, heritable and persistent in successive generation. He contradicted Darwinian variations that are small and directional.

22. Option (2) is correct. Explanation : In anaerobic sludge digesters, bacteria produce a mixture of gases like methane, hydrogen sulphide and CO2.

23. Option (1) is correct. Explanation : Transgenic mice are being developed and used in testing the safety of vaccines before they are used for humans. The polio vaccine was tested in mice.

24. Option (3) is correct. Explanation : Variations in number of individuals in a population is expressed as population density and population size. A population of more young individual than older individuals will show positive growth in future (after some yrs), i.e., it will increase after some time.

25. Option (1) is correct. Explanation : Corona radiata is the innermost layer of the cells of the cumulus oophorus and is directly adjacent to the zona pellucida, the outer protective glycoprotein layer of the ovum. Zona radiata is a striated membrane situated next the yolk of an ovum. Chorion is an extra embryonic foetal membrane which is responsible for the formation of placenta.

26. Option (1) is correct. Explanation : The ribosome consists of structural RNAs and about 80 different proteins. In its inactive state, it exists as two subunits, a large subunit and a small subunit. When the smaller subunit encounters the mRNA, the process of translation of the mRNA to protein begins.

27. Option (2) is correct. Explanation : All the genes, present on a particular chromosome form a linkage group. The number of linkage group of a species correspond to the total number of different chromosomes of that species. It is not simply the number of chromosomes in haploid set. For example, in human male there are 22 pairs of autosomes and X and Y sex chromosomes, that is, 24 linkage groups and in female = 22 pairs autosomes + 2X-chromosomes, that is, 23 linkage groups.

45

28. Option (4) is correct. Explanation : Germplasm storing centre in India is ICRISAT (International Crops research institute for Semi-Arid tropics located at Hyderabad (Patancheru)).

29. Option (2) is correct. Explanation : The first clinical gene therapy was given in 1990 to a 4-year old girl with adenosine deaminase (ADA) deficiency. This enzyme is crucial for the immune system to function. The disorder is caused due to the deletion of the gene for adenosine deaminase. In some children ADA deficiency can be cured by bone marrow transplantation.

30. Option (1) is correct. Explanation : Banana plant is the largest herbaceous flowering plant. It is a monocarpic plant which reproduces sexually only once in its life time. These plants are generally annual (wheat, rice) or biennials (like - carrot and raddish.)

31. Option (1) is correct Explanation : Fossils are the remains or impression of a primitive plant or animal which are embedded in rock and preserved in petrified form. These are generally found in sedimentary rocks because of the following reasons: (i) These rocks are formed (on the Earth surface, under the water, etc.) from slits and muds at temperatures and pressures that do not destroy fossil remnants. (ii) These rocks are not formed due to volcanic eruption. (iii) These rocks remain the same.

32. Option (2) is correct. Explanation : A wide range of pathogen (e.g., fungi, bacteria and virus) affects the yield of cultivated crop species. Fungicides are generally used to control parasitic fungi that either cause economic damage to crop or ornamental plants or endanger the health of domestic animals or humans. Whereas antibiotics are medicines that help to stop infections caused by bacteria. Therefore, fungicides and antibiotics are chemicals that kill pathogenic fungi and bacteria, respectively.

46

OSWAAL CUET (UG) Sample Question Papers, BIOLOGY

33. Option (3) is correct. Explanation : All of the above plants are prone to over-exploitation due to their respective properties. Nepenthes (pitcher plant) is an insectivorous plant. Psilotum is a pteridophyte and Aconitum is a medicinal plant.

34. Option (4) is correct. Explanation : If a genetic disease is transferred from a phenotypically normal but carrier female to only some of the male progeny, the disease is sex linked recessive disease. Most sex-linked (X-linked) conditions are recessive. Because, in this condition a person with two X-chromosomes (females) must have a change or mutation whereas in a person with one X-chromosomes (males), only one copy of a gene must have a mutation. A female with a mutation in one copy of a gene on the X-chromosome is said to be a ‘carrier’ for an X-linked condition. For X-linked recessive disorders, an unaffected carrier mother who has a mutation in a gene on the X-chromosome can transmit either the X-chromosome with this mutation or a normal X-chromosome to her children (as shown below in cross).

37. Option (4) is correct. Explanation : It was concluded by the transforming principle, that bacteria is transformed from heat killed S-strain to live R-strain which is non-virulent.

38. Option (3) is correct. Explanation : Both deoxyribose and ribose belongs to the class pentoses as it contains '5' carbon atoms.

39. Option (3) is correct. Explanation : Essential amino acids are required for normal health and growth. These are usually supplied with dietary protein because these are not synthesised by the body. Lysine and tryptophan are types of essential amino acids.

40. Option (2) is correct. Explanation : The historic convention on biological diversity held in Rio de Janeiro (Brazil) on 3–4 June, 1992 is known as the Earth Summit. CITES (Convention on International Trade in Endangered Species of wild flora and fauna) has helped in restricting poaching and loss of rare species. MAB stands for Man and Biosphere Programme, which undertakes establishment and maintenance of biosphere reserves.

41. Option (3) is correct. Explanation : Fungi and bacteria have been categorised as heterotrophs and saprotrophs which derive nourishment from the decaying organic matter and thereby decompose complex substances into simpler substances. Autosomal dominant inheritance refers to the pattern of inheritance of a condition directly or indirectly due to a dominant faulty gene located on autosome. Autosomal recessive inheritance is caused directly or indirectly due to presence of recessive faulty gene copy on autosome. Sex-linked dominant is a rare trait, caused by a single abnormal gene on the X-chromosome.

35. Option (2) is correct. Explanation : Amphetamines stimulates the nervous system, increases alertness and produces excitement.

36. Option (1) is correct. Explanation : The species-area relationship is represented in linear form on a log scale.

42. Option (3) is correct. Explanation : A nitrogenous base is attached to the pentose sugar by an N-glycosidic linkage to form a nucleoside, that is, Nucleoside = Nitrogen base + Pentose sugar. When a phosphate group is attached to the 5’-OH of a nucleoside through phosphodiester linkage, a nucleotide is formed, that is, Nucleotide = Nitrogen base + Pentose sugar + Phosphate (PO4). So, a nucleoside differs from a nucleotide as it lacks the phosphate group.

43. Option (2) is correct. Explanation : Organisms in option (1), (3) and (4) that is Spirogyra, Volvox and Nostoc are producers. Therefore, Agaricus is a heterotroph which is not a producer.

Solutions 44. Option (1) is correct. Explanation : In a test cross an organism showing a dominant phenotype whose phenotype is to be determined (Whether it is homozygous or heterozygous for that trait) is crossed with a recessive parent.

45. Option (1) is correct. Explanation : Catalytic converters contain costly metals like rhodium and platinumpalladium as catalysts, and when exhaust gas passes to fitted catalytic converter, the unburnt hydrocarbons (cause of cancer) are oxidised into water and carbon dioxide. Catalytic converters are mainly designed to reduce immediate, local air pollution.

46. Option (1) is correct. Explanation : Mango possess a single ovule in each ovary and orchids, watermelon, and papaya have multiple ovules present in each ovary.

47

47. Option (4) is correct. Explanation : Papaver, brinjal, and tomato all have multicarpellary, syncarpous gynoecium. In this condition carpels are more than one and fused.

48. Option (1) is correct. Explanation : Anatropous ovule is found in 82% angiosperm and it completely inverted ovule turned back 180 degrees on its stalk.

49. Option (2) is correct. Explanation : Central cell form binucleate endosperm mother cell upon fertilisation with one of the two sperm cells, forms triploid endosperm to nourish embryo development.

50. Option (1) is correct. Explanation : Androecium is the male part and gynoecium is the female part, and in those flower have both of these they are called bisexual flower.

qqq

SOLUTIONS OF Question Paper 1. Option (3) is correct.

6. Option (1) is correct. Explanation : XO mechanism shows male heterogamety. Males produce two different types of gametes.

Androecium Gynoecium Corolla Calyx Flower

Pedicel

3

7. Option (2) is correct.

Calyx

Corolla

Androecium Gynoecium

Explanation : All the four whorls of the plant with their relative position in flower can be indicated through following diagram. Sepals collectively form a whorl, called as calyx while technically the carpel is known as gynoecium. The floral whorls formed by petals and stamens are called as corolla and androecium, respectively.

2. Option (2) is correct. Explanation : Klinefelter's syndrome is the genetic disorder which is caused due to the presence of an additional copy of X-chromosome resulting in karyotype of 47, XXY chromosome but only the son will be affected by the disease.

3. Option (1) is correct. Explanation : Nicotine is highly addictive and harmful for human health. Nicotiana tabacum, the type of nicotine is found in tobacco plants. Nicotine present in tobacco stimulates adrenal gland to release adrenaline and nor-adrenaline hormone which in turn increases the blood pressure and heart rate.

4. Option (4) is correct. Explanation : The separated DNA fragments (by the process of gel electrophoresis) are visualised after staining the DNA with ethidium bromide followed by exposure to ultraviolet (UV)-radiation. These fragments are seen as orange coloured bands.

5. Option (3) is correct. Explanation : The pollen grains represent the male gametophytes. As the anthers mature and dehydrate, the microspores dissociate from each other and develop into pollen grains. So, embryo sac is to ovule as pollen grain is to an anther.

Explanation : Cirrhosis ,which is a deadly liver disease caused due to excessive consumption of alcohol which damages the liver in such a way that is generally irreversible.

8. Option (2) is correct. Explanation : The DNA-dependent RNA polymerase helps in transcription by catalysing the polymerisation in only one direction (i.e., 5'-3').

9. Option (3) is correct. Explanation : Study of altered physiology of host is called pathophysiology.

10. Option (3) is correct. Explanation : In some viruses, like retro-viruses (e.g., HIV), an enzyme called reverse transcriptase is used to generate complementary DNA (cDNA) from an RNA template. This process is termed reverse transcription.

11. Option (1) is correct. Explanation : The restriction endonuclease binds two magnesium ions. One of these ions,binds to the phosphate group where the cleavage occurs and is required for catalysis.

12. Option (2) is correct. Explanation : During microsporogenesis, meiosis occurs in microspore mother cells. As the anther develops, the microspore mother cells of the sporogenous tissue undergo meiotic divisions to form microspore tetrads. The microspore tetrad after dehydration is developed into pollen grains.

13. Option (2) is correct. Explanation : PCR techniques is used to identity traces of DNA but it can not diagnose pathogen or organism.

14. Option (2) is correct

Solutions Explanation : Column A

Column B

Explanation

A

(iv)

Darwin proposed the theory of evolution by Natural Selection. Ac c o r d i n g t o t h i s t h e o r y, i n d i v i d u a l s t h a t a r e better adapted to their environment survive and reproduce more successfully than less well-adapted individuals.

B

C

D

(i)

(ii)

(iii)

Oparin put forth abio g e n e s i s t h e o r y. Ac c o r d ing to abiogenesis, life is originated from the non-living things spontaneously. Lamarck proposed theory of inheritance of acquired characters. Use and disuse of organs is one of the important principles of Lamarckism. Alfred Wegener proposed continental drift theory in 1912. According to this theory, 250 million years ago all the continents were combined into one super-continent (landmass) called Pangaea. Then Pangaea begins to break and has been slowly been moving towards the current position of today’s continent.

15. Option (3) is correct. Explanation : Bt toxin protein exists as inactive protoxins but once an insect ingest the inactive toxin, it is converted into an active form due to the alkaline pH of the gut which solubilise the crystals.

16. Option (1) is correct Explanation : (p+q)2 = p2 + 2pq + q2 = 1 represents an equation used in population genetics. Population genetics is the study of changes in gene frequencies in population of organisms and the effects of such changes on evolution and adaptation. (p+q)2 = p2 + 2pq + q2 = 1 represents the Hardy-Weinberg principle. This principle says that allele frequencies in a population are stable and is constant from generation to generation (i.e., the gene pool remains a constant).

49

Here, p2 represent the frequency of a homozygousdominant allele (AA). q2 represents the frequency of homozygousrecessive allele (aa). 2pq is the frequency of heterozygous individual (Aa).

17. Option (4) is correct. Explanation : Bt cotton is a genetically modified (GM) plant which is resistant to insects. These plants are produced by the insertion of one or more genes from a common soil bacterium, Bacillus thuringiensis. These genes encode for the production of insecticidal proteins, and thus, genetically transformed plants produce one or more toxins as they grow.

18. Option (2) is correct. Explanation : Oxytocin is produced in the hypothalamus and is secreted by the posterior pituitary gland. It causes contraction during parturition and help to brings baby out from the mother womb.

19. Option (3) is correct. Explanation : In forest ecosystem tall trees of forest plants controls the light condition, that is, intensity duration and quality of light at the ground. A well growing herbaceous plant in forest receive less intensity duration and quality of light, but when it is transplanted in a park outside its natural habitat, the light will be received uninterrupted. So, due to change in its microclimate, it may not survive.

20. Option (4) is correct Explanation : Analogous organs arise due to convergent evolution. Analogous organs are those organs which have different origin but similar function (e.g., wings of butterfly and birds look alike). Anatomically these are not similar structures though they perform similar functions. Hence, analogous structures are a result of convergent evolution (i.e., different structures which are not closely related but evolve for the same function and hence having similarity).

21. Option (1) is correct. Explanation: Heroin is synthesised by acetylation of the two hydroxyl groups of morphine with acetyl chloride, hence its other names, diacetylmorphine or diamorphine.

22. Option (3) is correct. Explanation : A single stranded DNA or RNA, tagged with a radioactive molecule (probe) is allowed to hybridise to its complementary DNA in a clone of cells followed by detection using autoradiography.

50

OSWAAL CUET (UG) Sample Question Papers, BIOLOGY

23. Option (4) is correct. Explanation : Biotic potential is the natural capacity of a population to multiply/increase at its maximum rate under favourable environmental conditions. Population of Paramecium show 100 per hour growth, i.e., two individuals are produced by one.

24. Option (3) is correct. Explanation : Lichens is an organism that is formed by the symbiotic association of a fungus and an alga or cyanobacterium. It occurs as crusty patches or bushy growths on tree trunks, bare ground, etc. and represents a positive (beneficial) interaction between two different species.

25. Option (1) is correct. Explanation : Acrosomal reaction of the sperm occurs due to its contact with zona pellucida of the ova. The reaction that occurs in acrosome of sperm is triggered by the release of fertilizin. The main purpose of the acrosomal reaction is to start the fusion of the oocyte membrane with the sperm cell membrane allowing the combination of genetic material contained in both gametes, leading to the fertilisation of the oocyte.

26. Option (2) is correct. Explanation : When large habitats are broken up into small fragments due to various human activities, mammals and birds requiring large territories and certain animals with migratory habits are badly affected, leading to population declines.

27. Option (4) is correct. Explanation : The World Summit on Sustainable Development took place in Johannesberg, South Africa.

28. Option(4) is correct. Explanation : RNA was the first genetic material. There is now enough evidence to suggest that essential life processes (e.g., metabolism, translation and splicing), evolved from RNA. RNA is used to act as a genetic material as well as a catalyst (there are some important biochemical reactions in living systems that are catalysed by RNA catalysts and not by protein enzymes).

But, RNA being a catalyst was reactive and hence unstable. Therefore, DNA has evolved from RNA with chemical modifications that make it more stable. DNA being doublestranded and having complementary strand, further resists changes by evolving a process of repair.

29. Option (3) is correct. Explanation : Sacred grooves are method for in situ conservation of biodiversity and conserving rare threatened species.

30. Option (2) is correct Explanation : The major source of noise pollution, worldwide is due to transport system, that is, transport vehicles (both public and private). The other sources of noise pollution are various industries like textile, printing, sugar, engineering, and agricultural implements.

31. Option (3) is correct. Explanation : Transformation of the blastocyst into gastrula with primary germ layers by rearrangement of the cells is called gastrulation. During gastrulation, the embryo develops three germ layers (endoderm, mesoderm, and ectoderm).

32. Option (1) is correct. Explanation : 5'-ATGAATG-3' (coding strand) 5'-TACTTAC-3' (complementary strand) 5'-AUGAAUG-3' (RNA)

33. Option (2) is correct Explanation : Compressed natural gas (CNG) is the best and cheap fuel which is used to power vehicles, mostly buses and trucks. It has replaced petrol and diesel fuel as it is considered as clean fuel. It consists of around 90% methane, by compressing it to less than 1% of volume it occupies at standard atmospheric pressure. Propane and butane together form LPG or liquefied petroleum gas while ethane is used in chemical industry to produce ethene.

34. Option (3) is correct. Explanation : Salmonella typhi is a pathogenic bacterium that causes typhoid fever in human beings.

Solutions 35. Option (4) is correct. Explanation : Statin is produced by a yeast called Monascus purpureus. It is a blood cholesterol lowering agent.

36. Option (2) is correct Explanation : Secretion of seminal vesicle (paired), prostate gland (unpaired) and bulbo-urethral glands or Cowper’s glands (paired) constitute the seminal plasma. It contains various proteins and fructose as energy suppliers for sperm motility and is also responsible for making the largest proportion of the alkaline buffer.

37. Option (1) is correct Explanation : Humus is organic material formed by the decomposition of leaves. Microorganisms releases inorganic nutrients from humus by the process of mineralisation.

38. Option (1) is correct Explanation : Emergency contraceptives are effective if they are used within 72 hours of coitus. They can prevent unwanted pregnancies after unprotected sex, failed used of contraceptive devices or due to rape, etc. Two types of emergency contraceptives include emergencies pills or IUDs.

39. Option (1) is correct. Explanation : If both parents have sickle cell trait, there is a 25% chance with each pregnancy that the baby will have sickle cell anaemia. A child with sickle cell anaemia appears normal at birth.

40. Option (3) is correct. Explanation : Biogas (commonly called gobar gas) is produced by using cattle dung. The technology of biogas production from cow dung was developed in India mainly due to the efforts of Indian Agricultural Research Institute (IARI) and Khadi and Village Industries Commission (KVIC).

41. Option (3) is correct Explanation : Polyspermy may be defined as the fertilisation of an ovum by more than one sperm. During fertilisation, a sperm comes in contact with the zona pellucida layer of the ovum and induces changes in the membrane that block the entry of additional sperms. Thus, it ensures that only one sperm can fertilise an ovum.

51

42. Option (3) is correct Explanation : When evaporation exceeds precipitation, the soil moisture supporting vegetation will deplete and therefore growth of vegetation will cease. Hence, desert ecosystem, where vegetation is almost nil, is expected in an area where evaporation exceeds precipitation, and mean annual rainfall is below 100 mm.

43. Option (2) is correct Explanation : Sexually transmitted diseases are transmitted through sexual intercourse. They are also called as venereal diseases (VDs) or reproductive tract infections (RTI). Examples of STDs are syphilis, gonorrhoea, genital warts, trichomoniasis, AIDs, etc. In the above-mentioned diseases, AIDS does not affect any sex organ. It is a set of symptoms or illness that arises as a result of advanced HIV infection which has destroyed the immune system.

44. Option (3) is correct. Explanation : When a purine base is replaced by another purine and pyrimidine by another pyrimidine, it is transition.

45. Option (4) is correct. Explanation : Methanogenic bacteria are anaerobic microorganisms that grow in the presence of carbon dioxide and produce methane gas. These bacteria are not found in activated sludge. The microbes present in the activated sludge are aerobic bacteria that grow rapidly and form flocs.

46. Option (2) is correct. Explanation : The lac operon regulation can be in both negative and positive ways. It is a negative control system because expression is typically blocked by an active repressor (the lac repressor) that turns off transcription. And when CAP (catabolite gene activating protein) binds upstream of this operator region near the promoter and transcription increases, this is an example of a positive system.

47. Option (3) is correct. Explanation : The lac operon consists of 3 structural genes, and a promoter, a terminator, regulator, and an operator. The three structural genes are: lacZ, lacY, and lacA.

48. Option (2) is correct. Explanation : The messenger RNA produced by transcription carries information for the synthesis of all three proteins found in all three structural genes. Hence, it is a polycistronic messenger RNA.

52

OSWAAL CUET (UG) Sample Question Papers, BIOLOGY

49. Option (2) is correct. Explanation : Promoter helps in starting the process of transcription and provides a binding site to RNA polymerase.

50. Option (1) is correct. Explanation : The lac operon of E. coli contains genes involved in lactose metabolism. It's expressed only when lactose is present and glucose is absent.

qqq

SOLUTIONS OF Question Paper BIOLOGY

1. Option (3) is correct.

3. Option (3) is correct.

Explanation : In both types of reproduction, transfer of genetic materials takes place from one generation to another and also progeny have some resemblance with their parents. Gametic fusion (fertilisation of sperm and ovum to form zygote) and reduction division (also called meiosis) is present only in sexual mode of reproduction.

2. Option (4) is correct. Explanation : In a dihybrid cross, if you get 9 : 3 : 3 : 1 ratio, it denotes that alleles of two genes are segregating independently. Law of independent assortment states that “When two pairs of traits are combined in a hybrid, segregation of one pair of characters is independent of the other pair of characters”. Suppose crosses are made between a pea plant with round and yellow seeds and one with wrinkled and green ones. All F1 hybrids give yellow and round seeds. Since. yellow colour is dominant over the green and the round shape is dominant over the wrinkled. When the F1 hybrid plants are crossed to each other or allowed to self-fertilise, F2 generation form as represented in the following cross. P-generation

yyrr

YYRR

Yellow-round seed Gametes

F1-generation

YR

Green-wrinkled seed yr

YyRr

Yellow-round seed (dominant character) The F1 hybrid plants are allowed to self fertilise YyRr

4

YyRr

1 1 1 1 yr Gametes 1 YR 1 Yr 1 yR 1 yr 4 4 4 4 4 YR 4 Yr 4 yR 4 F2-generation yR yr YR Yr YR YYRR YYRr YyRR YyRr Yr YYRr YYrr YyRr Yyrr yR YyRR YyRr yyRR yyRr yr YyRr Yyrr yyRr yyrr Phenotypic ratio-9 : 3 : 3 : 1

The outcome of the dihybrid cross makes it clear that segregation of the seed colour is independent of the seed shape and both the parental and new combinations of the characters appear in the offspring, that is, assortment of genes of one pair is independent of the other pair.

Explanation : If any protein encoding gene is expressed in a heterologous host, it is called recombinant protein. The cells harbouring cloned genes of interest may be grown on a small scale in the laboratory. The cultures may be used for extracting the desired protein and then purifying it by using different separation techniques. The cells can also be multiplied in a continuous culture system where the used medium is drained out from one side while fresh medium is added from the other to maintain the cells in their physiologically most active log/exponential phase. This type of culturing method produces a larger biomass leading to higher yields of desired protein.

4. Option (3) is correct. Explanation : Population growth is determined by addition and loss of individual in a population. Population density is the number of individuals present per unit volume/area at given time. Thus, mortality and emigration would necessarily decrease the density of a population in a given habitat due to loss of individual resulting from mortality (deaths) and emigration. When natality rate will be more than mortality rate and increased immigration rate would necessarily increase the density of a population.

5. Option (1) is correct. Explanation : Clone cannot be applied to offsprings produced from sexual reproduction because sexual reproduction involves the fusion of male and female gametes from two parents. Therefore, the offsprings produced from them are not identical to the parents. This genetic recombination leads to variations which play an important role in evolution. Whereas, offsprings produced as a result of asexual reproduction are identical to each other, both physically as well as genetically. They are the exact copies of their parents. Hence, they are called clones.

54

OSWAAL CUET (UG) Sample Question Papers, BIOLOGY

6. Option (2) is correct. Explanation : According to law of segregation or law of purity of gametes, factors or alleles of a pair segregate from each other during gamete formation, such that a gamete receives only one of the two factors and do not show any blending. Therefore, the proposition made by Mendel that the factor controlling any character is discrete and independent and based on the observations that the offspring of a cross made between the plants having two contrasting characters shows only one character without any blending. Suppose, when two contrasting characters (for e.g., seed colour and seed shape - round yellow and wrinkled green, given below) of a plant are crossed, their dominant phenotypic traits were expressed in F1 generation and their recessive characters appear in the F2 generation. This shows that there is no blending of Mendelian factors in F1 generation but they stay together and only one of the character is expressed. Therefore, at the time of formation of gametes, these two factors separate or segregate otherwise recessive types will not appear in F2 generation.

8. Option (2) is correct. Explanation : In a DNA strand, the nucleotides are linked together by 3’→ 5’ phosphodiester linkage (bonds) to form a dinucleotide. More nucleotides can be joined in such a manner to form a polynucleotide chain.

9. Option (2) is correct. Explanation : Foot prints (pug marks) and faecal pellets of conserved animals are some parameters which can be used for tiger census in our country’s National park and sanctuaries.

10. Option (1) is correct. Explanation : In a fertilised embryo sac, the haploid, diploid and triploid structures are synergids, zygote and primary endosperm nucleus respectively.

11. Option (1) is correct. Explanation : In certain taxon of insects, 17 and 18 chromosome bearing organisms are males and females respectively. Because, insects have XO type of sex determination method. In certain insects, such as cockroach, and some roundworms lacks Y-chromosome, so that the male has only one sex chromosome, that is, ‘X’ besides autosomes. This condition in the male is designated as XO (where O means absence of one sex chromosome) and in the female, it is XX.

12. Option (1) is correct.

7. Option (2) is correct. Explanation : Selectable markers help in identifying and eliminating non-transformants and selectively permitting the growth of the transformants. The normal E. coli cells do not carry resistance against any antibiotic. Competent bacterial cells are made capable to take foreign DNA with chemical treatment (e.g., calcium chloride).

Explanation : Transduction is the process by which genetic material (DNA) is transferred from one bacterium to another through the mediation of a vector, like virus. Bacterial conjugation is the process of transfer of genetic material (plasmid) between bacterial cells by direct cell-to-cell contact or by a bridge-like connection between two cells. Transformation is the genetic alteration of a cell resulting from the direct uptake and incorporation of exogenous genetic material (exogenous DNA) from its surroundings and taken up through the cell membranes. Translation is the process in which cellular ribosomes create proteins. It is a part of the process of gene expression.

13. Option (2) is correct. Explanation : In a terrestrial ecosystem, biomass of producers is high while biomass decreases at next trophic levels and it is lowest at top level. Therefore, pyramid of biomass is upright.

Solutions Contrary to this, the pyramid of biomass in a sea or a lake is generally inverted because the biomass of fishes (consumers) exceeds that of phytoplankton (producers).

14. Option (4) is correct. Explanation : The outermost and innermost wall layers of microsporangium in an anther are respectively, epidermis and tapetum. A typical microsporangium is generally surrounded by four-wall layers, that is, the epidermis, (outermost protective layer), endothecia, (middle fibrous layers) and the tapetum (innermost nutritive layer).

15. Option (2) is correct. Explanation : Climax community is the topmost and stable species suited for the physical environment of an ecological succession. Therefore, climax community is in a state of equilibrium.

16. Option (2) is correct. Explanation : In eukaryotes, the monocistronic structural genes have interrupted coding sequences, that is, the genes in eukaryotes are split. The coding sequences or expressed sequences are defined as exons. These sequences (exons) appear in mature or processed RNA, thus exons are interrupted by introns or intervening sequences which do not appear in mature or processed RNA.

17. Option (1) is correct. Explanation : All of the given features are important to facilitate cloning into a vector but out of them, origin of replication (ori) is the most important feature in a plasmid to be used as a vector. This is due to the following reasons: (i) It is a DNA sequence that is responsible for initiating replication. Any piece of DNA when linked to this sequence can replicate within the host cells. (ii) It also controls the copy numbers of the linked DNA.

55

18. Option (1) is correct. Explanation : Brazil is the country with the greatest biodiversity of flora and fauna on the planet. It has the highest number of species of known mammals and fresh-water fish, and more than 50,000 species of trees and bushes. It takes first place in plant diversity. he climate of Brazil remains relatively T undisturbed, constant and predictable giving tune for diversification, which favours rich biodiversity.

19. Option (2) is correct. Explanation : Apomixis refers to the formation of seeds without fertilization. The embryos are genetically identical to the parental plant.

20. Option (3) is correct. Explanation : Insulin and C-peptide are linked when first made by the pancreas. C-peptide is removed from the pancreatic beta-cells during cleavage of insulin from pro-insulin.

21. Option (2) is correct. Explanation : Exotic or alien species may cause the disappearance of the native species through changed biotic interactions. Cynodon (doob grass) is not an invasive alien species. It is a major tropical grass which is found in all tropical and subtropical areas. It is highly tolerant to drought and heavy grazing, and therefore, extremely valuable for pasture. The other three Parthenium (Congress or carrot grass), Eichhornia (water hyacinth) and Lantana are alien species which pose threat to native species.

22. Option (4) is correct. Explanation : The synergids have a filiform apparatus which have an important role in guiding the pollen tube into the synergid.

23. Option (2) is correct. Explanation : According to Chargaff ’s rules of base pairing, (i) The amount of adenine is always equal to the amount of thymine and the amount of guanine is always equal to the amount of cytosine. (ii) Adenine is joined to thymine with two hydrogen bonds and guanine is joined to cytosine by three hydrogen bonds. Diagram showing essential features of plasmid pBR322

56

OSWAAL CUET (UG) Sample Question Papers, BIOLOGY (iii) The ratio of adenine to thymine and that of guanine to cytosine is always equal to A G one, that is., = = 1. T C In the given organism, the DNA is not following the Chargaff’s rule, hence it can be concluded that it is a single-stranded DNA, not double-stranded.

24. Option (2) is correct. Explanation : The vas deferens receives duct from the seminal vesicle and opens into urethra as ejaculatory duct. This ejaculatory duct which open into the urethra about half-way through the prostate gland, function to mix the sperm stored in the ampulla with fluids secreted by the seminal vesicles and to transport these substances to the prostate.

25. Option (4) is correct. Explanation : Matthew Meselson and Franklin Stahl in 1958 performed experiments on E. coli to prove that DNA replication is semiconservative but it had no contribution in the development of double helix model. In 1953, James Watson and Francis Crick, based on the X-ray diffraction data, produced by Maurice Wilkins and Rosalind Franklin, proposed a very simple but famous double helix model for the structure of DNA. Erwin Chargaff observed that for a double-stranded DNA, the ratios between adenine and thymine and guanine and cytosine are constant and equal to one.

26. Option (3) is correct. Explanation : Amphibians have the highest percentage presently, 32% of all the amphibian species in the world face the threat of extinction. Other than these, 23% of all mammal species and 12% of all bird species also facing the risk.

27. Option (3) is correct. Explanation : Amazon rain forest is known as the lungs of the planet Earth because they absorb large amount of carbon dioxide and are the main source of oxygen and living being.

28. Option (1) is correct. Explanation : Fimbriae, infundibulum, isthmus and ampulla are the parts of Fallopian duct, while labia minora is female external genitalia.

29. Option (1) is correct. Explanation : In E. coli, the lac operon gets switched on when lactose is present and it binds to the repressor.

Lactose acts as an inducer which binds to the repressor protein and forms an inactive repressor. The repressor fails to bind to the operator region. The RNA polymerase binds to the operator and transcript lac mRNA. Lac mRNA is poly-cistronic, that is, produces all three enzymes, beta-galactosidase, permease and trans-acetylase. In the presence of lactose, the lac repressor is released from the operator, and transcription proceeds at a slow rate. promoter operator lacZ lacY lacA RNA polymerase

repressor

lactose

In the absence of lactose, lac operon is switched off. When lactose is absent, i-gene regulates and produces repressor mRNA which translate repression. Then, the repressor protein binds to the operator region of the operon and as a result prevents RNA polymerase to bind to the operon. In the absence of lactose, the lac repressor binds the operator, and transcription is blocked. promoter operator lacZ lacY lacA repressor × RNA polymerase



30. Option (3) is correct. Explanation : Diphtheria is a serious bacterial infection that affects the mucous membranes of the throat and nose. Although, it spreads easily from one person to another, diphtheria can be prevented through the use of vaccines. Leprosy is a chronic, progressive bacterial infection caused by the bacterium Mycobacterium leprae. It primarily affects the nerves of the extremities, the skin, the lining of the nose, and the upper respiratory tract. The plague is a serious bacterial infection that can be deadly. Sometimes referred to as the “black plague,” the disease is caused by a bacterial strain called Yersinia pestis. This bacterium is found in animals throughout the world and is usually transmitted to humans through fleas.

31. Option (3) is correct Explanation : Golden Rice is a new type of rice that contains beta-carotene (provitamin A), which is converted into vitamin A as needed by the body and gives the grain its golden colour. It is developed through genetic engineering and produces two new enzymes that complete the beta-carotene expression in the rice grain.

Solutions 32. Option (2) is correct. Explanation : Both IMR (infant mortality rate) and MMR (maternal mortality rate) are responsible for affecting the growth rate inversely. It means, decline in IMR as well as MMR will result in high population growth and vice-versa. Hence, if there is an increased IMR and decreased MMR in a population, mothers are there to give births to infants whereas, the survival rate of infants becomes low. Therefore, in a particular population, increased IMR and decreased MMR will hinder their growth rate.

33. Option (3) is correct. Explanation : In humans, chromosome 1 has highest genes (~2,968) and the Y has the fewest (~231) genes.

34. Option (2) is correct. Explanation : Promoter helps in starting the process of transcription and provides a binding site to RNA polymerase.

35. Option (3) is correct. Explanation : The Indian Government has set up organisations such as GEAC (Genetic Engineering Approval Committee), which will make decisions regarding the validity of GM research and the safety of introducing GMorganisms for public services.

36. Option (3) is correct. Explanation : Medical termination of pregnancy is also called induced abortion. It cannot be used as a contraceptive method. MTP is done to get rid from unwanted pregnancies which arises due to rape, genetic abnormalities in child, or when mother has some risks by the developing child. MTP is safe up to first trimester of pregnancy, that is, 12 weeks. MTP done is in the supervision of medical practitioner under some law. MTPs are not always surgical as there are certain pills available which will help to abort the foetus. These pills act by inducing menstruation, which in turn, checks implantation of the zygote or detaches the implanted foetus.

37. Option (3) is correct. Explanation : Viviparity is considered to be more evolved because the developed young ones are protected inside the mother’s body and are looked after they are born leading to more chances of survival. These young ones are capable of independent existence (e.g., mammals).

57

38. Option (1) is correct. Explanation : When opioids bind to specific receptors present in our CNS, they block the pain messages sent from the body through the spinal cord to the brain.

39. Option (1) is correct. Explanation : Totipotency is the ability of a cell single cell to produce any of the different types of cells in the body. These cells are found in spores and zygote.

40. Option (1) is correct. Explanation : A person with first stage of syphilis generally has a sore or sores at the original site of infection. These sores usually occur on or around the genitals. These sores are usually (but not always) firm, round, and painless. Symptoms of secondary syphilis include skin rash, swollen lymph nodes, and fever. Tertiary syphilis is associated with severe medical problems. It can affect the heart, brain, and other organs of the body.

41. Option (3) is correct. Explanation : The most accepted line of descent in human evolution is - Ramapithecus → Homo habilis → Homo erectus → Homo sapiens.

42. Option (1) is correct. Explanation : Cytosine (C) is a pyrimidine i.e., a nitrogenous base. A combination of a nitrogenous base (purine or pyrimidine) with a pentose sugar is known as nucleoside. Thus, the combination of cytosine with ribose sugar results in the formation of a nucleoside called as cytidine.

43. Option (3) is correct. Explanation : Biogas is composed of Methane(50-70%), CO2(30-40%) with traces of hydrogen sulphide and hydrogen.

44. Option (3) is correct. Explanation : DNA replication starts with unwinding of DNA duplexes which are held together by hydrogen bond. Helicases move along the double stranded DNA and separate the strands by breaking hydrogen bonds between base pairs.

45. Option (3) is correct. Explanation : Dobson unit is used to measure the total amount of ozone present in the atmosphere with the help of photoelectric spectrophotometer (also called Dobson meter). Dobson (1889-1976) invented the photoelectric spectrophotometer in 1928. On the other hand, oxygen sensors are used to measure the exhaust gas concentration of oxygen.

58

OSWAAL CUET (UG) Sample Question Papers, BIOLOGY

46. Option (3) is correct. Explanation : The approach to biological control includes Preservation of natural enemies(predators & parasites) that are already established in an area.

47. Option (2) is correct. Explanation : Malaria and yellow fever are less likely to occur in a region with plenty of dragonflies because dragonflies eat mosquito which spreads such diseases thereby decreasing their occurence.

48. Option (3) is correct. Explanation : Adult dragonflies are beneficial because they eat pest, flying insects, particularly midges and mosquitoes and help to decrease the probability of diseases spread by vectors.

49. Option (3) is correct. Explanation : Dragonflies are large and heavybodied insects. The larvae (or Nymphs), which

live in water, eat almost anything living that is smaller than themselves. The adult dragonfly larvae are even known to catch and eat small fish. Usually, they eat bloodworms or other aquatic insect larvae. Dragonfly nymphs are such good hunters because they move really fast - by what looks like jet propulsion. One of the benefits of having dragonflies live nearby is that, they eat many times their weight in mosquitoes every day. Dragonflies also eat many different kinds of flies. Using a basket-like arrangement of their legs, adult dragonflies can actually catch and eat other bugs while they are flying.

50. Option (3) is correct. Explanation : Dragonflies have role in controlling the mosquitoes population and ultimately malarial disease as they are predators on them but they have no role to allow mosquitoes to reproduce.

qqq

SOLUTIONS OF Question Paper 4. Option (2) is correct.

1. Option (3) is correct. Explanation : Col- Column umn I II A

(iii)

Explanation

Snow blindness is the inflammation and photophobia caused by exposure of the eyes to ultraviolet rays) reflected from snow or ice.

B

(iv)

Biological oxygen demand (BOD) is related to biodegradable organic matter. It is the amount of dissolved oxygen that must be present in water in order to decompose the organic matter in the water, used as a measure of the degree of pollution.

C

(i)

DDT is a colourless odourless substance which is used as an insecticide. It is toxic to animals and is known to accumulate in the tissues in a process called bio-magnification.

D

(ii)

5

Eutrophication is a process by which a body of water becomes enriched in dissolved nutrients (such as phosphates) that stimulate the growth of aquatic plant life usually resulting in the depletion of dissolved oxygen.

2. Option (2) is correct. Explanation : Restriction endonucleases naturally target double stranded DNA and could cleave one or both the strands of the DNA.

3. Option (2) is correct. Explanation : Trichoderma is a free-living fungus which is common in soil and root ecosystems. It is an effective biocontrol agent and used extensively for soil-borne diseases. It has been also used successfully against many pathogenic fungi which belong to various genera, viz. Fusarium, Phytophthora and Scelerotia. It may suppress the growth of the disease-causing organisms in the rhizosphere through competition and thus reduce disease development. It produces antibiotics and toxins, such as trichothecin and a sesquiterpine, and trichodermin, which have a direct effect on other organisms.

Explanation : The unequivocal proof that DNA is the genetic material came from the experiments of Alfred Hershey and Martha Chase (1952). They worked with viruses that infect bacteria called bacteriophages.

5. Option (4) is correct. Explanation : Bio-magnification (also called as bioaccumulation) is the accumulation of nonbiodegradable toxic materials like mercury (Hg), DDT, etc., in different trophic levels. In the process of bio-magnification, the concentration of non-biodegradable pollutants increases at each successive trophic level of the food chain, thus, harming the environment at an alarming rate, whereas SO2(sulphur dioxide) is an air pollutant and acts as precursor of acid rain.

6. Option (2) is correct. Explanation : Restriction enzymes are the molecular scissors that cuts the DNA from specific recognition site.

7. Option(1) is correct. Explanation : Bacillus thuringiensis is to control butterfly caterpillar. These are available in sachets as dried spores which are mixed with water and sprayed on to vulnerable plants such as brassica and fruit trees, where these are eaten by the insect larvae. In the gut of the larvae, the toxin is released and the larvae get killed.

8. Option (3) is correct. Explanation : The RNA polymerase holoenzyme transcribes the structural gene and the terminator regions. RNA polymerase consists of a number of sub-units, including a sigma factor (transcription factor) that catalyses the process of transcription. It recognises the start signals or promoter region on DNA which then along with RNA polymerase binds to promoter to initiate the transcription. In eukaryotes, there are three RNA polymerases : I, II and III. The process includes a proof-reading mechanism.

9. Option (4) is correct.

60

OSWAAL CUET (UG) Sample Question Papers, BIOLOGY Explanation : Biodiversity hot-spots specifically refer to 25 biologically rich areas around the world that have lost at least 70% of their original habitat. They are characterised by the large number of flora and fauna, abundance of endemic species and also large number of alien or exotic species. They are mostly found in tropical and temperate regions. There are no biodiversity hot-spots in polar regions.

10. Option (2) is correct. Explanation : Activated sludge microorganisms need oxygen as they oxidise wastes to obtain energy for growth. Insufficient oxygen will slow down or kill off aerobic micro-organisms, make facultative organisms work less efficiently and ultimately lead to the breakage of flocs. It will also result in the production of the foul-smelling byproducts of anaerobic decomposition. So, sufficient oxygen must always be sustained in the aeration tank to ensure complete waste stabilisation.

11. Option (2) is correct. Explanation : Three codons UAG, UAA and UGA are the stop or termination codons.

12. Option (4) is correct. Explanation : Sickle-cell anaemia is a recessive autosomal gene disorder. This disease is controlled by a single pair of allele HbA and HbS. It is caused due to inheritance of a defective allele coding for beta globulin. It results in the transformation of HbA into HbS in which glutamic acid (Glu) is replaced by valine (Val) at sixth position in each of the two beta-chains of haemoglobin. This substitution occurs due to the single base substitution of the beta-globin gene from GAG (Glu) to GUG (Val). Whereas, the other codes GGG, AAG. GAA do not code for valine.

13. Option (4) is correct. Explanation : All of the above-mentioned plants are endangered plant species of India. Rauwolfia serpentina (sarpgandha), Santalum album (sandal wood) and Cycas beddonei are facing the threat of extinction due their medicinal and commercial importance.

14. Option (3) is correct. Explanation : Cocaine attaches to the dopamine transporter and blocks the normal recycling process, resulting in a buildup of dopamine in the synapse, which contributes to the pleasurable effects of cocaine.

15. Option (1) is correct.

Explanation : The segment of DNA that takes part in transcription is called transcription unit. It has three components (i) a promoter (ii) the structural gene and (iii) a terminator.

16. Option (3) is correct. Explanation : Plants use two abiotic (wind and water) and one biotic (animals) agent to achieve pollination. Majority of plants use biotic agents for pollination. Pollination by wind is more common amongst abiotic pollination. It requires the light and non-sticky pollen grains so that, they can be transported in wind currents. They often possess wellexposed stamens (so that the pollens are easily dispersed into wind currents) and large often feathery stigma to easily trap air-borne pollen grains. Wind pollination is common in grasses. Pollination by water is called hydrophily which is quite rare in flowering plants but occurs in aquatic plants. Zoophily is pollination through the agency of animals. Entomophily (pollination by insects) is the most common type of zoophily which occurs through the agency of animals.

17. Option (1) is correct. Explanation : The process of decomposition occurs at faster rate in warm and moist environment. Tropical rain forests favour these conditions and hence the process of decomposition would be the fastest in tropical rain forest.

18. Option (4) is correct. Explanation : Ringworm infections are caused by fungi belonging to the genera Microsporum, Trichophyton, and Epidermophyton. Macrosporum is an ecto-mycorrhizal zoosporic fungus causing diseases of economically important vascular plants.

19. Option (3) is correct. Explanation : Pleiotropy is a phenomenon in which a single gene express multiple effects. Sometimes, one trait will be very evident and others will be less evident, e.g., a gene for white eye in Drosophila also affect the shape of organs in male responsible for sperm storage as well as other structures. Similarly, sickle-cell contrasting characters controlled by multiple alleles at a single genetic locus, e.g., ABO blood group. Mosaicism describes the occurrence of cells that differ in their genetic component from other cells of the body. Polygeny refers to a single characteristic that is controlled by more than two genes. (it is also known as multifactorial inheritance).

Solutions 20. Option (4) is correct. Explanation : The littoral zone is the zone at the edge of a lake or ocean or in aquatic habitat which is alternatively exposed to air. Therefore, light is also available and immersed in water. This coastal zone is rich producers from surface to bottom, with a large number of brown and red algae attached at the zone. There are different zones in an aquatic ecosystem. Pelagic zone:-Any zone of water in a sea or lake that is neither close to bottom nor the shore is pelagic zone. Benthic zone : It is the bottom part of the water body (aphotic zone), light is not available here and it is dark. Microorganisms and benthic organisms are found in this zone. Lentic zone : It refers to standing or relatively still water in an aquatic ecosystem.

21. Option (1) is correct. Explanation : Associated lymphoid tissue (MALT) constitutes about 50% of the lymphoid tissue in human body.

22. Option (2) is correct. Explanation : Nullisomy - the loss of both pairs of homologous chromosomes; Such individuals are called nullisomic and their chromosomal composition is 2N-2.

23. Option (2) is correct. Explanation : Insect pollinated plants provide rewards such as nectar and pollen grains to sustain insect visits. In some species, floral rewards are in providing safe places to lay eggs.

Explanation : Binary fission is a mode of asexual reproduction in protozoan through which it produces two offsprings from parent individual. So, In 1st generation : there are 2 protozoans. In 2nd generation : there are 2 * 2 = 4 protozoans In 3rd generation : there are 4*2 = 8 protozoans In 4th generation 4  : there are total 16 protozoans. In 5th generation : there are total 32 protozoans In 6th generation : there are 64 protozoans Thus, the population of protozoan will be 64, after six generations.

27. Option (3) is correct. Explanation : The diameter of the strand is always constant due to a pairing of purine (adenine and guanine) and pyrimidine (cytosine and thymine). This specific bonding gives uniform width to the DNA.

28. Option (2) is correct. Explanation : Agriculture accounts for approximately 33% of India’s GDP (gross domestic product).

29. Option (3) is correct. Scutellum Plumule Coleoptile Cotyledons Hypocotyl

25. Option (3) is correct. Explanation : Change in sequence of nucleotide in DNA is known as mutation. A mutation involves a change in the sequence of nucleotides in a nucleic acid molecule.

26. Option (3) is correct.

Shoot apex Epiblast

Radicle Root cap

24. Option (4) is correct. Explanation : Polypeptide synthesis is signalled by two initiation codons - commonly AUG or methionine codon and rarely GUG or valine codon. AUG serves two main functions. It signals the start of translation and codes for the incorporation of the methionine into the growing polypeptide chain. AUG codes for methionine in both prokaryotes and eukaryotes.

61

Radicle Root cap Coleorhiza

Explanation : A typical dicotyledonous embryo consists of two cotyledons. While, embryos of monocotyledons possess only one cotyledon and it is called scutellum (in grass). Cotyledons of dicots, are simple structures generally thick and swollen due to storage of food reserves (as in legumes) and embryo of monocots consists of one large and shield shaped cotyledon known as scutellum situated towards one side (lateral) of the embryonal axis.

30. Option (1) is correct. Explanation : Sandal wood tree (Santalum album) is an example of partial root parasite. It grows on the roots of Dalbergia, Albizzia and Eucalyptus. The sucking roots of the plants attack the roots of the host trees and from them nutrients are absorbed.

62

OSWAAL CUET (UG) Sample Question Papers, BIOLOGY Mistletoe (Viscum) is considered as hemiparasite which derives a part of nourishment from host plant. Orobanche is an obligate holoparasite which attack strategic food crops, such as legumes and vegetables, Ganoderma mushrooms are known as king of herbs for its wonderful medicinal properties. They are commonly known as reishi mushroom. They are widely distributed, shelf like or knob-like fungi that feed either as saprobes on dead wood or as parasites on the live wood of hardwood trees.

31. Option (1) is correct. Explanation : The biggest constraint of the plant breeding is the availability of limited number of disease resistance genes that are present and identified in various crop varieties or wild relatives.

32. Option (3) is correct. Explanation : (1) Spermatogonia have 46 chromosomes and always undergo mitotic cell division. (2) Primary spermatocytes divide by meiotic cell division. (3) Spermatids are transformed into spermatozoa.

33. Option (3) is correct. Explanation : Polyspermy may be defined as the fertilisation of an ovum by more than one sperm. During fertilisation, a sperm comes in contact with the zona pellucida layer of the ovum and induces changes in the membrane that block the entry of additional sperms. Thus, it ensures that only one sperm can fertilise an ovum.

34. Option (2) is correct. Explanation : The reservoir for the gaseous type of bio-geochemical cycle exists in the atmosphere.

35. Option (1) is correct. Explanation : Sonalika and Kalyan Sona are varieties of wheat. Sonalika is developed from high yielding, semi-dwarf, fertiliser-responsive wheat variety. They were developed in 1963 at Indian Agriculture Research Institute (IARI), New Delhi, as a part of systematic programme for breeding semi-dwarf wheat varieties.

36. Option (1) is correct. Explanation : Point mutations refer to changes in the sequence of DNA bases, and include substitutions, insertions, and deletions of one or more bases.

37. Option (1) is correct. Explanation : The corona radiata is thick and the outer layer of follicular (granulosa), zona pellucida is inner and thick and non cellular. The zona pellucida surrounds the oocyte and located between the oocyte and the follicular cells. The vitelline membrane is a membrane enclosing an egg that comprises the zona pellucida in mammals.

38. Option (1) is correct. Explanation : A national level approach to build up a reproductively healthy society was taken up in India in 1950s.

39. Option (4) is correct. Explanation : Phosphorus, nitrogen and sulphur bio-geochemical cycles does not have losses due to respiration.

40. Option (2) is correct. Explanation : Fallopian tube has three parts first is isthmus, second is ampulla (ampullary – isthmic junction) which is the most common site for fertilization and the third is infundibulum which is farthest from the uterus, and the first cleavage division of zygote occurs in the ampullary-isthmic junction.

41. Option (4) is correct. Explanation : There are different types of reproduction to produce new organisms, like asexual reproduction (in which only one parent is involved to produce offspring), and sexual reproduction (in which two different parents with different genetic compositions are involved). Due to living in large biological diversities, each organism undergoes different mechanisms to reproduce. They all depend on their habitat, physiological properties, genetic constitution and other factors to reproduce and produce new organism.

42. Option (2) is correct. Explanation : Micro-propagation is propagation of plants in-vitro (tissue culture) to achieve a large number of plants in very short durations. This results in genetically identical plants which are widely used in forestry and floriculture.

43. Option (2) is correct. Explanation : DNA fingerprinting involves identifying differences in some specific regions in DNA sequence called as respective DNA, because in these sequences, a small stretch of

Solutions DNA is repeated many times. These sequences normally do not code for any proteins, but they form a large portion of human genome. These sequence show high degree of polymorphism and form the basis of DNA fingerprinting. As the polymorphisms are inheritable from parents to children, DNA fingerprinting is the basis of paternity testing in case of disputes.

44. Option (4) is correct. Explanation : Haemophilia is a sex-linked recessive disorder in which X-chromosome has the haemophilic gene. A haemophilic father can never pass the gene for haemophilia to his son.

45. Option (2) is correct. Explanation : Ectodermal cells form the central nervous system.

46. Option (4) is correct. Explanation : Bt cotton crops are fungal resistant, insect resistant and drought resistant.

47. Option (4) is correct.

63

Explanation : Bt cotton is a genetically modified organism which is pest resistant. It contains gene cry I Ac and cry II Ab of Bacillus thuringiensis. It is used to control lepidopterans, coleopterans and dipterans. Bt cotton can resist cotton bollworm and produce higher yields.

48. Option (4) is correct. Explanation : The advantage of GM plants are: they enhance nutritional value of food, are more tolerant to abiotic stresses, have helped to reduce post-harvest losses.

49. Option (3) is correct. Explanation : Bt is a microbe naturally found in soil. It makes proteins that are toxic to immature insects (larvae). There are many types of Bt and each target different insect groups.

50. Option (1) is correct. Explanation : Lepidopterans are an order of insects consisting of moths and butterflies. Some insects of this order can be killed by the bacteria Bacillus thuringiensis.

qqq

SOLUTIONS OF Question Paper 1. Option (4) is correct. Explanation : In grasshopper, male is heterogametic, i.e., XO (Gametes with X and gametes without X) and female is homogametic, i.e., XX (all gametes are with X-chromosomes). Eggs fertilised by sperms having an X-chromosome become females and those fertilised by sperms that do not have X-chromosome become males.

2. Option (3) is correct. Explanation : There is no natural death in single-celled organisms, like Amoeba and bacteria, because parental body is distributed amongst the offsprings. In Amoeba and bacteria, the parental body itself divides into two parts and becomes daughter cells. Then, each of these daughter cells rapidly grow into adult organisms. Therefore, the parent organism cannot be said to have died, it continues living as daughter cells.

3. Option (3) is correct. Explanation : Health may be defined as a person’s mental and physical condition. It is a state of complete physical, mental and social well-being and not merely the absence of disease or infirmity. When people are healthy, they are more efficient in their work. It also increases longevity of people and reduces infant and maternal mortality.

4. Option (3) is correct. Explanation : Retrovirus is commonly used as a vector for introducing a DNA fragment in human lymphocytes as in gene therapy of ADA deficiency.

5. Option (3) is correct. Explanation : The affected person inherited with one extra copy of 21st chromosome that forms trisomy condition.

6. Option (2) is correct. Explanation : Zoospores are microscopic motile asexual reproductive structures. They are produced by certain algae and some fungi. They move with the help flagella. Other common asexual reproductive structures are conidia (Penicillium), buds (Hydra) and gemmules (sponge).

6

7. Option(2) is correct. Explanation : The disease chikungunya is transmitted by Aedes mosquitoes (Aedes aegypti and Aedes albopictus). It is a viral disease caused by chikungunya virus (CHIKV). It causes fever and severe joint pain. Other symptoms include muscle pain, headache, nausea, fatigue and rash. Whereas housefly is the vector for cholera, cockroach transmit jaundice or yellow fever and is a carrier of food and water borne diseases. Female Anopheles is responsible for spreading malaria caused by Plasmodium species.

8. Option (2) is correct. Explanation : (a)  Rapid secretion of high level of oestrogen and LH triggers the ovulatory surge. (b) Oogenesis is initiated during the embryonic developmental stage when a couple of million gamete mother cell (oogonia) is formed within each foetal ovary (about third month of foetal ovary). Unlike sperm formation that starts at puberty, egg formation begins before birth. No more oogonia are formed and added after birth. (c) Sperms released from seminiferous tubules are poorly motile/non-motile. They undergo physical maturation in the head of epididymis and acquire increased motility and fertilising capacity. (d) Progesterone level is high during the postovulatory phase of menstrual cycle because after ovulation the remaining part of Graafian follicle becomes a temporary endocrine gland, corpus luteum. Corpus luteum secretes progesterone hormone to maintain the pregnancy.

9. Option (2) is correct. Explanation : The first clinical gene therapy was given in 1990 to a 4-year old girl with adenosine deaminase (ADA) deficiency. This enzyme is crucial for the immune system to function. The disorder is caused due to the deletion of the gene for adenosine deaminase. In some children, ADA deficiency can be cured by bone marrow transplantation. Bone marrow consists of lymphocytes which are a kind of WBCs. The enzyme ADA is present in lymphocytes.

10. Option (2) is correct. Explanation : Nullisomy - the loss of both pairs of homologous chromosomes; individuals are called nullisomic and their chromosomal composition is 2N - 2.

Solutions 11. Option (2) is correct. Explanation : Rheumatoid arthritis is an autoimmune disorder. It occurs when immune system mistakenly attacks own body’s tissues like, the synovium (the lining of the membranes that surround your joints). The resulting inflammation thickens the synovium, which can eventually destroy the cartilage and bone within the joint. The tendons and ligaments that hold the joint together weaken and stretch. Gradually, the joint loses its shape and alignment. Autoimmunity is an abnormal immune response in which the immune system of the body starts rejecting its own body cells or ‘selfcells and molecules. Sometimes, body loose its ability to differentiate between pathogen or foreign molecules from self-cell and attacks self-cells. This results in damage to the body.

12. Option (4) is correct. Explanation : First clinical gene therapy was given in the year 1990 to a 4-year old girl with ADA (Adenosine deaminase) deficiency. ADA deficiency causes SCID (Severe Combined Immuno Deficiency) in which B-lymphocytes and T-lymphocytes are not formed. The ADA gene provides instructions for producing the enzyme adenosine deaminase. This enzyme is produced in all cells, but the highest levels of adenosine deaminase occur in immune system cells called lymphocytes. The absence of ADA results in a dysfunctional immune system due to build-up of toxic metabolites.

13. Option (4) is correct. Explanation : ‘Continuity of germplasm’ theory was propounded by August weismann which explains that the characters which influence the germ cells get inherited.

14. Option (2) is correct. Explanation : An embryo sac is present at the centre of each ovule. The embryo sac consists of the gametophytic cell. The female gametophyte undergoes three mitotic cell divisions to form eight nucleate stages of the embryo sac. Among these, two cells at the center fused to form polar nuclei. Three cells move towards the top, called antipodals. The remaining three cells are forming egg apparatus with one egg and two synergids at the micropylar end. Thus, a typical angiosperm embryo sac at maturity consists of 7 cells with 8 nuclei.

15. Option (3) is correct. Explanation : Appendix is a vestigial organ (useless), not a lymphoid organ. It is muscular structure attached to the large intestine in the human body. It is an evolutionary remnant of a cecum that was once large and had vital digestive functions, which do not work the same way anymore.

65

Lymphoid organs are those organs where origin and/or maturation and proliferation of lymphocytes occur. Lymphoid organs are of two types : primary and secondary. Lymphoid Organs

Primary Lymphoid Organs It includes bone marrow and thymus where B-and T-cell lymphocytes mature and acquire their antigen-specific receptor.

Secondary Lymphoid Organs The organs where lymphocytes interact with the antigen and proliferate to become effector cells, e.g., spleen, lymph nodes, tonsils, Peyer’s patches of small intestine

16. Option (2) is correct. Explanation : Mammals have the lowest number of species i.e., 410 species in India.

17. Option (2) is correct. Explanation : Klinefelter's syndrome is the genetic disorder which is caused due to the presence of an additional copy of X-chromosome resulting in karyotype of 47, XXY chromosome but only the son will be affected by the disease.

18. Option (4) is correct. Explanation : For a double stranded DNA, the ratios between Adenine and Thymine and Guanine and Cytosine are constant and equals one is Chargaff ’s rule.

19. Option (4) is correct. Explanation : It is estimated that more than 70% of the world’s livestock population is in India and China.

20. Option (2) is correct. Mutations are random. Mutations in somatic tissues cannot be passed on to next generation, they must occur in germ cells. Darwin’s variations are small and directional, because with an accumulation of minor variations, speciation happens progressively over a number of generations.

21. Option (1) is correct. Explanation : The theory of chemical evolution was given and proved by Miller’s experiment.

22. Option (1) is correct. Explanation : The process of producing similar types of spores is called homosporous plants, as in pteridophytes. The process of producing two kinds of spores like macro and microspores, is called heterosporous plants. Genera like Selaginella and Salvinia produce two types of spores called microspores and macrospores.

66

OSWAAL CUET (UG) Sample Question Papers, BIOLOGY

23. Option (3) is correct. Explanation : In a forest ecosystem, light is an important abiotic component that controls a number of life processes in organism. In forest, its intensity duration and quality at ground is controlled by tall trees.

24. Option (3) is correct. Explanation : The diameter of the strand is always constant due to a pairing of purine (adenine and guanine) and pyrimidine (cytosine and thymine). This specific bonding gives uniform width to the DNA.

25. Option (1) is correct. Explanation : In false fruits, there is involvement of other parts along with ovary in formation of fruit.

26. Option (1) is correct. Explanation : The process of making recombinant DNA molecule involves the introduction of a desired gene into the DNA of a host that will produce the desired protein. Inducing a cloned eukaryotic gene to function in a prokaryotic host can be difficult sometime. The presence of long non-coding introns in eukaryotic genes may prevent correct expression of these genes in prokaryotes which lack RNA-splicing machinery.

27. Option (4) is correct. Explanation : The exine of the pollen grain is made up of sporopollenin which is one of the most resistant material known. It can withstand high temperatures and strong acids and alkali. No enzyme can degrade it.

28. Option (3) is correct. Explanation : Except black rot of crucifers, other three diseases are caused by fungi. Black rot of crucifer is a bacterial disease, caused by Xanthomonas campestris. It is considered as the most serious disease of crucifer crops worldwide. Rust of wheat is caused by a Puccinia, red rot of sugarcane is caused by Colletotrichum falcatum and smut of bajra is caused by Tolyposporium penicillariae.

29. Option (1) is correct. Explanation : A plasmid is a small, circular double-stranded DNA molecule that is separate from the main chromosome. It is found in bacteria and some yeast.

30. Option (4) is correct. Explanation : Polypeptide synthesis is signalled by two initiation codons - commonly AUG or methionine codon and rarely GUG or valine codon. AUG serves two main functions. It signals the start of translation and codes for the incorporation of the methionine into the growing polypeptide chain. AUG codes for methionine in both prokaryotes and eukaryotes.

31. Option (1) is correct Explanation : Parthenocarpy is the formation of seedless fruits without fertilization. The fruits developed from unfertilised ovary are called parthenocarpic fruits.

32. Option (3) is correct. Explanation : Lactic acid bacteria or Lactobacillus converts milk into curd. A small amount of curd, which contains millions of LAB, is added to the fresh milk as inoculum or starter. Then these bacteria multiply at suitable temperature and convert milk into curd, and thereby improve its nutritional quality by increasing the content of vitamin - B12.

33. Option (3) is correct. Explanation : In chemical method, the cell is treated with specific concentration of a divalent cation such as calcium which increase the pore size in cell wall. The cells are incubated with recombinant DNA on ice, followed by placing them briefly at 42°C and then putting it back on ice. This is called heat shock method. The bacteria now take up these recombinant DNA.

34. Option (2) is correct. Explanation : Beetles shows maximum diversity in species as it represent the largest order of insects.

35. Option (2) is correct. Explanation : The DNA-dependent RNA polymerase helps in transcription by catalysing the polymerisation in only one direction (i.e., 5'-3').

36. Option (2) is correct. Explanation : In some species, the diploid egg cell is formed without reduction division and develops into an embryo without fertilisation. It is an asexual reproduction which occurs in the absence of pollinators or in extreme environments.

Solutions In some species like citrus plants, nucellar cells surrounding the embryo sac start dividing and develop into embryos. It occurs in the megaspore mother cell without undergoing meiosis and produces diploid embryo sac through mitotic divisions. It helps in the preservation of desirable characters for indefinite period. Thus, it can be concluded that apomictic species produce diploid cells. Haploid cells will be formed during sexual reproduction when cell will undergo meiosis.

37. Option (1) is correct. Explanation : Xanthomonas campestris is a bacterial species that causes a variety of plant diseases.

38. Option (3) is correct. Explanation : Habitat destruction is the first major cause of species extinction.

39. Option (1) is correct. Explanation : The expected frequency can be solved by using (p+q)2 = p2 + 2pq + q2 = 1, which represents the Hardy-Weinberg principle. From the equation given above, MN = 2pq = 2 × 0.7 × 0.3 Thus, expected frequency of MN group = [(2 × 0.7 ×0.3)/1] × 100 = 42%.

40. Option (3) is correct. Explanation : Apiculture (also called bee keeping) is a process of rearing and management of honey bees for commercial production of the following : (a) Bees wax : It is used in industry for the preparation of cosmetics and polishes of various kinds. (b) Honey : It is a food of high nutritive value and also finds use in the indigenous systems of medicine. (c) Royal jelly : It is the extra-ordinary source of food for queen bees. It is rich in nutrition values and is believed to be a potent anti-oxidant. (d) Oil is not being considered as a product of apiculture.

41. Option (2) is correct. Explanation : Saccharomyces cerevisiae produces carbon dioxide (CO2) during the process of fermentation. It causes puffing up of the dough and make it soft and spongy. It is used to make foods like idli, dosa, bread, etc.

67

42. Option (3) is correct. Explanation : Antibiotic-resistant bacteria are the strains of bacteria that do not respond to antibiotics. Appearance of antibiotic-resistant bacteria is an example of pre-existing variation or mutation with in the bacterial population. When a bacterial population encounters a particular antibiotic, those sensitive to it die. But some bacteria having mutations become resistant to the antibiotic. Because if the bacteria have a mutation in the DNA which codes for one of those proteins, the antibiotic cannot bind to the altered protein; and the mutant bacteria survive. Soon, the resistance providing genes become widespread and the entire population becomes resistant. Adaptive radiation is development of different functional structures from a common ancestral which is also known as divergent evolution. Transduction is a process whereby foreign DNA is introduced into another cell via a viral vector.

43. Option (4) is correct. Explanation : Fungi form symbiotic association with the roots of higher plants called mycorrhiza (VAM), for example, Glomus. The fungal hyphae symbiont in these associations, absorb phosphorus from soil and passes it to the plant. Plants having such associations show other benefits also, such as resistance to root-borne pathogens, tolerance to salinity and drought, and an overall increase in plant growth and development. Mycorrhiza does not help the host plant in increasing its resistance to insects.

44. Option (1) is correct. Explanation : Sacred groves are highly protected by certain communities because they are of great religious importance to the communities. They have a significant role in in-situ conservation.

45. Option (3) is correct. Explanation : Gonorrhoea caused by Neisseria gonorrhoeae, Syphilis caused by Treponema pallidum, Genital herpes caused by HSV-2.

46. Option (4) is correct. Explanation : In logistic growth, population seldom grows beyond the carrying capacity of ecosystem.

47. Option (4) is correct. Explanation : If any species is flourishing under unlimited resources, it would reach exponential growth which can be depicted by equation: dN/dt =rN Where, N= population density at time t; r = intrinsic rate of natural increase.

68

OSWAAL CUET (UG) Sample Question Papers, BIOLOGY If we derive the integral form of the exponential growth equation, it can be written as: Nt =N0ert Where, Nt, = population density after time t; Nois population density at time zero; r = intrinsic rate of natural increase; e is the base of natural logarithm.

48. Option (1) is correct. Explanation : The equations correctly represents Verhulst-Pearl logistic growth is dN/ dt = rN(K-N)/K

49. Option (2) is correct. Explanation : 'r' represents Intrinsic rate of natural increase.

50. Option (3) is correct. Explanation : Since resources of growth for most animal populations are finite and become limiting sooner or later, so the logistic growth model is considered as more realistic.

qqq

SOLUTIONS OF Question Paper 1. Option (2) is correct. Explanation : Snails undergo aestivation to avoid summers. ‘Aestivation’ is known as summer’s sleep.

2. Option (3) is correct. Explanation : Innate immunity is non-specific type of defence, that is present at the time on birth.

3. Option (1) is correct. Explanation : DNA is the genetic material came from the experiments of Hershey and Chase (1952). They worked with viruses that infect bacteria called bacteriophages. They worked to discover whether it was protein or DNA from the virus that entered the bacteria. They grew some viruses on a medium that contained radioactive phosphorus and some others on a medium that contained sulphur. Viruses grown in the presence of radioactive phosphorus contained radioactive DNA but not radioactive protein because DNA contains phosphorus but protein does not. Similarly, viruses grown on radioactive sulphur contained radioactive protein but not radioactive DNA because DNA does not contain sulphur.

4. Option (3) is correct. Explanation : In both types of reproduction, transfer of genetic materials takes place from one generation to another and also progeny have some resemblance with their parents. Gametic fusion (fertilisation of sperm and ovum to form zygote) and reduction division (also called meiosis) are present only in sexual mode of reproduction.

5. Option (4) is correct. Explanation : Biosphere, one of the four layers that surround the Earth, is composed of all living organisms present on earth which interact with their physical environment. It is collectively used for all the ecosystems of world.

6. Option (1) is correct. Explanation : Marijuana is extracted from dried leaves and flowers of Cannabis sativa.

7. Option(3) is correct.

7

Explanation : DNA replication is said to be semi-conservative because of the process of replication, where the resulting double helix is composed of both an old strand and a new strand. The two resulting double helices, where each contain one "old" strand and one "new" strand of DNA, are identical to the initial double helix. Due to this reason, replication occurs continuously on one strand and discontinuously on the other strand. This is known as the semi-discontinuous mode of replication. Every new DNA molecule that is formed has a new and an old strand of the DNA. Thus, during DNA replication, entirely new DNA copies are not generated.

8. Option(3) is correct. Explanation : The bulbs are underground condensed shoots which have one or more buds that give rise to new plants. Onion is vegetatively propagated by bulbs.

9. Option (4) is correct. Explanation : Regulatory proteins, the accessory proteins that interact with RNA polymerase and affect its role in transcription. It controls the functions of structural genes and are called regulatory genes. Promoters, terminators, operators and repressor are some important regulatory genes. They can act both as activators and as repressors.

10. Option (3) is correct. Explanation : Binary fission is a mode of asexual reproduction in protozoan through which it produces two offsprings from parent individual. So, In 1st generation : there are 2 protozoans. In 2nd generation : there are 2 × 2 = 4 protozoans In 3rd generation : there are 4 × 2 = 8 protozoans In 4th generation : there are total 16 protozoans. In 5th generation : there are total 32 protozoans In 6th generation : there are 64 protozoans. Thus, the population of protozoan will be 64, after six generations.

11. Option (3) is correct.

70

OSWAAL CUET (UG) Sample Question Papers, BIOLOGY Explanation : Cocaine attaches to the dopamine transporter and blocks the normal recycling process, resulting in a build-up of dopamine in the synapse, which contributes to the pleasurable effects of cocaine.

12. Option (3) is correct. Explanation : Dioecious organisms are those organisms which have male and female reproductive organs in different individuals. These organisms are seen in both plants and animals. In dioecious angiosperms, male and female parts are found in separate flowers e.g., Mulberry. Hence, dioecious plants bear sexual dimorphism. Most animals are dioecious. Insects, spiders, fish, amphibians, reptilians, birds and mammals exhibit sexual dimorphism.

13. Option (1) is correct. Explanation : Sludge is a thick, soft and muddy deposits which can be treated by anaerobic digesters. Anaerobic digesters are large heated tanks in which micro-organisms break down biodegradable material in the absence of oxygen. It is then sent to oxidation ponds where heterotrophic bacteria continue the breakdown of the organic substances and solar ultraviolet (UV) light destroys the harmful bacteria. Chemicals are not used in this treatment.

14. Option (4) is correct. Explanation : In dioecious plants, the unisexual male flower is staminate, that is, bearing stamens only, while the female is pistillate or bearing pistils only. For the production of fruits and seeds fertilisation must take place, which is possible only in the presence of both male and female flowers. When the plant is dioecious, it will give rise to the following situations: If the plant is dioecious and bears only (i) pistillate flowers, fertilisation can take place with the help of pollinators. If the plant is dioecious and bears only (ii) staminate flowers, fertilisation cannot take place, because female gamete is non-motile which can’t reach the male gamete in order to fuse with it. When the plant is monoecious, that is, carrying both stamen and pistil together, it may lead to self-fertilisation and production of seed.

15. Option (4) is correct. Explanation : Parasitism is an interaction between two species in which one species (parasite) derives benefit while the other species (host) is harmed. For example, ticks and lice (parasites) present on the human body represent this interaction where in the parasites receive benefit (as they derive nourishment by feeding on the blood of humans). On the other hand, these parasites reduce host fitness and cause harm to the human body.

16. Option (3) is correct. Explanation : Autogamy is a method of selfpollination. It is a process in which the stigma of a flower receives pollens from the anther of same flower. For autogamy, both the sex organs of a chasmogamous flower should mature at the same time. As chasmogamous flowers open at maturity, pollen release and stigma receptivity should be synchronised for the process of autogamy. In such flowers, the length of anther and stigma plays secondary role in autogamy, e.g., in case of protandry (in which pollens mature early) and protogyny (in which stigma matures early) leads to crosspollination.

17. Option (3) is correct. Explanation : PCR techniques is helpful to detect very minute traces of virus or bacterial DNA and other multiple copies by DNA amplification.

18. Option (3) is correct. Explanation : Bone marrow and thymus are primary lymphoid organs as here immature lymphocytes differentiate into antigensensitive lymphocytes.

19. Option (1) is correct. Explanation : The juice which is prepared at home is turbid, due to the presence of fibres, pectin, etc. The bottled fruit juices are clearer because they are clarified by pectinases and proteases.

20. Option (1) is correct. Explanation : Pitcher plant (Nepenthes) is an insectivorous plant, found in rain forest of North-East India. They generally grow in nitrogen-deficient soil. Sundarbans are rich in fauna and flora. Mangrove plants are the characteristic of Sundarbans. Western Ghats are rich in biodiversity. Xerophytes are more common in Thar deserts.

21. Option (4) is correct. Explanation : Ribosomal RNA (rRNA), messenger RNA (mRNA) and transfer RNA (tRNA) are major classes of RNAs that are involved in gene expression. rRNAs bind protein molecules and give rise to ribosomes. mRNA carries coded information for translation into polypeptide formation. rRNA is also called soluble or adaptor RNA and carries amino acids to mRNA during protein synthesis.

22. Option (4) is correct. Explanation : Darwin published his observations and conclusions regarding evolution in the book “Origin of Species” in 1859.

Solutions 23. Option (2) is correct. Explanation : Megaspore mother cell divide meiotically to form four haploid megaspores which undergoes mitosis so that a haploid female gamete is formed. Antipodal cells are three in number. A large central cell is present in the embryo sac. The synergids bears filiform apparatus.

71

Extra nuclear genetic elements (also called as plasmids) shows the pattern of maternal inheritance.

24. Option (1) is correct. Explanation : Brazil is the country with the greatest biodiversity of flora and fauna on the planet. It has the highest number of species of known mammals and fresh-water fish, and more than 50,000 species of trees and bushes. It takes first place in plant diversity. The climate of Brazil remains relatively undisturbed, constant and predictable giving tune for diversification, which favours rich biodiversity.

25. Option (1) is correct. Explanation : Starch synthesis in pea seed is controlled by one gene which has two alleles (B and b). Pleiotropy is shown by B gene.

26. Option (4) is correct. Explanation : Rete testis, vasa efferentia and epididymis are parts of male reproductive system. Whereas isthmus is part of Fallopian tube which carries an egg from the ovary to the uterus.

27. Option (1) is correct. Explanation : Mendel’s Law of independent assortment holds good for genes situated on the non-homologous chromosome. According to law of independent assortment, when two or more characteristics are inherited, individual’s hereditary factors assort independently of one another during gamete production, giving different characters an equal opportunity of occurring together. It can be illustrated by the dihybrid cross (a cross between two truebreeding parents) that express different traits for two characteristics. When the genes are on separate chromosome, the two alleles of one gene (A and a) will segregate into gametes independently of the two alleles of the other gene (B and b). Equal numbers of four different gametes will form AB, aB, Ab, ab. But if the two genes are on the same chromosome, then they will be linked and will segregate together during meiosis, producing only two kinds of gametes. Homologous chromosomes are essentially similar in size but not identical. Each carries the same genetic information in same order but the alleles for each trait may not be the same.

Homologous chromosomes are essentially similar in size but not identical. Each carries the same genetic information in same order but the alleles for each trait may not be the same. Extra nuclear genetic elements (also called as plasmid) shows the pattern of maternal inheritance.

28. Option (2) is correct. Explanation : In-vitro fertilisation, cryopreservation and tissue culture are all ex-situ conservation methods. Ex-situ (off site) conservation is a set of conservation techniques which involves the transfer of a target species away from its native habitat. At present, gametes of threatened species can be preserved in viable conditions (at very low temperature −196°C) for longer duration by cryopreservation method. Fertilisation can be achieved in laboratory through the process of in-vitro fertilisation, and species can be propagated by tissue-culture method which is a plant breeding principle.

29. Option (1) is correct. Explanation : Variations during mutations of meiotic recombinations are random and directionless. Hugo de Vries proposed mutation theory on the basis of his work on evening primrose. He stated that mutations are sudden, heritable and persistent in successive generation. He contradicted Darwinian variations that are small and directional.

30. Option (1) is correct. Explanation : (a)  Gamete intrafallopian tube transfer can be used if Fallopian tubes are functioning normally.

72

OSWAAL CUET (UG) Sample Question Papers, BIOLOGY (b) Zygote intra-Fallopian transfer is a technique in which a woman’s egg is fertilised outside the body, then implanted in one of her Fallopian tubes. (c) Intra-cytoplasmic sperm injection technique is done for those infertile couples where there is some problem with male fertility or who have a history of failed IVF attempts. In ICSI, a viable single sperm is injected into the mature egg and the egg is fertilised (d) Intrauterine insemination is the placing of sperm into a woman’s uterus when she is ovulating. Insemination is performed at the time of ovulation, usually within 24–36 hours after the LH surge is detected, or after the “trigger” injection of human chorionic gonadotropin (hCG) is administered.

31. Option (2) is correct. Explanation : Approximately 2–10% of the solar energy that falls on the leaves of a plant is converted into chemical energy by the process of photosynthesis.

32. Option (3) is correct. Explanation : Monohybrid cross with incomplete dominance shows both genotypic and phenotypic ratio as same (1 : 2 : 1).

33. Option (3) is correct. Explanation : The theory of spontaneous generation (also called abiogenesis) was first suggested by Aristotle in his book “On the Generation of Animals” around 350 B.C. According to this theory, life can arise from non-living matter only. For example, organisms such as rats, flies and maggots arise within rotting meat and other decomposable items. This theory suggests that organisms do not descend from other organisms or from a parent, but only require certain environmental conditions to be fulfilled in order for their creation. This theory was disapproved by Francesco Redi, John Needham, Lazzaro Spallanzani and Louis Pasteur.

34. Option (4) is correct. Explanation : Hugo de Vries, a Dutch botanist, put forward his views regarding the formation of new species in 1901. According to him, new species are not formed by continuous variations but by sudden appearance of variations, which he named as mutations. Hugo de Vries stated that mutations are heritable and persist in successive generations. He conducted his experiments on Oenothera lamarckiana (evening primrose).

Mendel worked on Pisum sativum and gave principal of inheritance. T.H. Morgan worked on Drosophila and gave linkage theory.

35. Option (4) is correct. Explanation : The number of fungi species in the world is more than the combined total of the species of fish, amphibians, reptiles and mammals.

36. Option (3) is correct. Explanation : Urethral meatus, also known as the external urethral orifice, is the external opening or meatus of the urethra. The urethra originates from the urinary bladder and extends through the penis to its external opening called urethral meatus.

37. Option (4) is correct. Explanation : Enzyme P is restriction endonuclease that cuts the DNA into fragments while enzyme Q join the two fragments.

38. Option (4) is correct. Explanation : The organs which are present in reduced form and do not perform any function in the body but correspond to the fully developed functional organs of related animals are called vestigial organs. They are believed to be remnants of organs which were complete and functional in their ancestors. Some of these are nictitating membrane, muscles of pinna (part of external ear), vermiform appendix, caudal vertebrae (also called coccyx or tail bone), third molars (wisdom teeth), hair on body and nipples in male.

39. Option (3) is correct. Explanation : One-horned rhinoceros is specific to Kaziranga National Park, which is situated in Golaghat and Nagaon districts of (Assam). Bhitar Kanika National Park is located in Odisha and is specific for salt water crocodile. Bandipur National Park (Mysore) too is specific for tigers. Corbett National Park is situated in district Nainital (Uttarakhand) and is specific for tiger. It is the first National Park of India, which is famous for tigers.

40. Option (1) is correct. Explanation : Somaclones are genetically identical to their parent plant. These are produced through tissue culture technique.

41. Option (3) is correct.

Solutions Explanation : In ancient period, hands were used to collect food and to save themselves. Gradually men learnt to cook food; to make tools for their own purpose, this change in habit brings perfection in their hand. Similarly, there is an increase in the ability to communicate with others and develop community behaviour. Loss of tail takes a great role in course of evolution. But as in ancient period, men still eat hard nuts and hard roots (though they often take soft food also). Thus, change in diet is the most irrelevant change in the evolution of man.

42. Option (3) is correct. Explanation : Golden Rice is a new type of rice that contains beta-carotene (provitamin A), which is converted into vitamin A as needed by the body and gives the grain its golden colour. It is developed through genetic engineering and produces two new enzymes that complete the beta-carotene expression in the rice grain.

43. Option (2) is correct. Explanation : In vitro fertilisation, the zygote or early embryos develop up to 8 blastomeres is transferred into the fallopian tube.

44. Option (4) is correct. Explanation : Lamarck gave first theory of evolution known as Lamarckism in which he explained inheritance of acquired characters. According to which whatever an individual acquires characters in its lifetime due to internal vital force, effect of environment, new needs and use and disuse of organs they are inherited to the next generations. After several generations, the variations are accumulated up to such extent that they give rise to new species. This theory was proved wrong by August Weismann.

73

He put forward the theory of continuity of germplasm. Weismann cut off the tails of rats for as many as 22 generations and allowed them to breed, but tailless rats were never born.

45. Option (1) is correct. Explanation : Glomus is a genus of arbuscular mycorrhizal and form a symbiotic association with plants.

46. Option (4) is correct. Explanation : Blue-green algae are not popular as biofertilisers, besides adding organic matter to the soil and increasing its fertility because they cause algal bloom in polluted water bodies and have copious mucilage that makes the fields slippery.

47. Option (2) is correct. Explanation : The first transgenic animal were mice created by Rudolf Jaenisch.

48. Option (1) is correct. Explanation : Mice were the first transgenic animal and they show high successful rate of transformation among the mammals.

49. Option (4) is correct. Explanation : Dolly, the sheep was the first cloned animal from the somatic cell.

50. Option (2) is correct. Explanation : The Rosie was the first transgenic cow created in 1997 that consist of human alpha lactalbumin gene.

qqq

SOLUTIONS OF Question Paper 1. Option (3) is correct. Explanation : Medical termination of pregnancy is also called induced abortion. It cannot be used as a contraceptive method. MTP is done to get rid from unwanted pregnancies arises due to rape, genetic abnormalities in child, or when mother has some risks by the developing child. MTP is safe up to first trimester of pregnancy, that is, 12 weeks. MTP is done in the supervision of medical practitioner under some law. MTPs are not always surgical as there are certain pills available which will help to abort the foetus. These pills act by inducing menstruation, which in turn, checks implantation of the zygote or detaches the implanted foetus.

2. Option (3) is correct. Explanation : Pleiotropy is a phenomenon in which a single gene lead to multiple effects. Sometimes, one trait will be very evident and others will be less evident, e.g., a gene for white eye in Drosophila also affect the shape of organs in male responsible for sperm storage as well as other structures. Similarly, sickle-cell anaemic individuals suffer from a number of problems, all of which are pleiotropic effects of the sickle-cell alleles. Multiple allelism is the state of having more than two alternative contrasting characters controlled by multiple alleles at a single genetic locus, for e.g., ABO blood group. Mosaicism describes the occurrence of cells that differ in their genetic component from other cells of the body. Polygeny refers to a single characteristic that is controlled by more than two genes. (it is also known as multifactorial inheritance).

3. Option (1) is correct. Explanation : Pathogens are disease causing microorganisms. It may be bacteria, virus, fungi, etc. Pathogen disrupt the normal physiology of a multicellular animal or plant. Vector is an organism that spreads a disease or parasite from one animal to another.

4. Option (1) is correct. Explanation : DNA ligase (joining or sealing enzymes) are also called genetic gum. They join two individual fragments of doublestranded DNA by forming phosphodiester bonds between them. Thus, they help in sealing gaps in DNA fragments. Therefore, they act as a molecular glue.

8

5. Option (2) is correct. Explanation : Both IMR (infant mortality rate) and MMR (maternal mortality rate) are responsible for affecting the growth rate inversely. It means decline in IMR as well as MMR will result in high population growth and vice-versa. Hence, if there is an increased IMR and decreased MMR in a population mothers are there to give births to infants whereas the survival rate of infants becomes low. Therefore, in a particular population, increased IMR and decreased MMR will hinder their growth rate.

6. Option (1) is correct. Explanation : In certain taxon of insects, 17 and 18 chromosome bearing organisms are males and females respectively. Because, insects have XO type of sex determination method. In certain insects, such as cockroach, and some roundworms lacks Y-chromosome, so that the male has only one sex chromosome, that is, ‘X’ besides autosomes. This condition in the male is designated as XO (where O means absence of one sex chromosome) and in the female it is XX.

7. Option(3) is correct. Explanation : A person with sickle cell anaemia is less prone to malaria because in sickle cell anaemia, red blood cells becomes abnormal in shape, if they inherit two faulty copies of the gene for the oxygen-carrying protein haemoglobin. The faulty gene persists because even carrying one copy of it confers some resistance to malaria. Therefore, people with normal haemoglobin A are susceptible to death from malaria but people with sickle cell trait who have one gene for haemoglobin A and other for haemoglobin S have a greater chance of surviving malaria and do not suffer adverse consequences from the haemoglobin S gene.

8. Option (4) is correct. Explanation : Bt cotton is a genetically modified (GM) plant which is resistant to insects. These plants are produced by the insertion of one or more genes from a common soil bacterium, Bacillus thuringiensis. These genes encode for the production of insecticidal proteins, and thus, genetically transformed plants produce one or more toxins as they grow.

Solutions 9. Option (2) is correct. Explanation : Extinction of passenger pigeon (Ectopistes migratorius) was due to the combined effects of their natural dramatic population fluctuations and human over-exploitation. Factors like increase in number of predatory birds, non-availability of food and bird flu virus infection did not contribute to the extinction of passenger pigeon.

10. Option (4) is correct. Explanation : Sterilisation techniques are generally foolproof methods of contraception with least side effects. Yet, this is the last option for the couples because of (i), (ii) , (iii) and (iv).

11. Option (2) is correct. Explanation : In eukaryotes, the monocistronic structural genes have interrupted coding sequences, that is, the genes in eukaryotes are split. The coding sequences or expressed sequences are defined as exons. These sequences (exons) appear in mature or processed RNA, thus exons are interrupted by introns or intervening sequences which do not appear in mature or processed RNA.

16. Option (1) is correct. Explanation : 5′-ATGAATG-3′ (coding strand) 5′-TACTTAC-3′ (complementary strand) 5′-AUGAAUG-3′ (RNA)

17. Option (2) is correct. Explanation : Colostrum, a yellowish fluid secreted by mother during the initial days of lactation, has abundant IgA antibodies. It protects infants from several diseases. It is found in saliva and tear also. IgA is the second most human immunoglobulin in serum.

18. Option (1) is correct. Explanation : ColColumn I umn II

13. Option (1) is correct. Explanation  : A retrovirus is RNA virus which is capable to synthesise DNA during infection.

14. Option (3) is correct. Explanation : Biodiversity is not uniform throughout the world. It varies with the change in latitude and altitude, it is affected by latitudinal gradients and species-area relationship.

15. Option (1) is correct. Explanation : Acrosomal reaction of the sperm occurs due to its contact with zona pellucida of the ova. The reaction that occurs in acrosome of sperm is triggered by the release of fertilizin. The main purpose of the acrosomal reaction is to start the fusion of the oocyte membrane with the sperm cell membrane allowing the combination of genetic material contained in both gametes, leading to the fertilisation of the oocyte.

Explanation

A

(iii)

Environment Protection Act came into force on November 1986 at the birth anniversary of late Prime Minister of India Smt. Indira Gandhi.

B

(iv)

Air prevention and control of pollution Act came into force in 1981 which is meant for the preservation of quality of air.

C

(i)

Water Act 1974 is meant for restoration of quality of all type of surface and groundwater.

D

(ii)

In 1987 Amendment of Air Act to include noise into pollution came into existence

12. Option (3) is correct. Explanation : Interferon, belongs to cytokinin, is a protein which enhances the role of immune system. It is produced by virus-infected cells, that inhibit reproduction of the invading virus and induce resistance to further infection. Serotonin is a neurotransmitter that leads to depression. Colostrum is present in the lactating mother’s breast milk which contains antibodies to protect the new born against disease. Histamine is also a neurotransmitter involved in inflammatory response.

75

19. Option (1) is correct. Explanation : Morula is a developmental stage between the zygote and blastocyst. It is a solid ball of 16–32 blastomeres, produced by a series of cleavage of the early embryo from which a blastula is formed.

20. Option (4) is correct. Explanation : Matthew Meselson and Franklin Stahl in 1958 performed experiments on E. coli to prove that DNA replication is semiconservative. But it had no contribution in the development of double helix model.

76

OSWAAL CUET (UG) Sample Question Papers, BIOLOGY In 1953, James Watson and Francis Crick, based on the X-ray diffraction data produced by Maurice Wilkins and Rosalind Franklin, proposed a very simple but famous double helix model for the structure of DNA. Erwin Chargaff observed that for a double-stranded DNA, the ratios between adenine and thymine and guanine and cytosine are constant and equal to one.

21. Option (4) is correct. Explanation : Inbreeding increases homozygosity, that is, state of possessing two identical alleles, one inherited from each parent. It is necessary if we want to evolve a pure line in any animal. Inbreeding exposes harmful recessive genes that are eliminated by selection. It also helps in accumulation of superior genes and elimination of less desirable genes.

22. Option (3) is correct. Explanation : Out of the given options, bone takes the longest time for biodegradation, whereas cotton, paper and jute can be easily degraded by the action of microbes. Bone is made up of a protein fraction, mineral fraction and organic compounds.

23. Option (4) is correct. Explanation : The major hormones secreted by the human placenta are : Human chorionic gonadotropin (hCG), estrogen, progesterone and human placental lactogen. LH (luteinising hormone) is produced by gonadotropic cells in the anterior pituitary gland. This hormone is considered a gonadotrophic hormone because of its role in controlling the function of ovaries in females and testes in males, which are known as the gonads.

24. Option (3) is correct. Explanation : In humans, chromosome 1 has highest genes (~2,968) and the Y has the fewest (~231) genes.

25. Option (2) is correct. Explanation : Hilsa, sardines, mackerel and pomfrets are some of the marine (i.e., ocean or sea) fishes. Whereas Catla, rohu and common carp are freshwater fishes.

26. Option (4) is correct. Explanation : Mystus, Mangur and Mrigala are freshwater fishes whereas Mackerel is the only marine fish in the option.

27. Option (4) is correct. Explanation : E. coli cloning vector pBR322 showing restriction sites (Hind III, EcoR I, BamH I, Sal I, Pvu II, Pst I, Cla I), ori and antibiotic resistance genes (ampR and tetR). Rop codes for the proteins involved in the replication of the plasmid.

28. Option (2) is correct. Explanation : Apomixis refers to the formation of seeds without fertilisation. The embryos are genetically identical to the parental plant. Parthenocarpy is the process of formation of fruits without fertilisation. Sexual parts are involved in vegetative propagation and sexual reproduction involves fertilisation.

29. Option (2) is correct. Explanation : Melanisation in peppered moth is an example in support of the theory of natural selection. Lamarckism, the first theory of evolution, was proposed by Jean Baptiste de Lamarck. It includes four main factors : (i) Internal vital force, (ii) Effect of environment and new needs, (iii) Use and disuse of organs and (iv) Inheritance of acquired characters. Cave-dwelling animals lack pigment due to absence of light in the caves. As aquatic birds had to go to water due to lack of food, etc., they develop web between the toes. Snakes, in order to escape from mammals started living in narrow crevices. So, in order to accommodate their bodies, they lost limbs (as snakes have been evolved from lizard-like ancestors).

30. Option (3) is correct. Explanation : In artificial insemination [AI] technique, the semen collected either from the husband or a healthy donor is artificially introduced either into the vagina or into the uterus (IUI—Intra Uterine Insemination) of the female.

31. Option (2) is correct. Explanation : In agarose gel electrophoresis, the DNA fragments separate out (resolve) according to their size or length because of the sieving property of agarose gel. It means, the smaller the fragment size, the farther it will move.

32. Option (1) is correct. Explanation : In a typical complete, bisexual and hypogynous flower the arrangement of floral whorls on the thalamus from the outermost to the innermost is as follows : (i) Calyx : It is the outermost whorl of sepals. (ii) Corolla : It is a whorl of petals inside the calyx.

Solutions (iii) Androecium : It is a whorl of stamens inside the corolla. (iv) Gynoecium : It is a whorl of pistils (in the centre of the flower forming inner most whorls).

33. Option (2) is correct. Explanation : Lichens can be used as an atmospheric pollution indicator because they do not grow in areas that are polluted. They are sensitive (especially phycobiont) to oxides of nitrogen and sulphur, hence unable to synthesise organic food and do not grow well. Therefore, lichens grow in areas where sulphur dioxide pollution is very less (e.g., in Manali and Darjeeling). Lepidoptera is an insect order. Lycopersicon is scientific name of tomato. Lycopodium is an example of pteridophyte.

34. Option (1) is correct. Explanation : Analogous structures are not anatomically similar structures though they perform similar functions. Hence, analogous structures are a result of convergent evolution, different structures evolving for the same function and hence having similarity.

35. Option (4) is correct. Explanation : EcoRI comes from Escherichia coli RY 13. In EcoRI, the letter ‘R’ is derived from the name of strain. Roman numbers following the names indicate the order in which the enzymes were isolated from that strain of bacteria.

36. Option (2) is correct. Explanation : External fertilisation occurs outside the female body, usually in water. It takes place in most aquatic organisms, such as a majority of algae and fishes as well as amphibians. During external fertilisation, organisms release large number of gametes in the surrounding medium to increase the chances of syngamy. A major disadvantage is that the offsprings are extremely vulnerable to predators threatening their survival up to adulthood.

37. Option (4) is correct. Explanation : Divergent evolution occurs in the same structure which have developed along different directions due to adaptation to different needs, examples, forelimbs, heart, brain of vertebrates. Eyes of octopus, bat and cheetah are examples showing convergent evolution.

77

38. Option (1) is correct. Explanation : Wine and beer are produced without distillation whereas whisky, brandy and rum are produced by distillation of the fermented broth.

39. Option (2) is correct. Explanation : Gene expression, which results in the formation of a polypeptide, can be regulated at several levels. In eukaryotes, the regulation occurs at transcriptional level (formation of primary transcript), processing level (regulation of splicing), and transport of mRNA from nucleus to the cytoplasm, and translational level. While in prokaryotes, control of the rate of transcriptional initiation is the predominant site for control of gene expression.

40. Option (2) is correct. Explanation : In recent years, ex-situ conservation has advanced by preserving the gametes of threatened species in viable and fertile condition for long periods using cryopreservation techniques.

41. Option (3) is correct. Explanation : The bulbs are underground condensed shoots which have one or more buds that give rise to new plants. Onion is vegetatively propagated by bulbs.

42. Option (3) is correct. Explanation : Palaeontological evidences for evolution refer to the evidences from fossils. Fossils are the naturally preserved remains or traces of organisms that lived in the geologic past. These organisms are preserved in the form of skeletal, foot prints, trails, casts and moulds, resin (amber), etc. The study of fossils is called palaeontology. Development of embryo refers to embryological evidences for evolution. Whereas, homologous and analogous organs provide evidences for comparative anatomy and morphology.

43. Option (4) is correct. Explanation : Biochemical oxygen demand (BOD) is estimated by measuring the amount of oxygen consumption or is a measure of the organic pollution of water. It refers to the amount of dissolved oxygen required to decompose the organic matter in waste water. A high BOD indicates heavy pollution with little oxygen remaining for fish.

78

OSWAAL CUET (UG) Sample Question Papers, BIOLOGY

44. Option (3) is correct. Explanation : In polymerase chain reaction, polymerisation or extension step is catalysed by Taq polymerase enzyme. PCR is carried out in the following three steps (i) Denaturation : The double-stranded DNA is denatured by applying high temperature of 95°C–97°C for 15 seconds. Each separated single strand now acts as template for DNA synthesis. (ii) Annealing : Two sets of primers are added which anneal to the three ends of each separated strand. Primers act as initiators of replication. (iii) Extension : DNA polymerase extends the primers by adding nucleotides complementary to the template provided in the reaction. A thermostable DNA polymerase (Taq DNA polymerase) is used in the reaction which can tolerate the high temperature of the reaction. All these steps are repeated many times to obtain several copies of desired DNA.

45. Option (4) is correct. Explanation : Evolution of life shows that life forms had a trend of moving from water to land. The early vertebrates were fishes (which lived in only water). Some fishes gradually changed into amphibians which can live both

on land and in water. Certain amphibians then transformed into the reptiles (live on land) and some of the latter finally evolved into birds (can fly) and then mammals. Thus, showing life forms moved from water to land.

46. Option (4) is correct. Explanation : The altitude of an area can impact like climate that are mostly rainy and have low temperature while the altitude of an area impact the ultimate with respect to equator and poles.

47. Option (4) is correct. Explanation : Lithosphere is concerned with land, hydrosphere with water bodies and atmosphere is concerned of space. Hydrological cycle circulates in all these three subsystems.

48. Option (3) is correct. Explanation : More will be the water holding capacity of the soil, less will be the air spaces less soil aeration.

49. Option (1) is correct. Explanation :Edaphology is the study of soil useful for plant growth.

50. Option (2) is correct. Explanation : Euryhaline animals like salmon can adopt wide range of salinity as they are found in areas where river and sea meets.

qqq

SOLUTIONS OF Question Paper 1. Option (1) is correct. Explanation : The whole plant body of maize plant including shoot tip cells remains in diploid (2n) condition. As the microspore mother cell is a part of reproductive organ, the chromosome number in these cells will remain same as the individual, that is, 2n = 20. These microspore mother cells are further responsible for producing male gametes, that is, haploid (n) by reduction division.

2. Option (4) is correct. Explanation : ZZ / ZW type of sex determination is seen in birds, reptiles and fish. Thus, peacock shows ZZ/ZW sex determination type. In this type, female has heteromorphic (ZW) sex chromosomes and the male has homomorphic (ZZ) sex chromosomes. In Platypus, the sex determination is of XX-XY type in which both males and females have ten sex chromosome each. The male has XY, XY, XY, XY, XY and female has XX, XX, XX, XX, XX. In snails, the sex determination is environmentally induced, while in cockroaches it is of XX-XO types. In this type, Y-chromosome is absent. In this, the presence puffs unpaired X-chromosomes determines the masculine sex.

3. Option (3) is correct. Explanation : AIDS, caused by HIV, is a sexually transmitted disease. It cannot be transmitted by shaking hands with the AIDS infected person. Transmission of HIV-infection generally occurs by the following : (i) Sexual contact with infected person. (ii) Transfusion of contaminated blood and blood products. (iii) Sharing infected needles as in case of intravenous drug abusers. (iv) Infected mother to her child through placenta.

4. Option (4) is correct. Explanation : According to Allen's rule, the mammals from colder climates or areas show shorter extremities like ears and limbs as compared to the mammals of warm region. This help to minimize heat loss and maintain homeostasis.

9

5. Option (2) is correct. Explanation : Both angiosperms and humans exhibit internal fertilisation and remain reproductively active throughout their reproductive phase. It means that the formation of egg takes place not only once, but also many times in their life time. In humans, egg released from the ovary reaches the Fallopian tube (for fertilisation) and from there to uterus for implantation. So, the egg is considered as motile not stationary. Similarly, in flowering plants (angiosperms), the gametes are non-motile cells within gametophytes, but for the fusion to take place, the non-motile male gametes are carried to female gamete by pollen tubes.

6. Option (2) is correct. Explanation : Pedigree analysis is the study of an inherited trait in a group of related individuals. It helps to determine the pattern and characteristics of the trait which includes its mode of inheritance, age of onset, and phenotypic variability. Mendelian inheritance in humans is difficult to study, therefore it is enhanced by analysis of family pedigrees or the results of mating that have already occurred. By analysing a pedigree, we may be able to predict how the trait is inherited. It is a visual tool for documenting the biological relationship in families and to determine the mode of inheritance (dominant, recessive, etc.,) of genetic diseases. Quantitative trait, polygenic trait and maternal traits are not studied by pedigree analysis. Quantitative traits are those measurable phenotypic traits that depends on the cumulative actions of many genes and the environment for e.g., crop yield, weight, gain in animals, IQ, etc. Polygenic traits occur when a trait is controlled by more than one gene. This means that each dominant allele adds to the expression of the next dominant allele. Maternal traits are those inherited traits which are expressed from the maternal parent to the subsequent offsprings.

7. Option (1) is correct.

80

OSWAAL CUET (UG) Sample Question Papers, BIOLOGY Explanation : The Ti plasmid (tumour-inducing plasmid) of Agrobacterium tumefaciens has been modified (does not cause tumour) and used as a cloning vector. The Ti plasmid integrates a segment of its DNA, termed T-DNA into the chromosomal DNA of its host plant cells.

8. Option (1) is correct. Explanation : Breed is a group of animals that are related by descent and are similar in most characters like general appearance, features, size, configuration, etc. The other three options are incorrect. Race is used in classification system to categorise humans into large and distinct populations or groups based on anatomical, cultural, ethnic or geographical differences. Variety is a genetically and morphologically distinct subset of a species that is geographically isolated from other populations within that species. A species is defined as a group of individuals that potentially interbreed in nature.

9. Option (4) is correct. Explanation : Adult filament of multicellular alga have haplontic life cycle in which the meiotic division occurs after the formation of zygote. So, the filament of this alga has haploid vegetative cells and haploid gametangia. A multicellular gametophyte or gametangia which is haploid (n) alternates with a multicellular sporophyte which is diploid (2n). A mature sporophyte produces spores (haploid cells) by meiosis, a process which reduces the number of chromosomes to half, i.e., from 2n to n.

10. Option (1) is correct. Explanation : Mendel's law of dominance states that on crossing two parents of pure line with contrasting traits, the progeny exhibits character of only one type of trait which is dominant.

11. Option (3) is correct. Explanation : Several South Indian states raise 2–3 crops of rice annually. The agronomic feature that makes this possible is early yielding rice variety. These varieties of rice are a group of crops which are created intentionally during the Green Revolution to increase the global food production.

12. Option (3) is correct. Explanation : Pectin and cellulose are the main components of the cell wall of plant cells. These can be removed by digestion with a combination of pectinase and cellulase enzymes to isolate the protoplast.

13. Option (2) is correct. Explanation : Autecology is the study of interrelationship of the organisms of a species to biotic or abiotic environment.

14. Option (3) is correct. Explanation : The pollen grains represent the male gametophytes. As the anthers mature and dehydrate, the microspores dissociate from each other and develop into pollen grains. So, embryo sac is to ovule as pollen grain is to an anther.

15. Option (1) is correct. Explanation : Aneuploidy is a condition in which a person has one or a few more number of chromosomes above or below the normal chromosome number. In this case, organism either gains or loses one or more chromosomes but not a complete set. Polyploidy is defined as the addition of entire set of chromosomes. It can be triploidy (3n), tetraploidy (4n), pentaploidy (5n), etc. Allopolyploidy is the polyploidy in which chromosome sets are non-homologous. In other words, we can say that the allopolyploids are derived from a stock which is heterozygous. Monosomy is the process in which one chromosome is removed from diploid set of chromosomes (2n-1).

16. Option (1) is correct. Explanation : The capacity of a cell or an explant (any part of plant taken out and grown in test tube) to grow into a whole plant is called totipotency.

17. Option (2) is correct. Explanation : It is essential to have few cloning sites in cloning vector. It is because, in order to link the alien DNA, the vector needs to have very few recognition sites for the commonly used restriction enzymes. Presence of more than one recognition site within the vector will generate several fragments, which will complicate the gene cloning. Also, the vector requires a selectable marker, which helps in identifying and eliminating non-transformants and selectively permitting the growth of the transformants.

18. Option (4) is correct. Explanation : The outermost and innermost wall layers of microsporangium in an anther are respectively, epidermis and tapetum. A typical microsporangium is generally surrounded by four-wall layers, that is, the epidermis, (outermost protective layer), endothecia, (middle fibrous layers) and the tapetum (innermost nutritive layer).

19. Option (2) is correct.

Solutions Explanation : In a DNA strand, the nucleotides are linked together by 3’→5’ phosphodiester linkage (bonds) to form a dinucleotide. More nucleotides can be joined in such a manner to form a polynucleotide chain.

20. Option (3) is correct. Explanation : Column A Column B

Explanation

A

(iv)

Aspergillus niger produces citric acid.

B

(iii)

Acetobacter aceti produces acetic acid.

C

(ii)

Clostridium butylicum produces butyric acid.

D

(i)

Lactobacillus produces lactic acid.

21. Option (2) is correct. Explanation : After completion of bio-synthetic stage, the product has to be subjected through a series of processes before it is ready for marketing as a finished product. The processes include separation & purification, which is collectively referred to as downstream processing.

22. Option (3) is correct. Explanation : World’s most problematic aquatic weed is Eichhornia (water hyacinth) because its growth cannot be controlled and it covers the entire pond or lake and impacts water flow. Eichhornia (water hyacinth) is a free-floating perennial plant which has been introduced in India to check water pollution, as this weed absorbs mercury, cadmium, lead and nickel from sewage water. It is the world’s most problematic aquatic weed as it is an invasive species and can form thick mats. If these mats cover the entire surface of the pond, they can cause oxygen depletions and kills fish.

23. Option (3) is correct. Explanation : Plants use two abiotic (wind and water) and one biotic (animals) agent to achieve pollination. Majority of plants use biotic agents for pollination. Pollination by wind is more common amongst abiotic pollination. It requires the light and non-sticky pollen grains so that, they can be transported in wind currents. They often possess wellexposed stamens (so that the pollens are easily dispersed into wind currents) and large oftenfeathery stigma to easily trap air-borne pollen grains. Wind pollination is common in grasses.

81

Pollination by water is called hydrophily which is quite rare in flowering plants but occurs in aquatic plants. Zoophily is pollination through the agency of animals. Entomophily (pollination by insects) is the most common type of zoophily which occurs through the agency of animals.

24. Option (1) is correct. Explanation : It is the inheritance in which both alleles of a gene are expressed in a hybrid.

25. Option (1) is correct. Explanation : Hepatitis-B is initially not caused by contamination of water. It is caused by the exposure to infectious agents. It can be transmitted through blood transfusion, sexual contact, saliva etc. Jaundice, cholera and typhoid are water and food borne diseases.

26. Option (2) is correct. Explanation : DNA polymerase adds deoxyribose nucleotides to the free 3′-end of the growing polynucleotide chain so that the replication of the 3′→5′ strand of the DNA molecule is continuous (growth of the new strand in 5′→3′ direction). Since, DNA-dependent DNA polymerase catalyses polymerisation only in one direction (5′→3′), discontinuous synthesis of DNA occurs in the other strand.

27. Option (1) is correct. Explanation : Edaphology is the study of soil useful for plant growth.

28. Option (4) is correct. Explanation : The first living beings were singlecelled prokaryotes, like bacteria. These living beings were present in the environment of soupy sea having abundant organic molecules. Nutritionally, they were chemo-heterotrophs.

29. Option (3) is correct. Explanation : Artificial hybridisation is one of the major methods of crop improvement programme. This cross will make sure that only the desired pollen grains are used for pollination and the stigma is protected from contamination (from unwanted pollen). This is achieved by emasculation and bagging techniques.

82

OSWAAL CUET (UG) Sample Question Papers, BIOLOGY If the female parent produces unisexual flowers; there is no need for emasculation (a process of removal of anther). The female flower buds are bagged before the flowers open. When the stigma becomes receptive, pollination is carried out using the desired pollen and the flower rebagged. This protects them from contamination by unwanted pollen grains. When the female parent bears bisexual flowers, removal of anthers from the flower bud before the anther dehisces is necessary.

30. Option (2) is correct. Explanation : Splicing is the removal of intervening sequences of introns and the joining of expressing sequences or exons in a specific order during transcription.

31. Option (4) is correct. Explanation : In the given list, influenza (ii) and small pox (iv) are infectious diseases. Infectious diseases can be spread from one person to another, through contact with body fluids, by aerosols (through coughing and sneezing), or via a vector whereas, cancer and allergy are non-communicable diseases and cannot spread from infected persons to a healthy person.

32. Option (1) is correct. Explanation : Retrovirus is commonly used as a vector for introducing a DNA fragment in human lymphocytes.

33. Option (2) is correct. xplanation : Secretion of seminal vesicle E (paired), prostate gland (unpaired) and bulbo-urethral glands or Cowper’s glands (paired) constitute the seminal plasma. It contains various proteins and fructose as energy suppliers for sperm motility and is also responsible for making the largest proportion of the alkaline buffer.

34. Option (1) is correct. Explanation : If the base sequence of a codon in mRNA is 5′-AUG-3′, the sequence of tRNA pairing with it must be 5′-UAC-3′.

5' - AU G - 3 ' ( C o d e i n m R N A ) 5'- U A C - 3 ' ( t R N A ) 5'

AUG UAC

3' mRNA

5' 3' tRNA

35. Option (4) is correct. Explanation : Some organisations and multinational companies exploit or get patent for the biological resources of other nations without proper authorization from the concerned countries. This is called biopiracy.

36. Option (3) is correct. Explanation : Haemozoin is a toxin released by Plasmodium species, which is responsible for the chill and high fever recurring every three to four days. To continue their life-cycle, Plasmodium enters the human body as sporozoites and multiply within the liver cells, resulting in the rupture of the RBCs. The ruptured RBCs is associated with release of a toxic substance, haemozoin. Streptococcus produces streptomycin and streptococcal pyrogenic exotoxin which shows haemolytic condition. Haemophilus produces cytolethal distending toxin (HdCDT) which inhibit mammals cell proliferation.

37. Option (4) is correct. Explanation : The endometrium is the innermost of the three layers of tissue that lines the uterus (the expandable organ in which a foetus is nourished and develops before birth).

38. Option (2) is correct. Explanation : The DNA-dependent RNA polymerase helps in transcription by catalysing the polymerisation in only one direction (i.e., 5′→3′).

39. Option (1) is correct. Explanation : Transgenic animals are animals whose DNA is manipulated to possess and express an extra (foreign) gene e.g., Rosie transgenic cow.

Solutions 40. Option (3) is correct. Explanation : Appendix is a vestigial organ (useless), not a lymphoid organ. It is muscular structure attached to the large intestine in the human body. It is an evolutionary remnant of a cecum that was once large and had vital digestive functions, which do not work the same way anymore. Lymphoid organs are those organs where origin and/or maturation and proliferation of lymphocytes occur. Lymphoid organs are of two types : primary and secondary. Lymphoid Organs

83

Cell mediated immune response, mediated by T-lymphocyte is able to differentiate between self and non-self cells/organ. This type of immune response recognises the body’s non-cells or other tissue or organs from other individual as foreign antigen and cause rejection of the graft.

44. Option (3) is correct. Explanation : Crystals of Bt toxin produced by some bacteria do not kill the bacteria themselves because toxin is inactive.

45. Option (3) is correct. Primary Lymphoid Organs

Secondary Lymphold Organs

It includes bone marrow and thymus where B and T-cell lymphocytes mature and acquire their antigenspecific receptor.

The organs where lymphocytes interact with the antigen and proliferate to become effector cells, e.g., spleen, lymph nodes, tonsils, Peyer’s patches of small intestine

41. Option (4) is correct. Explanation : Sickle cell anaemia is a genetic disorder. It is s an inherited form of anaemia where in the haemoglobin is defective resulting in crescent shaped RBCs. It cannot be treated with iron supplements. But it offers resistance against malaria. In African population, where malaria is rampant, sickle cell anaemia is a protective disorder. Hence, all the above are true.

42. Option (1) is correct. Explanation : Breast-feeding is one of the temporary and natural contraceptive methods which reduce the secretion of oestrogen hormone and thereby suppress ovulation (release of egg) and cause amenorrhoea. The contraceptive effect of the lactational amenorrhoea method is a result of increased levels of prolactin. In this condition, production and secretion of gonadotropin-releasing hormones are inhibited resulting in low secretion of oestrogen. As ovulation cannot occur without a surge in oestrogen levels, and if a woman does not ovulate, pregnancy is prevented.

43. Option (4) is correct. Explanation : Transplantation is the replacement of a diseased organ or tissue of an individual with healthy organ or tissue of same or another individual. These transplants or graft gets rejected, if it is recognised as foreign antigen by the body’s immune systems.

Explanation : Oxygen was absent in the atmosphere at the time of origin of life. Hydrogen atoms were most numerous and most reactive in primitive atmosphere. First hydrogen atoms combined with all oxygen atoms to form water and leaving no free oxygen. Thus, primitive atmosphere was reducing atmosphere (without free oxygen) unlike the present oxidising atmosphere (with free oxygen).

46. Option (3) is correct. Explanation : There are more than 30 biodiversity hot-spots in the world. India have 4 biodiversity hot-spots.

47. Option (1) is correct. Explanation : Due to habitat loss, animals migrate and loses their favourable condition, food, temperature, water which makes it difficult for them to survive. Hence, they become extinct.

48. Option (1) is correct. Explanation : Conservation of plants and species within their suitable boundary of natural surroundings.

49. Option (4) is correct. Explanation : Dolphin is the national aquatic animal of India.

50. Option (4) is correct. Explanation : The species of an organism lasting with the change in suitable condition of survival with time. Such species are called critically endangered.

qqq

SOLUTIONS OF Question Paper 1. Option (4) is correct. Explanation : Nuisance growth of aquatic plants and bloom forming algae in natural water bodies is generally due to the presence of high concentration of dissolved organic and inorganic nutrient materials. Phosphorus favours the growth of aquatic weed plants like Eichhornia (water hyacinth) and certain bluegreen algae.

2. Option (3) is correct. Explanation : Bt toxin is produced by a bacterium called Bacillus thuringiensis (Bt). The Bt toxin protein exist as inactive pro-toxins, but once an insect ingests the inactive toxin, it is converted into an active form of toxin due to the alkaline pH of the gut which solubilise the crystals.

3. Option (1) is correct. Explanation : Crossing over separates genes away from each other. So, the physical distance between the two genes and percentage of recombination shows a direct relationship. More the distance between two genes, more is the frequency of crossing over between them and hence more is the percentage of recombination.

4. Option (3) is correct. Explanation : ICSI (intra-cytoplasmic sperm injection) is a method in which embryo is formed in the laboratory and sperm is directly injected into the ovum.

5. Option (2) is correct. Explanation : The bones of fore-limbs of whale, bat, cheetah and man are similar in structure, because they share a common ancestor.

6. Option (4) is correct. Explanation : Waste water consists of three types of impurities - suspended particles (solids), colloidal impurities and dissolved material (inorganic and organic) like calcium, ammonia, toxic material, phosphate, sodium, calcium, and nitrate.

10

Out of them, suspended solids (such as sand, silt, and clay impurities) are easily removed in sewage treatment plants. Most of these solids tends to settle if waste water is left undisturbed for some time.

7. Option (3) is correct. Explanation : As a method of cellular defence, RNAi takes place in all eukaryotic organisms. This method involves silencing of a specific mRNA due to the complementary dsRNA molecule that binds to and prevents translation of the mRNA (silencing).

8. Option (1) is correct. Explanation : Heroin, commonly called smack, is chemically diacetyl morphine. It is a white, odourless, bitter, crystalline compound which is obtained by acetylation of morphine, extracted from the latex of poppy plant (Papaver somniferum). Leaves of Cannabis sativa (commonly called bhang) produces cannabinoids. Dhatura flower produces tropane alkaloids (mainly sco polamines, hyoscyamine and atropine). Atropine reduce salivation and bronchial secretions before surgery. It is also used to dilate the pupil before eye exams and to relieve pain which is caused by swelling and inflammation in the eye. Cocaine is extracted from the dried leaves and young twigs of Erythroxylum coca.

9. Option (3) is correct. Explanation : Linkage may be defined as the relationship between genes on the same chromosome that causes them to be inherited together, therefore it will not result in variations among siblings. In linkage, there is a tendency to maintain the parental gene recombination except for occasional crossovers. Independent assortment of genes means that allele pair separate during the formation of gametes independently; it means that traits are transmitted to offspring independently of one another. Crossing over is the exchange of genetic material between homologous chromosomes. It is one of the final phases of genetic recombination. Mutation is the sudden inheritable change in genetic material of an organism which transfers to the next generation.

Solutions 10. Option (2) is correct. Explanation : In zygote intra-Fallopian transfer, the zygote is collected from a female donor and placed in Fallopian tubes rather than directly in the uterus. ZIFT is the result of combining IVF and GIFT.

11. Option (4) is correct. Explanation : The green scum seen in the fresh water bodies mainly consists of green algae and blue-green algae, while the red algae is mostly marine.

12. Option (2) is correct. Explanation : ADA stands for Adenosine deaminase.

13. Option (4) is correct. Explanation : A. Trophoblast is the peripheral cells of the blastocyst, which attach the zygote (fertilised ovum) to the uterine wall and become the placenta and the membranes that nourish and protect the developing organism. B. Cleavage is the repeated mitotic division of cells in the early embryo. C. Inner cell mass forms within the blastocyst, prior to its implantation within the uterus. It differentiates to form embryo. D. Implantation is a process of attachment and invasion of the uterus endometrium by the blastocyst (conceptus).

14. Option (4) is correct. Explanation : The Miller-Urey experiment created primitive earth condition in the laboratory and demonstrated the origin of first form of life from pre-existing non-living inorganic molecules. These primitive earth conditions include high temperature, volcanic storms and reducing environment containing methane (CH4), ammonia (NH3), hydrogen (H2) and water (H2O). They ultimately found that a large number of simple organic compounds including some amino acids such as alanine, glycine and aspartic acid can be synthesised during chemical origin of life.

15. Option (2) is correct. Explanation : Reserpine is an alkaloid found in the roots of Rauwolfia serpentina and R. vomitoria. It is used with or without other medications to treat high blood pressure (hyper-tension). It lowers blood pressure by slowing down the nervous system. This allows blood vessels to relax and dilate (widen), which helps heart beats more slowly and improves blood flow.

16. Option (4) is correct.

85

Explanation : A recombinant DNA molecule can be produced in the absence of Escherichia coli. Restriction enzymes and DNA ligases can be used to make a stable recombinant DNA molecule, with DNA fragments that have been spliced together from two different organisms.

17. Option (2) is correct. Explanation : Here is this case, the women is a carrier. Both the son and daughter will inherit the X-chromosome but only the son will be diseased.

18. Option (3) is correct. Explanation : Polyspermy may be defined as the fertilisation of an ovum by more than one sperm. During fertilisation, a sperm comes in contact with the zona pellucida layer of the ovum and induces changes in the membrane that block the entry of additional sperms. Thus, it ensures that only one sperm can fertilise an ovum.

19. Option (2) is correct. Explanation : In-vitro fertilisation, cryopreservation and tissue culture are all exsitu conservation methods. Ex-situ (off site) conservation is a set of conservation techniques which involves the transfer of a target species away from its native habitat. At present, gametes of threatened species can be preserved in viable conditions (at very low temperature −196°C) for longer duration by cryopreservation method. Fertilisation can be achieved in laboratory through the process of invitro fertilisation, and species can be propagated by tissue-culture method which is a plant breeding principle.

20. Option (3) is correct. Explanation : Deoxyribonuclease enzyme is not used in the process of isolating DNA from bacteria as this enzyme causes the lysis of DNA molecules.

21. Option (2) is correct. Explanation : Fact

22. Option (2) is correct. Explanation : Locality factors or environment of soil is described by the term Edaphic factor.

23. Option (4) is correct. Explanation : Dolly, the sheep was the first cloned animal from the somatic cell.

86

OSWAAL CUET (UG) Sample Question Papers, BIOLOGY

24. Option (3) is correct. Explanation : The genetic constitution of female child is XX.

25. Option (2) is correct. Explanation : A. Sperm head contains nucleus with densely coiled chromatin fibres surrounded by acrosome. Nucleus transfers the genetic material to next generation. B. Middle piece of sperm contains large number of mitochondria to provide energy needed for the movement. C. Acrosome present in the cap of sperm contains hydrolytic enzymes which help sperm to penetrate the egg. These enzymes break down the outer membrane of the ovum, called the zona pellucida, allowing the haploid nucleus in the sperm cell to join with the haploid nucleus in the ovum. D. Tail of sperm helps in propelling or swims the sperm cell forwards to meet the egg.

26. Option (2) is correct. Explanation : As the changes in an ecological succession are orderly, sequential parallel with the changes in the physical environment. Therefore, sequence of communities of a primary succession in water is phytoplankton, free-floating hydrophytes, rooted hydrophytes, sedges, grasses, and trees.

27. Option (3) is correct. Explanation : The heaviest molecule in the protoplasm is DNA. DNA is a compound of very high molecular weight (over one million), it has a giant molecule made of smaller molecules linked together, but its molecular weight is variable.

28. Option (2) is correct. Explanation : If both parents have sickle cell trait, there is a 50% chance with each pregnancy that the baby will have sickle cell trait. A child with sickle cell anemia appears normal at birth.

29. Option (4) is correct. Explanation : Functioning of our ecosystem provides us services like cycling of nutrients, prevention of soil erosion, pollutant absorption and reduction of the threat of global warming, etc.

30. Option (4) is correct. Explanation : DTP vaccine can prevent diphtheria, tetanus, and pertussis. Diphtheria and pertussis spread from person to person. Tetanus enters the body through cuts or wounds. Diphtheria can lead to difficulty breathing, heart failure, paralysis, or death.

31. Option (3) is correct Explanation : N-glycosidic linkage in DNA, is the nitrogen carbon linkage between the nitrogen of purine or pyrimidine bases and the carbon of the sugar group. This bond does not provide stability.

32. Option (3) is correct. Explanation : Statement (i) and (ii) are correct regarding female gametophyte of flowering plant. The female gametophyte or embryo sac is located inside the nucellus, enclosed within the integuments. In a majority of flowering plants, one of the megaspore is functional while the other three degenerates. Three repeated mitotic divisions of the functional megaspore results in the formation of seven-celled or eightnucleate embryo sac. Six of the eight nuclei are organised at the two poles. Three cells grouped at micropylar end forms egg-apparatus and 3 at the chalazal end form antipodal cells. The large central cell at the centre has two polar nuclei. The meiotic divisions in the formation of embryo sac are strictly free nuclear, that is nuclear divisions are not followed immediately by cell-wall formation. Gametophyte is situated at micropylar end not at chalazal end.

33. Option (3) is correct. Explanation : Variations in number of individuals in a population can be expressed as population density and population size. A population of younger individual than older individuals will show positive growth in future (after some years.), i.e., it will increase after some time.

34. Option (4) is correct. Explanation : Regulatory proteins, the accessory proteins that interact with RNA polymerase and affect its role in transcription. It controls the functions of structural genes and are called regulatory genes. Promoters, terminators, operators and repressor are some important regulatory genes. They can act both as activators and as repressors.

35. Option (3) is correct. Explanation : All the four whorls of the plant with their relative position in flower can be indicated through following diagram. Sepals collectively form a whorl, called as calyx while technically the carpel is known as gynoecium. The floral whorls formed by petals and stamens are called as corolla and androecium, respectively. Androecium Gynoecium Corolla Calyx Pedicel Calyx Flower

Corolla

Androecium Gynoecium

Solutions 36. Option (3) is correct. Explanation  : Allen’s law states that extremities (e.g., ears, tails, etc.) of animals that live in very cold climates become progressively smaller. Cope’s law states that there is a tendency for animals to increase in size during the long course of evolution. Dollo’s law states that evolution is irreversible. Bergman’s law states that warm blooded animals become larger in the northern and colder parts of their range.

37. Option (2) is correct. Explanation : dN dN =rN =rN dt dt rr=0.028-0.008=0.02 =0.028-0.008=0.02 =2015-2005=10 dt years dt=2015-2005=10 years N =N =14 million million N= N2005=14 2005

dN =change in in population population in in 10 10 years years dN=change dN =0.02×14 dN=0.02×14 dN dN=0.02×14×10 =0.02×14×10 dN dN=2.8 =2.8 million million dN = N − N 2005 dN=N2015 2015 − N 2005 N = dN + N 2005 N2015 2015=dN+N 2005 =2.8+14 =2.8+14 =16.8 =16.8 ≈≈ 17 17 mi million llion

38. Option (4) is correct. Explanation : Bt represent the bacteria Bacillus thuringiensis that makes toxic protein for some insects.

39. Option (1) is correct. Explanation : The strains of corynebacterium diphtheriae makes toxins causing serious infection ‘Diphtheria’ which leads to difficulty in breathing, heart failure, paralysis and sometimes even death.

40. Option (1) is correct. Explanation : This attachment is an essential step in the synthesis of protein. This attachment is brought by aminoacyl-t-RNA synthetase.

41. Option (3) is correct. Explanation : Innate immunity is a non-specific type of defence, that is present at the time on birth.

42. Option (1) is correct. Explanation: Heroin is synthesised by acetylation of the two hydroxyl groups of morphine with acetyl chloride, hence its other names, diacetylmorphine or diamorphine.

87

43. Option (1) is correct. Explanation : Lobe fish is an example for link species. About 350 mya, fish with stout and strong fins could move on land and go back to water. These were called lobe and they evolved into the first amphibians that lived on both land and water (e.g., Coelacanth). Coelacanth is an ancient fish and refers to as a living fossil. Dodo is an extinct flightless bird. Sea weed is a multi-cellular benthic marine algae and chimpanzees are the closest living relatives of humans.

44. Option (1) is correct. Explanation : Pistil is the female reproductive part (gynoecium) of a flower. It is centrally located and consists of ovary (a swollen base), which contains the potential seeds or ovules; style (a stalk), arising from the ovary; and a pollen-receptive tip, the stigma. Inside ovary, it is attached to a parenchymatous cushion called placenta, either singly or in cluster.

45. Option (2) is correct. Explanation : During the sexual reproduction, the events which take place after the formation of zygote are called post-fertilisation events. The process of embryo development from the zygote (called embryogenesis) takes place after the fertilisation. In all flowering plants, the zygote is formed inside the ovule. In ovule, the zygote divides several times to form an embryo.

46. Option (3) is correct. Explanation : The primary effluent is transferred into large aeration tanks where it is constantly agitated mechanically and the air having oxygen is pumped into it which allows fast growth of useful aerobic microbes into flocs (masses of bacteria associated with fungal filaments to form mesh-like structures).

47. Option (3) is correct. Explanation : The effluent after primary treatment is passed into the settling tank where the bacterial 'flocs' are allowed to settle down and this sediment is called Activated sludge. A small part of the activated sludge is pumped back into the aeration tank to serve as the inoculum(starter).

48. Option (1) is correct. Explanation : The activated sludge is pumped back into the aeration tank to serve as the inoculum(starter).

88

OSWAAL CUET (UG) Sample Question Papers, BIOLOGY

49. Option (1) is correct. Explanation : During this digestion, bacteria produce a mixture of gases such as methane, hydrogen sulphide, and carbon dioxide. These gases form biogas and can be used as a source of energy. The treated effluent from the secondary treatment plant is generally released into natural water bodies like rivers and streams.

50. Option (3) is correct. Explanation : Biogas production from cow dung was developed in India largely due to the efforts of IARI, khadi and village Industries commission.

qqq

Attention ambitious achievers! Prepare to conquer the CUET(UG) exam with Oswaal360! Unlock the path to success with double the value of your purchase and gain access to: Captivating Video Lectures

Challenging Practice Tests

Realistic Mock Tests

Don't miss out on our upcoming series of Exclusive Mock Tests, designed to boost your confidence and ensure success. Prepare for other competitive exams like NEET, JEE, and more, and pave the way to a bright future.

Scan now and Unlock the door to your dream college with Oswaal360 by your side!

CUETBI

Share your review of the books and help your fellow students & juniors.

Biology

Scan to Review

Attention ambitious achievers! Prepare to conquer the CUET(UG) exam with Oswaal360! Unlock the path to success with double the value of your purchase and gain access to: Captivating Video Lectures

Challenging Practice Tests

Realistic Mock Tests

Don't miss out on our upcoming series of Exclusive Mock Tests, designed to boost your confidence and ensure success. Prepare for other competitive exams like NEET, JEE, and more, and pave the way to a bright future.

Scan now and Unlock the door to your dream college with Oswaal360 by your side!

CUETBI

Share your review of the books and help your fellow students & juniors.

Biology

Scan to Review